Download as pdf or txt
Download as pdf or txt
You are on page 1of 78

IDMP TEST SERIES-2023

High Yielding PRELIMS TEST SERIES

Test-23
Science & Tech, Environment Revision
Topic Covered:
 Bio Technology + IPR
 Health
 Energy, research and development
 Environment revision

IDMP-2023
Contact us :info@onlyias.com

OnlyIAS Nothing Else Visit :dpp.onlyias.in


Contact : +91-7007 931 912

Q.1) Consider the following statements regarding Which of the statements given above is/are correct?
Science and Engineering Research Board (SERB): a) 1 and 2 only
1. SERB is a non-statutory body, working under the b) 2 and 3 only
Department of Science and Technology to promote c) 1 and 3 only
basic research in science and engineering d) All of the above
2. It is chaired by the Minister of State (Independent
Charge) of the Ministry of Science and Technology Q.4) Consider the following statements regarding the
and shall have other senior government officials Laser Interferometer Gravitational-Wave
and eminent scientists as members. Observatory Project:
3. It has launched a scheme “SERB-SURE” which 1. Gravitational Waves are the 'ripples' in space-time
provides research support to active researchers travelling at the speed of light caused by some of the
belonging to state universities and colleges only. most violent and energetic processes in the Universe.
Which of the statements given above is/ are not 2. Detection of Gravitational Waves by LIGO will
correct? provide a test of Einstein's theory of general relativity
a) 2 only and information about black holes and neutron stars
b) 2 and 3 only existence.
c) 1 and 3 only 3. The proposed LIGO India project at Theni,Tamilnadu
d) 1, 2 and 3 will join the global network of gravitational wave
detectors.
Q.2) Which of the following statement is correct with Which of the statements given above is/are correct?
respect to Genetically Modified (GM) Crops? a) 1 and 2 only
a) In India, the Genetic Engineering Appraisal b) 2 and 3 only
Committee (GEAC),under the Ministry of science c) 1 and 3 only
and technology, is the apex body that allows for d) All of the above
the commercial release of GM crops.
b) GM Crops are that type of plant whose DNA has Q.5) Consider the following statements regarding
been modified through genetic engineering for Human genome:
embedding a new trait to the plant which occurs 1. A genome refers to all of the genetic material in an
naturally in the species. organism, and the human genome differs largely
c) Bt cotton and Bt Brinjal are the only GM crops that between one individual and another.
are allowed for commercial cultivation in India. 2. Human Genome Project started in 1988 by World
d) Genetically Modified technology adoption has Health Organization whose goal was the complete
reduced chemical pesticide use. mapping and understanding of all the genes of
human beings.
Q.3) Consider the following statements regarding the 3. Telomere-2-Telomere project has managed to
Medical Oxygen: sequence the remaining 8% of the human genome,
1. Oxygen can be produced in gaseous form using 92% of which was already sequenced through the
Pressure Swing Adsorption Technique. Human Genome Project (HGP).
2. The National Pharmaceutical Pricing Authority, Which of the statements given above is/ are correct?
under the Ministry of Health and Family Welfare, a) 1 and 2 only
Controls and monitors the medical oxygen prices in b) 3 only
India. c) 2 and 3 only
3. Medical Oxygen can cause Retrolenticular fibroplasia d) 1, 2 and 3
in premature infants if they are exposed to oxygen
concentrations greater than 40%.

DPP 2023 DAY 160 1


Contact us :info@onlyias.com

OnlyIAS Nothing Else Visit :dpp.onlyias.in


Contact : +91-7007 931 912

Q.6) Consider the following statements regarding Temperature Thermal Desalination (LTTD)
steel slag: technology
1. The steel slag is generated from a steel furnace Which of the statements given above is/ are correct?
burning at around 1,500-1,600 degree centigrade. a) 2 only
2. Steel slag can be used for road construction but the b) 1 and 2 only
cost per square metre of slag road is more than c) 1 and 3 only
that of bitumen road. d) 1, 2 and 3
3. Steel slag needs blasting, drilling or crushing before
using slag as road construction material. Q.9) Consider the following statements regarding
Which of the statements given above is/ are correct? Biodegradable plastics:
a) 1 only 1. Biodegradable plastics are plastics that can be
b) 2 only decomposed by the action of living organisms,
c) 2 and 3 only usually microbes completely into water and and
d) 1 and 2 only biomass
2. Biodegradable plastics are commonly produced
Q.7) Consider the following statements regarding with renewable raw materials, micro-organisms,
Calcium ions batteries: petrochemicals, or combinations of all three.
1. The calcium ion is monovalent, and hence one ion 3. All bioplastics that are derived partly or entirely
insertion will deliver one electron per ion during from biomass are biodegradable
battery operation. Which of the statements given above is/are not
2. Calcium ions can be a greener, more efficient, and correct?
less expensive energy storage alternative to a) 2 only
lithium-ions. b) 3 only
3. The disadvantages of Calcium ions are its larger size c) 1 and 3 only
and higher charge density relative to lithium ions. d) 1, 2 and 3
Which of the statements given above is/are correct?
a) 1 and 2 only Q.10) Consider the following statements regarding
b) 2 and 3 only ASPAGNIITM :
c) 2 only 1. Department of Biotechnology- National Institute of
d) 1, 2 and 3 Immunology (DBT-NII) receives ASPAGNII trademark
for India’s First Indigenous Tumour Antigen SPAG9.
Q.8) Consider the following statements regarding 2. Currently, ASPAGNII is being used in dendritic cell
desalination process: (DC) based immunotherapy in acquired
1. Low-temperature thermal desalination is a immunodeficiency syndrome (AIDS).
technique for desalination that works on the Which of the statements given above is/are Incorrect?
notion that water evaporates at lower a) 1 only
temperatures, at low pressures. b) 2 only
2. LTTD technology is found suitable for Lakshadweep c) Both 1 and 2
islands where the required temperature difference d) Neither 1 nor 2
of about 15⁰ C between sea surface water and
deep-sea water is found in the vicinity of Q.11) Consider the following statements about
Lakshadweep coasts only as of now. Kalanamak Rice:
3. The National Institute of Ocean Technology (NIOT), 1. Kalanamak is traditionally grown in usar soils
autonomous Institute has developed Low characterised by higher salt concentration and high Ph
in the NorthEast region.

DPP 2023 DAY 160 2


Contact us :info@onlyias.com

OnlyIAS Nothing Else Visit :dpp.onlyias.in


Contact : +91-7007 931 912

2. This non-basmati scented rice variety is highly states to install blending infrastructure for the
resistant to notorious common rice diseases such as production of fortified rice.
panicle blast, stem rot and brown spot. Which of the statements given above is/are Incorrect?
3. It has been rebranded as Buddha Rice in order to a) 1 and 2 only
promote the sale of this GI-tagged rice in Buddhist b) 2 and 3 only
countries. c) 1 and 3 only
Which of the statements given above is/are correct? d) All of the above
a) 3 only
b) 1 and 3 only Q.15) Consider the following statements regarding
c) 1 and 2 only PM Gati-Shakti National Master Plan:
d) 2 and 3 only 1. It incorporates the infrastructure schemes
Bharatmala road project and Sagarmala waterways
Q.12) Consider the following statements regarding R plan.
Value : 2. This plan will not cover pharmaceutical clusters and
1. The R number signifies the average number of defence corridors.
people that one infected person will pass the virus to. 3. It will leverage spatial planning tools with ISRO’s
2. An R number lower than 1 indicates that the disease satellite imagery.
will spread and the number of active cases will Which of the statements given above is/are correct?
increase. a) 3 only
Which of the statements given above is/are correct? b) 1 and 3 only
a) 1 only c) 1 and 2 only
b) 2 only d) All of the above
c) Both 1 and 2
d) Neither 1 nor 2 Q.16) Which of the following statement is Incorrect
about Flex Fuel Vehicle (FFV)?
Q.13) Consider the following statements regarding a) Flex fuel vehicles (FFV) are capable of running on
Agrivoltaic Farming : 100 percent petrol or 100 per cent bio-ethanol but
1. It can resolve land availability constraints for setting not the combination of both.
up solar farms and helps check soil erosion. b) It helps in addressing the problems of surplus food
2. Setting up of such integrated agri-photovoltaic farms grains.
is typically a labour-intensive proposition. c) It will provide an option for using biofuels and
Which of the statements given above is/are Incorrect? reduce dependence on oil imports.
a) 1 only d) It will provide direct benefits to farmers and help in
b) 2 only doubling the farmer’s income.
c) Both 1 and 2
d) Neither 1 nor 2 Q.17) Consider the following statements regarding
Q.14) Consider the following statements regarding LiDAR (light detection and ranging) Technology:
Food Fortification: 1. It is a remote sensing technology that uses light in
1. Fortification is necessary to address deficiency of the form of a pulsed laser to measure ranges
micronutrients or micronutrient malnutrition, also (distances) to a target.
known as “hidden hunger”, a serious health risk. 2. Topographic LiDAR uses water-penetrating green
2. The Food Corporation of India (FCI) sets standards light to also measure seafloor and riverbed elevations.
for food fortification in the country. Which of the statements given above is/are correct?
3. The Food Safety and Standards Authority of India a) 1 only
(FSSAI) has made it mandatory for the rice millers in all b) 2 only

DPP 2023 DAY 160 3


Download From - https://upscmaterial.online/

Contact us :info@onlyias.com

OnlyIAS Nothing Else Visit :dpp.onlyias.in


Contact : +91-7007 931 912

c) Both 1 and 2 Q.21) Consider the following statements regarding


d) Neither 1 nor 2 Priority Watch List:
1. Priority Watchlist is a list of countries having
Q.18) Consider the following statements regarding "serious intellectual property rights deficiencies".
Biotech-PRIDE: 2. The listed countries are identified by a Special 301
1. Biotech-PRIDE are guidelines that have been report published annually by the World Intellectual
introduced in the interest of data sharing in order Property Organisation.
to further research and innovation in India. Which of the statements given above is/ are correct?
2. The guidelines will be implemented by the Indian a) 1 only
Biological Data Centre (IBDC) under the b) 2 only
Department of Biotechnology. c) Both 1 and 2
Which of the statements given above is/are correct? d) Neither 1 nor 2
a) 1 only
b) 2 only Q.22) Consider the following statements regarding
c) Both 1 and 2 Traditional Knowledge Digital Library:
d) Neither 1 nor 2 1. The Traditional Knowledge Digital Library was set
Q.19) Consider the following statements regarding up by the World Intellectual Property Organisation
DNA technology regulation Bill: in 2001.
1. The DNA Technology Regulation Bill allows 2. The objective of Traditional Knowledge Digital
profiling of victims accused of crimes, and those Library is to protect the ancient and traditional
reported missing, as well as storing of their DNA knowledge of the country from exploitation

e
information solely in the National DNA Data Bank. through biopiracy and unethical patents.

n
2. The Bill provides for the establishment of a DNA 3. The limitation of the Traditional knowledge digital

li
Regulatory Board and the Secretary, Department library is that the content is available in English and
of Biotechnology, will be the ex officio Chairperson French language only.

n
of the Board. Which of the statements given above is/ are correct?
o
3. The Bill aims to minimise errors in criminal a) 1 and 3 only
l.
investigations and improve the justice delivery b) 2 only
a

system. c) 2 and 3 only


ri

Which of the statements given above is/ are correct? d) 1 and 2 only
a) 1 and 3 only
te

b) 3 only Q.23) Consider the following statements regarding


c) 2 and 3 only AlphaFold:
a

d) 1, 2 and 3 1. AlphaFold is an Artificial Intelligence-based


m

electromagnetic wave pattern prediction tool.


c

Q.20) Which of the following statements correctly 2. AlphaFold is developed by the World Health
describe the KAPILA campaign recently seen in the Organization for effective disease prediction.
s

news? 3. AlphaFold is fed with protein sequences as input


p

a) A campaign for information dissemination about where protein sequences enter through one end
.u

indigenous cow breeds. and the predicted three-dimensional structures


b) An Intellectual Property Literary and awareness come out through the other.
w

campaign. Which of the statements given above is/ are


w

c) A DNA forecasting technique for Indigenous milch incorrect?


animals. a) 2 only
w

d) None of the above b) 3 only

DPP 2023 DAY 160 4


https://upscmaterial.online/
Download From - https://upscmaterial.online/

Contact us :info@onlyias.com

OnlyIAS Nothing Else Visit :dpp.onlyias.in


Contact : +91-7007 931 912

c) 1 and 2 only 3. It aggravates the antibiotic resistance in bacteria.


d) 1 and 3 only Which of the statements given above are correct?
a) 1 and 2 only
Q.24) What is/are the significance of RNA b) 2 and 3 only
interference (RNAi) technology which gained c) 1 and 3 only
popularity in recent years? d) All of the above
1. Formulating drugs capable of reducing cholesterol
levels. Q.28) Consider the following statements regarding
2. Silencing the entire gene for protein synthesis. Genome Sequencing:
3. Gene mapping and annotation in plants. 1. It means deciphering the exact order of base pairs
4. Treating acute viral infections in an individual.
Which of the statements given above is/are correct? 2. It can help in identifying the genomic causes of rare
a) 1 and 3 only disease but cannot help in the understanding of
b) 2 and 4 only how virus spreads and evolves.
c) 1, 3 and 4 only 3. Genome India Project by Council for Scientific and
d) All of the above Industrial Research aims to build a reference
genome.
Q.25) Consider the following statements regarding 4. Nanopore gene sequencing works by monitoring
IPRs (Intellectual Property Rights): changes to an electrical current as nucleic acids are
1. Commercialization of IPRs is one of the objectives of passed through a protein nanopore.
National IPR Policy. Which of the statements given above are correct?

e
2. Indian Patent Act allows ever-greening of patents a) 1, 2 and 4 only

n
over a product by introducing minor changes to it. b) 1 and 4 only

li
3. In circumstances of “national emergency or in c) 2 and 3 only
circumstances of extreme urgency’’ state governments d) All of the above

n
can issue Compulsory Licensing which is permitted o
under the WTO’s TRIPS Agreement. Q.29) Consider the following statements regarding
l.
Which of the statements given above is/are Incorrect? India Innovation Graphene Centre (IIGC):
a

a) 2 and 3 only 1. India’s first graphene innovation center to be


ri

b) 2 only established in Karnataka.


c) 1 and 3 only 2. A laser-induced form of graphene masks was
te

d) All of the above produced by Indian researchers that inactivated


the coronavirus species.
a

Q.26) The Pilanesberg Resolution and the Manhattan 3. Graphene is the thinnest and most electrically and
m

Principles are associated with which of the following? thermally conductive material in the world.
c

a) Water Research & Technology Which of the statements given above is/are incorrect?
b) Nanoscience and nanotechnology a) 3 only
s

c) Renewable energy and carbon pricing b) 2 and 3 only


p

d) The concept of One health c) 1 and 2 only


.u

d) All of the above.


Q.27) Which of the following are the concerns related
w

to Genetically Modified (GM) Crops: Q.30) Consider the following statements regarding
w

1. It can develop resistance towards weeds that are Digital Health ID card:
difficult to eradicate.
w

2. It reduces the usage of herbicides.

DPP 2023 DAY 160 5


https://upscmaterial.online/
Download From - https://upscmaterial.online/

Contact us :info@onlyias.com

OnlyIAS Nothing Else Visit :dpp.onlyias.in


Contact : +91-7007 931 912

1. The health ID provides individual medical records b) 1 and 2 only


including doctor visits, diseases and drugs c) 1 and 3 only
consumed. d) 2 and 3 only
2. Aadhaar is mandatory for generating the health ID
card. Q.33) Which of the following statements correctly
3. The personal health records will be accessible explain Precision Medicine?
through an app or a website with ownership lying a) An Artificial Intelligence powered diseases
with the individual. diagnostic tool.
Which of the statements given above is/ are correct? b) A therapeutics tool focuses on mental health and
a) 3 only neurology.
b) 1 and 3 only c) Medicine that can be tailored according to the
c) 1 and 2 only patient's unique genetic make-up.
d) 1, 2 and 3 d) An advanced gene-editing technology.

Q.31) Which of the following are advantages of Q.34) Which of the statements correctly explains
Blockchain technology in the health sector? ‘Smart Pills’ recently seen in the news?
1. It identifies, tracks, and traces prescription a) An Artificial Intelligence based application that
medications and vaccine distribution. monitors and diagnoses the human body.
2. It enables a user-centred means of gathering b) The miniature electronic devices that perform
health information securely. extremely advanced functions such as sensing,
3. It provides robust safeguards for patient imaging, and drug delivery

e
Information and anonymizing their data. c) An Artificial Intelligence powered diseases

n
4. It can facilitate complete health product diagnostic tool.

li
traceability. d) A mobile controlled pill focusing on mental health
Choose the correct statements using the code given and neurology

n
below: o
a) 1 and 4 only Q.35) Consider the following statements regarding
l.
b) 2 and 3 only use of magnetocaloric material for cancer treatment:
a

c) 2, 3 and 4 only 1. The magnetocaloric materials are certain materials


ri

d) 1, 2, 3 and 4 in which application and removal of a magnetic


field causes the materials to become warmer or
te

cooler.
Q.32) Consider the following statements regarding 2. The drawback in Magnetic hyperthermia like the
a

Aarogya Setu App: lack of control of temperature can be avoided by


m

1. Aarogya setu digital health services are powered by using magnetocaloric materials, as it can provide
c

Ayushman Bharat Digital Mission (ABDM). controlled heating.


2. It allows users to receive digital lab reports, Which of the statements given above is/are correct?
s

prescriptions and diagnosis seamlessly from a) 1 only


p

verified healthcare professionals and health b) 2 only


.u

service providers. c) Both 1 and 2


3. Using Aarogyasetu you can download the d) Neither 1 nor 2
w

vaccination certificate but cannot request for


w

changes in certificate. Q.36) Consider the following statements regarding


Which of the statements given above is/are correct? Carbon capture and storage:
w

a) 1 only

DPP 2023 DAY 160 6


https://upscmaterial.online/
Download From - https://upscmaterial.online/

Contact us :info@onlyias.com

OnlyIAS Nothing Else Visit :dpp.onlyias.in


Contact : +91-7007 931 912

1. Carbon capture and storage is the process of Q.39) Consider the following statements regarding
capturing carbon dioxide (CO2) after it enters the AgriStack:
atmosphere. 1. AgriStack is a collection of technologies and digital
2. CO2 is captured from large point sources, such as a databases that focuses on farmers and the
chemical plant or biomass power plant, and then agricultural sector.
stored in an underground geological formation. 2. Private sector companies are also involved as far as
3. CO2 cannot be captured directly from an industrial building of the Agristack is concerned.
source such as a cement kiln. 3. Each farmer will receive a unique digital
Which of the statements given above is/are correct? identification (farmers' ID) which will be linked to
a) 1 and 3 only the individual's Aadhaar ID.
b) 3 only Which of the statements given above is/ are correct?
c) 1 and 2 only a) 1 and 2 only
d) 2 only b) 1 and 3 only
c) 3 only
Q.37) Consider the following statements regarding d) 2 and 3 only
Green Hydrogen:
1. Green Hydrogen is produced when natural gas is Q.40) Which of the following statements correctly
split into hydrogen and CO2. explain CODE19 recently seen in the news?
2. The process to make green hydrogen is powered by a) A guideline to save citizens from Covid19
renewable energy sources, such as wind or solar. b) A hackathon that aims to find solutions against the
3. The National Hydrogen Mission aims for COVID-19.

e
production of 5 million tonnes of Green hydrogen c) NCERT guidelines for digital education

n
by 2030. d) A secret communication code used in the Kargil

li
Which of the statements given above is/are correct? War.
a) 2 and 3 only

n
b) 1 and 3 only Q.41) The garbage is treated with bio-organisms or
o
c) 1 only natural elements like air and sunlight so that the
l.
d) 1, 2 and 3 biodegradable elements in the waste break down
a

over time. Which of the following term best describes


ri

Q.38) Which of the following are applications of the given definition?


Digital therapeutics and diagnostics? a) Greenwashing
te

1. Genetic engineering and molecular medicine. b) Pyrolysis


2. Seamless continuum of care and tracking health c) Bio-mining
a

parameters in real time. d) Biosparging


m

3. Predictive Analytics of diseases and more robust


c

doctor-patient interactions. Q.42) Consider the following statements regarding


4. Reduce cybersecurity risk related to health data. Polio:
s

Choose the correct statements using the code given 1. It is a highly infectious Bacterial disease, which
p

below: mainly affects young children and is transmitted by


.u

a) 1 and 4 only person-to-person spread mainly through the


b) 1, 2 and 3 faecal-oral route.
w

c) 1, 3 and 4 2. For a country to be declared polio-free, the wild


w

d) 1, 2, 3 and 4 transmission of all three kinds of Polio has to be


stopped.
w

DPP 2023 DAY 160 7


https://upscmaterial.online/
Download From - https://upscmaterial.online/

Contact us :info@onlyias.com

OnlyIAS Nothing Else Visit :dpp.onlyias.in


Contact : +91-7007 931 912

3. Injectible Polio Vaccine is made up of live- 1. Artemis I Mission. - A testing a method of


attenuated virus, while oral Polio Vaccine is made planetary defense against near-Earth objects
up of heat-killed virus that cannot cause the 2. CAPSTONE. - Cubesat to test a unique,
disease in any case, because the pathogen is not elliptical lunar orbit.
alive. 3. Lucy mission - First space mission to study
Which of the statements given above is /are correct? the Trojans
a) 1 and 2 only 4. DART mission - Next generation of lunar
b) 2 and 3 only exploration
c) 2 only Which of the pairs given above is/ are correctly
d) 1, 2 and 3 matched ?
a) 1 and 3 only
Q.43) Consider the following statements regarding b) 2 and 3 only
gene editing and related technologies: c) 2 and 4 only
1. Gene editing technology changes an organism’s d) 1, 2 , 3 and 4
DNA by allowing genetic material to be added or
removed, but it cannot alter the DNA at particular Q.46) India is signatory to which of the following
locations in the genome. Voluntary Pledges/Declarations/Agendas launched
2. CRISPR/Cas9-based disease therapeutics have during the COP26 (Glasgow) talks?
been achieved only ex-vivo delivery and not in-vivo 1. Breakthrough Agenda
delivery 2. Global Methane Pledge
Which of the statements given above is/ are not 3. Declaration on “accelerating the transition to 100%

e
correct? zero-emission cars and vans”

n
a) 1 only 4. Glasgow Leaders’ Declaration on Forests and Land

li
b) 2 only Use
c) Both 1 and 2 Which of the options given above is/are correct?

n
d) Neither 1 nor 2 a) 1 and 3 only o
b) 2 and 4 only
l.
Q.44) Consider the following statements regarding c) 1, 2 and 4 only
a

hydrogen: d) All of the above


ri

1. Hydrogen is the lightest and most abundant


element in the universe and is abundantly found in Q.47) Consider the following statements regarding
te

earth’s atmosphere in its elemental form. the Loss and Damage of Paris Agreement :
2. Green Hydrogen Policy was notified by the Ministry 1. Loss is the negative impacts in relation to which
a

of Power to regulate and facilitate the transition reparation or restoration is possible while Damage is
m

from fossil fuels to green hydrogen the negative impacts in relation to which reparation or
c

Which of the statements given above is /are correct? restoration is impossible.


a) 1 only 2. The Paris agreement proposed provisions of finance
s

b) 2 only for loss and damage for the first time.


p

c) Both 1 and 2 3. The Santiago Network is a network to enable loss


.u

d) Neither 1 nor 2 and damage fund flow from developed to developing


countries.
w

Which of the statements given above is/are Incorrect?


w

Q.45) Consider the following pairs of space missions a) 1 and 2 only


and their objectives : b) 2 and 3 only
w

Space missions. Objectives c) 3 only

DPP 2023 DAY 160 8


https://upscmaterial.online/
Download From - https://upscmaterial.online/

Contact us :info@onlyias.com

OnlyIAS Nothing Else Visit :dpp.onlyias.in


Contact : +91-7007 931 912

d) None of the above 2. India Ranked 10 in the Climate Change Performance


Index 2022 Report.
Q.48) Consider the following statements regarding Which of the statements given above is/are Incorrect?
Global Resilience Index Initiative (GRII) : a) 1 only
1. It has been launched to build a universal model for b) 2 only
assessing resilience to climate risks at Paris (CoP 21). c) Both 1 and 2
2. It is initiated by Coalition for Disaster Resilient d) Neither 1 nor 2
Infrastructure (CDRI) to provide a globally consistent
model for the assessment of resilience across all Q.52) What are the potential reasons for the rise in
sectors and geographies. pollution in the National Capital Region of New Delhi
Which of the statements given above is/are correct? during winter?
a) 1 only 1. Normal vertical distribution of temperature
b) 2 only 2. Lighter wind and Geographical location
c) Both 1 and 2 3. Pre-Harvest season and stubble burning
d) Neither 1 nor 2 4. Bursting of Firecrackers and Burning of waste for
warmth
Q.49) Recently established Net-Zero Banking Alliance Which of the statements given above is/are correct?
(NZBA) is convened by? a) 1 and 2 only
a) World Economic Forum (WEF) b) 3 and 4 only
b) World Bank through Green Climate Fund (GCF) c) 1, 3 and 4 only
c) European Union and United states of America d) All of the above

e
d) United Nations Environment Programme (UNEP)

n
Q.53) Consider the following statements regarding

li
Q.50) Consider the following statements regarding Clydebank Declaration:
the Commission for Air Quality Management (CAQM) 1. It aimed at establishing green shipping corridors like

n
: the zero-emission maritime routes between 2 (or
o
1. It has given the central pollution control board more) ports.
l.
powers to operationalise measures under the Graded 2. The signatory countries agreed to support the
a

Response Action Plan (GRAP) on air pollution. establishment of at least 6 green corridors by 2025.
ri

2. It will supersede bodies such as the central and state 3. India is a signatory of the declaration.
pollution control boards of Delhi, Punjab, Haryana, UP Which of the statements given above is/are correct?
te

and Rajasthan. a) 1 only


3. It is a permanent and non-statutory body. b) 2 and 3 only
a

Which of the statements given above is/are correct? c) 1 and 2 only


m

a) 1 and 2 only d) All of the above


c

b) 2 and 3 only
c) 1 and 3 only Q.54) It is a Project Tiger reserve in the Pakke Kessang
s

d) All of the above district in northeastern India. The tract of the


p

protected area falls within the land area of Pakke river


.u

Q.51) Consider the following statements regarding and Kameng river. This Tiger Reserve has won India
Climate Change Performance Index (CCPI): Biodiversity Award 2016 in the category of
w

1. CCPI was developed by the World Economic forum ‘Conservation of threatened species’ for its Hornbill
w

(WEF) as a tool for increasing transparency in Nest Adoption Programme.It is known for its amazing
international climate change policy. sightings of four resident hornbill species.
w

DPP 2023 DAY 160 9


https://upscmaterial.online/
Download From - https://upscmaterial.online/

Contact us :info@onlyias.com

OnlyIAS Nothing Else Visit :dpp.onlyias.in


Contact : +91-7007 931 912

Which of the following reserve is best suited for the c) A web portal to enhance access to legal
statement? information and empower the citizens about the
a) Nameri Tiger Reserve legal powers
b) Pakhui Tiger Reserve d) A web portal to help farmers and consumers to
c) Namdapha Tiger Reserve perform online purchases
d) Kamlang Tiger Reserve
Q.58) What are the main drivers of the majority of the
Q.55) Consider the following statements regarding flash drought events in India ?
National Interlinking of Rivers Authority (NIRA): 1. Amplification of evaporative demand via increased
1. NIRA is an independent autonomous body that will solar insolation and global warming
work along with the National Water Development 2. Cyclonic conditions
Agency (NWDA). 3. High precipitation variability in Tropics and Sub-
2. It will coordinate with neighbouring countries and Tropics
concerned states and departments. 4. Oscillation of the Inter-Tropical Convergence Zone
3. It has powers on issues related to environment, (ITCZ)
wildlife and forest clearances under river linking Which of the statements given above is/are correct?
projects and their legal aspects. a) 1 and 2 only
Which of the statements given above is/are correct? b) 3 and 4 only
a) 1 and 2 only c) 1, 3 and 4 only
b) 2 and 3 only d) All of the above
c) 1 and 3 only

e
d) All of the above Q.59) Consider the following statements regarding

n
National Coal Gasification Mission:

li
Q.56) Consider the following statements regarding 1. Coal gasification is the process of converting coal
the River Cities Alliance (RCA): into synthesis gas which is a mixture of methane (CH4)

n
1. The National Mission for Clean Ganga (NMCG) and and sulphur dioxide (SO2). o
National Institute for Urban Affairs (NIUA) have 2. The mission includes mapping of gasification
l.
collaborated together to launch the River Cities potential of coalfields especially in south eastern India.
a

Alliance (RCA). 3. The Ministry of Power aims to achieve gasification of


ri

2. RCA includes cities exclusively from non-Ganga basin 1000 Million Tonnes (MT) of coal by 2050.
states to ideate, discuss and exchange information for Which of the statements given above is/are Incorrect?
te

sustainable management of urban rivers. a) 1 only


Which of the statements given above is/are correct? b) 2 only
a

a) 1 only c) 1 and 3 only


m

b) 2 only d) All of the above


c

c) Both 1 and 2
d) Neither 1 nor 2 Q.60) What are the potential results of Interlinking of
s

Rivers (ILR)?
p

Q.57) Recently launched ‘E-AMRIT PORTAL’ is 1. Boost inland waterway transportation


.u

associated with? 2. Tackling problems of floods and droughts


a) A web portal to disseminate all educational e- 3. Achieving food security and reducing volatility in the
w

resources food-grain prices


w

b) A web portal on electric vehicles (EVs) 4. Reduce disease burden and increase distress
migration
w

Which of the statements given above is/are correct?

DPP 2023 DAY 160 10


https://upscmaterial.online/
Download From - https://upscmaterial.online/

Contact us :info@onlyias.com

OnlyIAS Nothing Else Visit :dpp.onlyias.in


Contact : +91-7007 931 912

a) 1, 2 and 3 only 2. The Climate Equity Monitor is aimed at monitoring


b) 1 and 4 only the performance of all the member countries of UN
c) 2 and 3 only based on the principles of equity and common but
d) All of the above differentiated responsibilities and respective
capabilities (CBDR-RC)
Q.61) Consider the following statements regarding Which of the statements given above is/ are correct?
Cheetah Reintroduction project: a) 1 only
1. Cheetah has been protected under Appendix I of b) 2 only
the Convention on International Trade in c) Both 1 and 2
Endangered Species of Wild Fauna and Flora d) Neither 1 nor 2
(CITES)
2. African cheetahs are slightly smaller than their Q.64) India proposed a five-fold strategy for it to play
Asian counterparts and are found only in Namibia its part in helping the world get closer to 1.5 degrees
3. Kuno National Park, given the status of national Celsius on the first day of the global climate meet in
park named after Kuno River, one of the major 26th Conference of Parties (CoP26) to the United
tributaries of the Narmada River Nations Framework Convention on Climate Change,
Which of the statements given above is/ are not Glasgow. Which of the following are included in the
correct? fivefold strategy?
a) 1 only 1. India will get its non-fossil energy capacity to 500
b) 2 and 3 only gigawatts by 2030
c) 1 and 3 only 2. India will achieve net zero by 2050

e
d) 1, 2 and 3 3. India will meet 50 per cent of its energy

n
requirements till 2030 with renewable energy

li
Q.62) Consider the following statements regarding Select the correct answer using the code given below:
Indus River dolphins: a) 1 only

n
1. Like other freshwater dolphins, the Indus River b) 2 and 3 only o
dolphin is an important indicator of the health of a c) 1 and 3 only
l.
river and it was declared the State aquatic animal d) 1, 2 and 3
a

of Jammu and Kashmir


ri

2. They can only be found in the lower parts of the Q.65) Consider the following statements regarding
Indus River in Pakistan and in River Beas, a Global Methane Pledge:
te

tributary of the Indus River in India. 1. This pledge was announced with the aim of
Which of the statements given above is /are correct? reducing methane emissions by 30% by the year
a

a) 1 only 2030, as compared to 2020 levels.


m

b) 2 only 2. More than 103 countries have signed up so far,


c

c) Both 1 and 2 including major methane emitters like China,


d) Neither 1 nor 2 Russia and US except India.
s

3. Approximately 60 % of methane emission comes


p

Q.63) Consider the following statements regarding from natural sources which is higher than the
.u

Climate Equity Monitor : anthropogenic sources that amounts to 40% of


1. The Climate Equity Monitor, an online dashboard emission
w

for assessing equity in climate action at the Which of the statements given above is /are correct?
w

international level has been launched by Ministry a) 1 and 2 only


of Statistics and programme implementation b) 2 only
w

c) 1 only

DPP 2023 DAY 160 11


https://upscmaterial.online/
Download From - https://upscmaterial.online/

Contact us :info@onlyias.com

OnlyIAS Nothing Else Visit :dpp.onlyias.in


Contact : +91-7007 931 912

d) 1, 2 and 3 Q.69) Recently a new species was discovered from the


tribal regions of Edamalakkudy in Idukki. The
Q.66) Recently, a glacier has been formally named scientific name of the species was decided as
Glasgow Glacier in honour of Glasgow, Scotland, “Cryptocarya Muthuvariana” , recognising the
which is hosting the high-level UN climate conference. contribution of Muthuvar tribes in protecting the
This glacier is a 100-kilometre-long body of ice which forests and the community of tribals that are residing
has been rapidly melting due to global warming. The in the region. “Cryptocarya Muthuvariana” belongs to
Glasgow glacier is located in which of the following which of the following biological species ?
places? a) Butterfly
a) Canada b) Snake
b) Iceland c) Plants
c) Antarctica d) Fungi
d) Karakoram Himalayas
Q.70) Consider the following statements regarding
Q.67) The United Nations Environment Programme Flash droughts occurrence in India :
(UNEP) is the leading environmental authority in the 1. A majority of the flash drought events in India
United Nations system. Which of the following occurred during the post monsoon season
reports is/ are released by the United Nations especially across the central, northwest and
Environment Programme ? northeast regions of the country
1. Emissions Gap Report 2. Unlike conventional droughts that develop over
2. Sustainable Urban Cooling Handbook months, flash droughts intensify very quickly.

e
3. Annual Frontiers report Which of the statements given above is/ are correct?

n
4. Food Waste Index Report a) 1 only

li
Select the correct answer using the code given below: b) 2 only
a) 1 and 3 only c) Both 1 and 2

n
b) 2, 3 and 4 only d) Neither 1 nor 2 o
c) 1, 2 and 4 only Q.71) Consider the following statements regarding
l.
d) 1, 2 ,3 and 4 Medical Termination of Pregnancy (MTP) Amendment
a

Act, 2021:
ri

Q.68) Consider the following statements regarding 1. Opinion of the State-level medical board is
Extinction of species : essential for a pregnancy to be terminated up to 20
te

1. Background extinction rate refers to the standard weeks in case of substantial foetal abnormalities.
rate of extinction in Earth's geological and 2. The act do not allow unmarried women to
a

biological history after humans became a primary terminate a pregnancy after 16 weeks in the case
m

contributor to extinctions. of failure of contraceptive method or device.


c

2. Currently our planet is experiencing a seventh 3. The “name and other particulars of a woman
mass extinction as the result of human-induced whose pregnancy has been terminated shall not be
s

climate change referred to as the Holocene or revealed”, except to a person authorised in any law
p

Anthropocene extinction that is currently in force.


.u

Which of the statements given above is/ are correct? Which of the statements given above is/ are correct?
a) 1 only a) 1 only
w

b) 2 only b) 2 and 3 only


w

c) Both 1 and 2 c) 3 only


d) Neither 1 nor 2 d) 1, 2 and 3
w

DPP 2023 DAY 160 12


https://upscmaterial.online/
Download From - https://upscmaterial.online/

Contact us :info@onlyias.com

OnlyIAS Nothing Else Visit :dpp.onlyias.in


Contact : +91-7007 931 912

Q.72) Consider the following statements regarding benefit and maternity benefit in case of women
PM Adi Adarsh Gram Yojna (PMAAGY) : employees
1. It is envisaged to cover villages having at least 50% Which of the statements given above is/ are correct?
Scheduled Castes population and 500 Scheduled a) 1 and 2 only
Castes across States with notified Scheduled Castes b) 2 only
2. The scheme will be implemented during 2021-22 c) 3 only
to 2029-30 and it aims to provide basic d) 1, 2 and 3
infrastructure in specified villages
3. PM Adi Adarsh Gram Yojna envisions to mitigate Q.75) Consider the following statements regarding
gaps in prominent 8 sectors of development Criminal Procedure (Identification) Act, 2022 :
including Solid waste management and Drainage 1. The act prohibits the police from taking physical
Which of the statements given above is /are correct? and biological samples of convicts as well as those
a) 3 only accused of crimes.
b) 2 and 3 only 2. National Crime Records Bureau (NCRB) will store,
c) 1 and 3 only preserve, share with any law enforcement agency
d) 1, 2 and 3 and destroy the record of measurements at
national level.
Q.73) Consider the following statements regarding 3. CrPC is the principal criminal code of India that
Har Ghar Jal scheme : defines crimes and provides punishments for
1. It is a flagship programme of the Ministry of Rural almost all kinds of criminal and actionable wrongs.
development in partnership with States/ UTs to Which of the statements given above is /are correct?

e
ensure tap water connection in every rural a) 3 only

n
household by 2024. b) 2 and 3 only

li
2. The mission empowers Gram Panchayats /rural c) 2 only
communities to plan, implement, manage, own, d) 1, 2 and 3

n
operate, and maintain their own in-village water o
supply systems. Q.76) Consider the following statements regarding
l.
3. No Union territory has become the Har ghar Jal Fast track courts:
a

certified state yet. 1. Fast Track Courts (FTCs) are set up by the Central
ri

Which of the statements given above is/ are correct? government Government in consultation with the
a) 2 and 3 only concerned High Courts to ensure swift
te

b) 2 only dispensation of justice.


c) 1 and 3 only 2. Article 247 gives power to Parliament to establish
a

d) 1 and 2 only certain additional courts for the better


m

administration of laws made by it with respect to a


c

Q.74) Consider the following statements regarding matter enumerated in the Union List.
the Employees' State Insurance Scheme: Which of the statements given above is /are not
s

1. Employees' State Insurance Scheme was launched correct?


p

by Ministry of Finance to provide social security to a) 1 only


.u

the industrial workers in certain contingencies b) 2 only


2. Employees’ State Insurance Corporation (ESIC), a c) Both 1 and 2
w

non-statutory corporate body is responsible for the d) Neither 1 nor 2


w

administration of ESI Scheme.


3. The employees registered under the scheme are Q.77) Consider the following statements regarding
w

entitled to medical treatment, unemployment the office of Vice President in India :

DPP 2023 DAY 160 13


https://upscmaterial.online/
Download From - https://upscmaterial.online/

Contact us :info@onlyias.com

OnlyIAS Nothing Else Visit :dpp.onlyias.in


Contact : +91-7007 931 912

1. Electoral College of Vice President consists of both Q.80) Consider the following statements:
elected and nominated members of the Parliament 1. Traditional Knowledge Digital Library is a database
and does not include the members of the state of Indian traditional knowledge established by the
legislative assemblies Council of Scientific and Industrial Research (CSIR)
2. Article 61 mentions Violation of constitution as the and the Ministry of Environment, Forest and
ground for removal of Vice President from his Climate Change.
office 2. Biological Diversity Act, 2002 provides for
Which of the statements given above is /are not preservation, maintenance of Traditional
correct ? knowledge along with equitable sharing of benefits
a) 1 only arising from use of the knowledge with
b) 2 only communities.
c) Both 1 and 2 3. India is a signatory to Convention on Biological
d) Neither 1 nor 2 Diversity and Cartagena Protocol, which are
concerned with trade-in bioresources and use of
Q.78) Consider the following statements regarding Traditional Knowledge.
Tribunals in India : Which of the statements given above is /are correct?
1. While Article 323 A contemplates the a) 1 and 3 only
establishment of tribunals for public service b) 2 only
matters only, Article 323 B contemplates the c) 2 and 3 only
establishment of tribunals for certain other d) 1, 2 and 3
matters like taxation and land reforms.

e
2. Armed Forces Tribunal is a military tribunal Q.81) Consider the following statements regarding

n
established in 2009 by the Army Act 1950 has both Elephant population in India :

li
Original Jurisdiction and appellate jurisdiction 1. Project Elephant was launched by the Government
Which of the statements given above is /are correct? of India in the year 1992 as a Centrally Sponsored

n
a) 1 only Scheme to protect elephants, their habitat and
o
b) 2 only corridors.
l.
c) Both 1 and 2 2. The Project Elephant is implemented in all states
a

d) Neither 1 nor 2 and Union territories of India


ri

3. Recently central government has notified


Q.79) Consider the following statements regarding Agasthyamalai Elephant Reserve, in the state of
te

thylacine: Kerala
1. The thylacine, the only animal in the Thylacinid Which of the statements given above is/ are correct ?
a

family to survive in modern times, was a marsupial a) 1 only


m

mammal and was native to the African mainland b) 2 and 3 only


c

and the islands of Africa c) 2 only


2. According to the International Union for d) 1, 2 and 3
s

Conservation of Nature (IUCN), the thylacine, also


p

known as the Tasmanian tiger, was extinct in 1982. Q.82) Consider the following statements regarding
.u

Which of the statements given above is/ are not India’s endeavour in Antarctica:
correct? 1. India is one of the 29 Consultative Parties to the
w

a) 1 only Antarctic Treaty which was signed in 1983


w

b) 2 only 2. India is not a party to the Madrid protocol also


c) Both 1 and 2 called as Protocol on Environmental Protection to
w

d) Neither 1 nor 2

DPP 2023 DAY 160 14


https://upscmaterial.online/
Download From - https://upscmaterial.online/

Contact us :info@onlyias.com

OnlyIAS Nothing Else Visit :dpp.onlyias.in


Contact : +91-7007 931 912

the Antarctic Treaty which provides for b) 2 only


comprehensive protection of Antarctica. c) 3 only
3. India has two active research stations Maitri at d) 2 and 3 only
Schirmacher Hills, and Himadri at Larsemann Hills
in Antarctica. Q.85) Consider the following statements regarding
Which of the statements given above is /are not Centre for Wildlife Rehabilitation and Conservation
correct? (CWRC) :
a) 1, 2 and 3 1. CWRC is the only facility in India where orphaned
b) 2 only and injured wild animals of several species are
c) 2 and 3 only hand-raised or treated and are subsequently
d) 1 and 3 only returned to the wild.
2. It is Strategically located adjacent to the Panbari
Q.83) Consider the following statements regarding Reserve Forest near Nilgiri national park in
Convention on International Trade in Endangered Tamilnadu , the centre attends to a wide range of
Species of Wild Fauna and Flora (CITES) : wildlife emergencies
1. CITES , drafted as a result of a resolution adopted Which of the statements given above is/ are correct?
in 1963 at a meeting of members of the a) 1 only
International Union for Conservation of Nature, b) 2 only
entered into force in 1975. c) Both 1 and 2
2. The CITES Secretariat is administered by d) Neither 1 nor 2
International Union for Conservation of Nature and

e
is located at Gland, Switzerland. Q.86) This lake was known as the “pearl of the south"

n
3. Species may be added to or removed from . It is an endorheic lake in the Muthanna area, Iraq

li
Appendix I and II, or moved between them,at any near to the Euphrates River. This lake has no inlet or
time and by any Party unilaterally. outlet, but it draws water from the Euphrates through

n
Which of the statements given above is /are not a system of joint cracks and fissures which transport
o
correct? water to aquifers beneath it. The Lake was declared a
l.
a) 1 only Ramsar site in 2014. Which of the following lakes is
a

b) 2 and 3 only referred to in the above lines ?


ri

c) 1 and 2 only a) Tanganyika


d) 3 only b) Baikal
te

c) Victoria
Q.84) Consider the following statements regarding d) Sawa
a

Bharat Stage Emission Standards (BSES) :


m

1. The National Highways Authority of India sets the Q.87) Consider the following statements regarding
c

permissible pollution levels and timeline to Comprehensive Economic Cooperation and


implement an equivalent by vehicle makers. Partnership Agreement (CECPA) :
s

2. While the BS4 emission norms allow a motor 1. The India-Mauritius CECPA is the first trade
p

vehicle to emit not more than 60mg /km of NO2 agreement, a kind of free trade pact signed by India
.u

(nitrogen oxides), the BS6 emission norms have with a country in Africa.
increased it to 80mg/km 2. Mauritius was the largest source of Foreign Direct
w

3. The BS6 emission norm is not only for motor Investment (FDI) into India in the financial year FY
w

vehicles but also for the fuel being dispensed. 2021-22.


Which of the statements given above is /are correct? Which of the statements given above is /are correct ?
w

a) 1 and 2 only a) 1 only

DPP 2023 DAY 160 15


https://upscmaterial.online/
Download From - https://upscmaterial.online/

Contact us :info@onlyias.com

OnlyIAS Nothing Else Visit :dpp.onlyias.in


Contact : +91-7007 931 912

b) 2 only 1. India never adhered to the One China Policy and it


c) Both 1 and 2 have established diplomatic relations with Taiwan
d) Neither 1 nor 2 as a independent country since 1949
2. India has also signed a Double-Taxation Avoidance
Q.88) Consider the following statements regarding Agreement and a Bilateral Investment Treaty with
India- Egypt bilateral relations : Taiwan.
1. Egypt released a commemorative postage stamp Which of the statements given above is /are not
to celebrate 25 years of diplomatic relations correct?
between Egypt and India a) 1 only
2. Egypt was one of the founding members of the b) 2 only
Non-Aligned Movement (NAM) along with India , c) Both 1 and 2
Indonesia ,Yugoslavia and Ghana d) Neither 1 nor 2
3. India and Egypt organize a bilateral military
exercise named Exercise Eastern Bridge to enhance Q.91) Consider the following statements regarding
mutual understanding between the Indian Air Treaty on the Non-Proliferation of Nuclear Weapons:
Force and the Egyptian Air Force. 1. The Non-Proliferation Treaty or NPT is an
Which of the statements given above is /are correct ? international treaty whose objective is to prohibit
a) 2 only the use of nuclear energy, and to further the goal
b) 3 only of achieving nuclear disarmament
c) 2 and 3 only 2. India, Israel, Pakistan and South Sudan have never
d) 1 and 3 only accepted the NPT, three of which possess or are

e
thought to possess nuclear weapons

n
Q.89) Consider the following statements regarding 3. NPT was opened for signature in 1968, but the

li
The International Transport Forum (ITF) : treaty has not entered into force yet.
1. International Transport Forum is an inter- Which of the statements given above is /are correct?

n
governmental organisation within the United a) 3 only o
Nations system and is the only global body with a b) 2 only
l.
mandate for all modes of transport. c) 1 and 3 only
a

2. The organisation brings together all UN member d) 1, 2 and 3


ri

countries with the aim to advance the global


transport policy agenda Q.92) Consider the following statements regarding
te

3. The ITF maintains the International Road Traffic Essential Commodities act 1955 :
and Accident Database (IRTAD), a comprehensive 1. The Essential Commodities act was enacted to stop
a

database of statistics related to road safety hoarding and black marketing of food and gives
m

Which of the statements given above is/ are not consumers protection against irrational spikes in
c

correct? prices of essential commodities.


a) 3 only 2. There is a specific definition of essential
s

b) 1 only commodities in the Essential Commodities Act,


p

c) 1 and 2 only 1955.


.u

d) 1, 2 and 3 3. The Act gives powers to the state government to


add or remove a commodity in the Schedule.
w

Q.90) Consider the following statements regarding Which of the statements given above is / are correct?
w

India Taiwan bilateral relationship: a) 3 only


b) 1 only
w

c) 2 and 3 only

DPP 2023 DAY 160 16


https://upscmaterial.online/
Download From - https://upscmaterial.online/

Contact us :info@onlyias.com

OnlyIAS Nothing Else Visit :dpp.onlyias.in


Contact : +91-7007 931 912

d) 1 and 3 only to various financial services to the weaker sections


& low income groups
Q.93) Consider the following statements regarding 2. Under the scheme, a basic savings bank deposit
Mine and Minerals Development and Regulation (BSBD) account can be opened in any bank branch
(MMDR) Act 2015: or Business Correspondent outlet, by persons not
1. The Act provides for a non-exclusive having any other account.
reconnaissance permit for all minerals including 3. A PMJDY account is treated as inoperative if there
coal, lignite, and atomic minerals are no customer induced transactions in the
2. The central government has the ownership over all account for over a period of five years.
offshore minerals extracted from the sea or ocean Which of the statements given above is/ are correct?
floor in the Indian maritime zones. a) 1 only
3. Captive mines are mines where the produced coal b) 2 only
or minerals can be used for their own use as well c) 2 and 3 only
as for sale. d) 1, 2 and 3
Which of the statements given above is/ are correct?
a) 2 only Q.96) Consider the following statements regarding
b) 2 and 3 only National Pharmaceutical Pricing Authority:
c) 1 and 3 only 1. The National Pharmaceutical Pricing Authority
d) 1, 2 and 3 (NPPA) is a statutory body and a regulatory agency
that controls the prices of pharmaceutical drugs in
Q.94) Consider the following statements regarding India

e
Mahatma Gandhi National Rural Employment 2. NPPA was constituted as an attached office of the

n
Guarantee Act 2005 or MGNREGA: Department of Pharmaceuticals under, Ministry of

li
1. It is a centrally sector scheme for providing Chemicals and Fertilizers
livelihood security and the creation of durable Which of the statements given above is/ are not

n
assets in rural areas which is being Implemented by correct? o
Ministry of Labour & Employment a) 1 only
l.
2. At least half of the of beneficiaries of this scheme b) 2 only
a

have to be women as mandated by this act c) Both 1 and 2


ri

3. Wages must be paid according to the statutory d) Neither 1 and 2


minimum wages specified for agricultural
te

labourers in the state under the Minimum Wages Q.97) Consider the following statements regarding
Act, 1948. Grameen Udyami Project:
a

Which of the statements given above is /are not 1. Grameen Udyami is a project that aims to promote
m

correct? skill development and entrepreneurship among


c

a) 1 and 3 only students belonging to minority religion from


b) 2 only backward regions
s

c) 1 and 2 only 2. It’s a unique multiskilling project funded by


p

d) 2 and 3 only National Skill Development Corporation that is


.u

being implemented in all states and Union


Q.95) Consider the following statements regarding territories of India
w

Pradhan Mantri Jan Dhan Yojana : 3. National Skill Development Corporation (NSDC) is a
w

1. PMJDY is a Central Sector Scheme launched by non-profit public limited company established
Ministry of Rural development for Ensuring access under Section 25 of the Companies Act, 1956.
w

Which of the statements given above is /are correct?

DPP 2023 DAY 160 17


https://upscmaterial.online/
Download From - https://upscmaterial.online/

Contact us :info@onlyias.com

OnlyIAS Nothing Else Visit :dpp.onlyias.in


Contact : +91-7007 931 912

a) 1 and 2 only a) 1 only


b) 3 only b) 3 only
c) 2 only c) 1 and 2 only
d) 2 and 3 only d) 1, 2 and 3

Q.98) Consider the following statements regarding


public goods:
1. The public goods involve rivalry or competition
among consumers for its usage since consumption
by one person will restrict its use by another.
2. Public goods involve cost, and therefore the non-
payers are excluded from the consumption.
Which of the statements given above is/are correct?
a) 1 only
b) 2 only
c) Both 1 and 2
d) Neither 1 nor 2

Q.99) Which one of the following best describes the


term “Moonlighting “ with reference to the Indian
economy ?

e
a) It is an technological innovation that significantly

n
alters the way that consumers, industries, or

li
businesses operate.
b) The situation in which the inflation rate is high or

n
increasing, the economic growth rate slows, and o
unemployment remains steadily high.
l.
c) The act of employees working at an extra job
a

beyond regular working hours, usually without the


ri

knowledge of the employer.


d) The losses incurred by the creditors (banks in this
te

case) on resolving the bad debts or stressed assets.


a

Q.100) Consider the following statements regarding


m

tax to GDP ratio :


c

1. The tax to GDP ratio is the ratio of tax collected


compared to national gross domestic product
s

(GDP)
p

2. Higher the tax to GDP ratio, the better the


.u

country's financial position.


3. India’s Gross tax to GDP improved to 40 percent in
w

FY21 that is much higher than the emerging market


w

economy average of 21 percent and OECD average


of 34 percent.
w

Which of the statements given above is/ are correct?

DPP 2023 DAY 160 18


https://upscmaterial.online/
Download From - https://upscmaterial.online/

Contact us :info@onlyias.com

OnlyIAS Nothing Else Visit :dpp.onlyias.in


Contact : +91-7007 931 912

Q.1) Ans: D ● In India, the Genetic Engineering Appraisal


Exp: Committee (GEAC) is the apex body that allows
● Statement 1 is not correct : Science and for the commercial release of GM crops. In
Engineering Research Board (SERB):SERB is a 2002, the GEAC had allowed the commercial
statutory body, working under Department of release of Bt cotton.It functions under the
Science and Technology.It was set up in 2009, Ministry of Environment, Forest and Climate
through SERB ACT, 2008. It was set up to Change (MoEF&CC).It evaluates research into
promote basic research in science and GM plants and recommends, or disapproves,
engineering as well as to give financial their release into farmer fields.
assistance to scientists, academic institutions, ● GM crops are that type of plants whose DNA
industrial concerns and R&D laboratories. has been modified through genetic
● Statement 2 is not correct : The Board is engineering for embedding a new trait to the
chaired by the Secretary to the Government plant which does not occur naturally in the
of India in the Department of Science and species. Genetic engineering aims to transcend
Technology and shall have other senior the genus barrier by introducing an alien gene
government officials and eminent scientists as in the seeds to get the desired effects and the
members. alien gene could be from a plant, an animal or
● Statement 3 is not correct : SERB-SURE even a soil bacterium. Across the world, GM
scheme provides research support to active variants of maize, canola and soybean etc , are
researchers belonging to state universities available.
and colleges including private universities and ● India has approved commercial cultivation of

e
colleges across India to undertake research only one GM crop, i.e Bt cotton. India

n
and development in frontier areas of science, approved the bacillus thuringiensis (Bt) variety

li
engineering and quantitative social science. of GM cotton in 2002 mainly since many
The scheme will seed new research growers began cultivating it illegally at the start

n
opportunities at various state & private of 2000. No GM food crop has ever been
o
universities and colleges and will propagate approved for commercial cultivation in the
l.
scientific temper in faculty and students of country.
a

these institutions.SERB-SURE scheme provides ● On average, GM technology adoption has


ri

research support to active researchers reduced chemical pesticide use by 37%,


belonging to state and private universities and increased crop yields by 22%, and increased
te

colleges, including self-financed institutions farmer profits by 68%. Yield gains and
working within these universities, across India, pesticide reductions are larger for insect-
a

to undertake research and development in resistant crops than for herbicide-tolerant


m

frontier areas of science, engineering, and crops.


c

quantitative social science.


s

Reference:
Reference :
p

https://www.thehindubusinessline.com/economy/a
https://www.indiascienceandtechnology.gov.in/anno
.u

gri-business/indian-government-indecisive-in-
uncementsopportunity/serb-invites-call-proposals-
approving-gm-crops-says-usda/article37996037.ece
state-university-research-excellence-serb-sure
w

https://theprint.in/india/how-modi-govt-has-
scuttled-gm-farming-in-india-by-moving-bt-brinjal-
w

trials-to-states/628682/
Q.2) Ans: D
w

Exp:

DPP 2023 DAY 160 19


https://upscmaterial.online/
Download From - https://upscmaterial.online/

Contact us :info@onlyias.com

OnlyIAS Nothing Else Visit :dpp.onlyias.in


Contact : +91-7007 931 912

Q.3) Ans: C supplemental oxygen can cause severe damage


Exp: to the lungs and other organ systems.
● Statement 1 is correct: The most common Liquid
Medical Oxygen production method is Reference:
separation of oxygen in what are known as Air https://pib.gov.in/PressReleasePage.aspx?PRID=171
Separation Units or ASUs. ASUs are basically 6197
plants that separate large volumes of gases. https://indianexpress.com/article/cities/delhi/delhi-
They use a method called Fractional Distillation govt-medical-oxygen-policy-second-covid-19-wave-
Method to produce pure oxygen from 7486001/
atmospheric air, which consists mostly of
nitrogen and oxygen - 78% nitrogen, 21% oxygen
and remaining 1% other gases including argon, Q.4) Ans: A
carbon dioxide, neon, helium, and hydrogen. It is Exp:
then transported using cryogenic containers. ● Statement 1 is correct: Gravitational waves
Oxygen can also be produced non- are distortions or ‘ripples’ in the fabric of
cryogenically, in gaseous form, using selective space-time caused by some of the most
adsorption(Pressure Swing Adsorption violent and energetic processes in the
Technique). This method leverages the property Universe. They transport energy as
that under high pressure, gases tend to be gravitational radiation and pass through
attracted to solid surfaces. The higher the matter without interacting with it. Strongest
pressure, the more the adsorption of gas. sources of gravitational waves are among

e
● Statement 2 is incorrect: According to the Drug enigmatic objects in our universe like black

n
Prices Control Order, 2013, Medical oxygen is holes, supernovas, neutron stars and Big Bang.

li
placed under the National List of Essential Gravitational waves were first predicted in
Medicines (NLEM). The National Pharmaceutical 1916 by Albert Einstein on the basis of his

n
Pricing Authority will monitor and control the Theory of General Relativity. o
prices of the National List of Essential Medicines ● Statement 2 is correct: LIGO stands for Laser
l.
(NLEM). So, the NPPA controls and monitors the Interferometer Gravitational-Wave
a

medical oxygen prices in India.NPPA is an Observatory. It is a large-scale Physics


ri

independent body set up in 1997. It is under the experiment carried out to detect Gravitational
Department of Pharmaceuticals, Ministry of waves.Gravitational waves will provide a test
te

Chemicals and Fertilizers. of Einstein's theory of general relativity under


● Statement 3 is correct: If it is used above the extreme conditions of gravity where it has
a

prescribed amount, the medical oxygen creates never before been tested. They will also give
m

certain side effects. This is why the doctor's more information about the unimaginably
c

prescription of oxygen is essential. Medical dense form of matter that makes up neutron
Oxygen can cause Retrolenticular fibroplasia in stars. Gravitational waves will also tell us
s

premature infants if they are exposed to about how many objects like black holes and
p

oxygen concentrations greater than 40%. In neutron stars exist in the Universe.They will
.u

short, it is a condition of abnormal growth of give us insight into what happens during some
blood vessels in the eye. Retrolenticular of the Universe's most violent explosions.
w

fibroplasia is the leading cause of child blindness ● Statement 3 is incorrect: The LIGO india
w

today in the world. The disease occurs in infants observatory is scheduled for completion in
of low weight at birth, commonly those born 2024, and will be built in the Hingoli District of
w

prematurely.Excessive or inappropriate Maharashtra. LIGO India is a planned

DPP 2023 DAY 160 20


https://upscmaterial.online/
Download From - https://upscmaterial.online/

Contact us :info@onlyias.com

OnlyIAS Nothing Else Visit :dpp.onlyias.in


Contact : +91-7007 931 912

advanced gravitational-wave observatory to whose goal was the complete mapping and
be located in India as part of the worldwide understanding of all the genes of human
network. LIGO- India project is piloted by beings. The genetic sequence made available
Department of Atomic Energy (DAE) and in 2003 from the HGP was known as standard
Department of Science and Technology human reference genome, known as Genome
(DST).The LIGO-India project will be jointly Reference Consortium build 38 (GRCh38)
coordinated and executed by three Indian ● Statement 3 is correct : Now, the fully
research institutions: the Inter-University sequenced genome is the result of the efforts
Centre for Astronomy and Astrophysics of a global collaboration called the Telomere-
(IUCAA), Pune and Department of Atomic 2-Telomere (T2T) project. The invention of new
Energy organisations: Institute for Plasma methods of DNA sequencing and
Research (IPR), Gandhinagar and the Raja computational analysis helped complete the
Ramanna Centre for Advanced Technology reading of the remaining 8% of the genome.A
(RRCAT), Indore. With its establishment, India global collaboration called the Telomere-
will join the global network of gravitational 2Telomere (T2T) project has managed to
wave detectors. sequence the remaining 8% of the human
genome, 92% of which was already
Reference: sequenced through the Human Genome
https://www.thehindu.com/news/national/other- Project (HGP).
states/225-hectare-maha-land-handed-over-for-first-
ligo-project-in-india/article37554103.ece Reference:

e
https://indianexpress.com/article/explained/complet

n
Q.5) Ans: B e-human-genome-explained-7848706/

li
Exp:
● Statement 1 is not correct: A genome refers to

n
all of the genetic material in an organism, and Q.6) Ans: A o
the human genome is mostly the same in all Exp:
l.
people, but a very small part of the DNA does ● Statement 1 is correct: The steel slag is
a

vary between one individual and another. generated from a steel furnace burning at
ri

Every organism’s genetic code is contained in around 1,500-1,600 degree centigrade in the
its Deoxyribose Nucleic Acid (DNA), the form of molten flux material as an impurity.
te

building blocks of life. The discovery that DNA The molten material is poured into the slag pits
is structured as a “double helix” by James for cooling as per the customized procedure
a

Watson and Francis Crick in 1953, started the and further processed to develop stable steel
m

quest for understanding how genes dictate life, slag aggregates.


c

its traits, and what causes diseases. Each


genome contains all of the information needed ● Statement 2 is incorrect: The approximate
s

to build and maintain that organism. In construction cost per square meter of a
p

humans, a copy of the entire genome contains processed steel slag road is Rs 1,150 as against
.u

more than 3 billion DNA base pairs. Rs 1,300 for a bitumen road and Rs 2,700 for a
● Statement 2 is not correct : Human Genome cement or a concrete one. The lifespan of a
w

Project (HGP). HGP was started in 1988 by a cement or concrete road is over 30 years while
w

special committee of the U.S. National that of bitumen and steel slag road is around
Academy of Sciences and was an 15 years.
w

international, collaborative research program

DPP 2023 DAY 160 21


https://upscmaterial.online/
Download From - https://upscmaterial.online/

Contact us :info@onlyias.com

OnlyIAS Nothing Else Visit :dpp.onlyias.in


Contact : +91-7007 931 912

Hence steel slag used for road construction batteries comes with issues such as scarcity,
cost per square metre of slag road is less than high prices, and safety concerns.
bitumen road.
● Statement 3 is correct: The larger size and
● Statement 3 is incorrect: The carbon footprint higher charge density of calcium ions relative
for steel slag roads is much lower as the ones to lithium impairs diffusion kinetics and cyclic
built using natural aggregates are mined and stability.
processed. Besides, after mining and crushing, That's why the higher ionic charge and the
the material also needs to be transported from larger size of calcium ions relative to lithium
one place to another. When it comes to steel makes it very challenging to insert calcium ions
slag road, there is no blasting, drilling or into the battery electrodes.
crushing as the material is waste coming out of As a result, despite having more electrons than
a steel industry which is processed and lithium, a lithium cation will carry a higher
converted to the form of aggregate material charge density than a calcium cation due to the
used for construction. lithium cation’s smaller ionic radius i.e
0.152nm while calcium have 0.197.
Reference: To overcome this problem scientists are
https://www.google.com/amp/s/indianexpress.com/a developing a special class of materials called
rticle/explained/surat-indias-first-steel-slag-road- molybdenum vanadium oxides that contain
7847675/lite/ large hexagonal and heptagonal shaped
https://swarajyamag.com/infrastructure/bro-to-build- channels or tunnels that run through the

e
border-road-in-arunachal-pradesh-using-steel-slag material.

n
Reference:

li
Q.7) Ans: B https://www.theengineer.co.uk/content/news/calciu
Exp: m-ion-battery-breakthrough-could-offer-alternative-

n
● Statement 1 is incorrect: Monovalent atoms to-lithium-ion-cells/ o
are those atoms which show a valency of 1,
l.
while divalent atoms are those which show a
a

valency of 2. For example, sodium (Na), Q.8) Ans: D


ri

chlorine, potassium etc are monovalent while Exp:


magnesium, oxygen, calcium are divalent. The ● Statement 1 is correct : Low-temperature
te

calcium ion is divalent, and hence one ion thermal desalination (LTTD) is a technique for
insertion will deliver two electrons per ion desalination. It works on the notion that
a

during battery operation. This allows for a water evaporates at lower temperatures, at
m

highly efficient battery with reduced mass and low pressures. It utilizes vacuum pumps for
c

volume of calcium ions. creating a low pressure and low-temperature


environment, resulting into evaporation of
s

● Statement 2 is correct: Calcium ions could be water at a temperature gradient of 8 °C.


p

used as a greener, more efficient, and less Cooling water is supplied from deep sea
.u

expensive energy storage alternative to depths, which is pumped up to condense


lithium-ions in batteries because of its evaporated water vapor. We get purified water
w

abundance and low cost. The researchers from from the resulting condensate.
w

Rensselaer Polytechnic Institute in the US ● Statement 2 is correct : Three desalination


noted that long-term use of lithium-ion plants based on the LTTD technology have
w

been developed and demonstrated at

DPP 2023 DAY 160 22


https://upscmaterial.online/
Download From - https://upscmaterial.online/

Contact us :info@onlyias.com

OnlyIAS Nothing Else Visit :dpp.onlyias.in


Contact : +91-7007 931 912

Kavaratti, Agati and Minicoy Islands of Union primary purpose of biodegradable plastics is to
Territory of Lakshadweep. The capacity of replace traditional plastics that persist in
each of these LTTD plants is 1 Lakh litre of landfills and harm the environment. Therefore,
potable water per day. Union Territory of the ability of microorganisms to break down
Lakshadweep has been entrusted the work of these plastics is an incredible environmental
establishing 6 more LTTD plants at Amini, advantage. Despite the release of C02 into
Androth, Chetlet, Kadmat, Kalpeni and Kiltan environment, biodegradable plastics leave a
with a capacity of 1.5 lakhs litres/day. Further, smaller footprint then petroleum-based
the LTTD technology is found suitable for plastics that accumulate in landfills and cause
Lakshadweep islands where the required heavy pollution which is why they are explored
temperature difference of about 15⁰ C as alternatives to traditional plastic.
between sea surface water and deep-sea ● Statement 2 is correct : Biodegradable plastics
water is found in the vicinity of Lakshadweep are commonly produced with renewable raw
coasts only as of now. materials, micro-organisms, petrochemicals,
● Statement 3 is correct : Ministry of Earth or combinations of all three. It is also often
Sciences (MoES) through its autonomous called bio-based plastic. It can either be made
Institute National Institute of Ocean by extracting sugar from plants like corn and
Technology (NIOT) has developed Low sugarcane to convert into polylactic acids
Temperature Thermal Desalination (LTTD) (PLAs), or it can be made from
technology for conversion of sea water to polyhydroxyalkanoates (PHAs) engineered
potable water which has been successfully from microorganisms.

e
demonstrated in Lakshadweep islands. NIOT is ● Statement 3 is not correct : While the words

n
managed by a Governing Council and the "bioplastic" and "biodegradable plastic" are

li
Director is the head of the Institute. Major aim similar, they are not synonymous. Not all
of starting NIOT under the Ministry of Earth bioplastics plastics that are derived partly or

n
Sciences, is to develop reliable indigenous entirely from biomass are biodegradable, and
o
technologies to solve the various engineering some biodegradable plastics are fully
l.
problems associated with harvesting of non- petroleum based. As more companies are
a

living and living resources in the Indian keen to be seen as having "Green" credentials,
ri

Exclusive Economic Zone (EEZ), which is about solutions such as using bioplastics are being
two-thirds of the land area of India. investigated and implemented more. However
te

there are many skeptics who believe that


Reference : bioplastics will not solve problems others
a

https://newsonair.com/2022/08/14/roshni-indias- expect.
m

first-saline-water-lantern-uses-seawater-for-power/
c

https://pib.gov.in/PressReleaseIframePage.aspx?PRID Reference : https://newsonair.com/2022/08/17/govt-


=1843523 approves-startup-loan-for-manufacturing-
s

commercialising-compostable-plastic/
p

Q.9) Ans: C
.u

Exp:
● Statement 1 is not correct :Biodegradable Q.10) Ans: B
w

plastics are plastics that can be decomposed Exp:


w

by the action of living organisms, usually ● Statement 1 is correct: Department of


microbes, into water, carbon dioxide, Biotechnology- National Institute of
w

inorganic compounds and biomass. The Immunology (DBT-NII) receives ASPAGNII

DPP 2023 DAY 160 23


https://upscmaterial.online/
Download From - https://upscmaterial.online/

Contact us :info@onlyias.com

OnlyIAS Nothing Else Visit :dpp.onlyias.in


Contact : +91-7007 931 912

trademark for India’s First Indigenous Tumour varieties to the farmers. Kalanamak is
Antigen SPAG9. SPAG9 was discovered by Dr traditionally grown using no fertiliser,
Anil Suri in 1998. ASPAGNIITM may revolutionise herbicide and pesticide, which makes it
cancer treatment. suitable for organic cultivation.It's a successful
● Statement 2 is incorrect: Currently, ASPAGNII is adapter to usar soils characterised by higher
being used in dendritic cell (DC) based salt concentration and high Ph.
immunotherapy in cervical, ovarian cancer and ● Statement 2 is correct: Kala Namak (non-
will be used in breast cancer. Immunotherapy basmati scented rice variety) is highly resistant
is a new approach that exploits the body’s inner to notorious common rice diseases such as
capability to put up a fight against cancer. With panicle blast, stem rot and brown spot.It is
this approach, either the immune system is given drought tolerant and normally grown under
a boost, or the T cells are “trained’’ to identify rain-fed conditions. The water requirement is
recalcitrant cancer cells and kill them. In this quite low as compared to basmati.
personalised intervention, those patients ● Statement 3 is correct: It is also known as
expressing SPAG9 protein can be treated with 'Buddha ka Mahaprasad' (an offering to Lord
the DC-based vaccine approach. Buddha) . This scented variety has been
cultivated since 600 BC i.e., since and probably
Reference: before the Buddhist era. It is also known as
https://pib.gov.in/PressReleaseIframePage.aspx?PRI ‘Buddha rice’ as it is believed that Mahatma
D=1724462 Buddha broke His fast on the day of
https://www.biospectrumindia.com/news/58/18843 enlightenment when kheer made from

e
/nii-receives-trademark-for-indias-first-indigenous- Kalanamak rice was offered to Him by Sujatha.

n
tumour-antigen-spag9.html It has been rebranded as 'Buddha Rice',

li
presenting it as an offering made to the
Buddhist monks, in order to promote it in

n
Q.11) Ans: D Buddhist countries. The Government of Uttar
o
Exp: Pradesh is promoting Kala Namak rice under
l.
● Statement 1 is incorrect: Kalanamak (‘Kala’ One District One Product (ODOP)and has
a

means black derived from the husk and the sanctioned a project of Rs. 12.00 crore for
ri

suffix ‘namak’ means salt) is one of the finest promotion of Kala Namak rice. It has also been
quality aromatic rice grown in India. Kala featured in the book 'Speciality Rices of the
te

Namak rice is grown in the tarai area of World' by the Food and Agriculture
Siddharth Nagar and ten neighbouring Organization of the United Nations.Kalanamak
a

districts of eastern Uttar Pradesh. The famed rice was granted the Geographical Indication
m

Kalanamak is rated as very high grade in rice (GI) Tag in 2012 by the Government of India.
c

varieties and grows abundantly in the


Himalayan Tarai belt of Eastern Uttar Pradesh Reference:
s

bordering Nepal which comprises of the https://pib.gov.in/PressReleasePage.aspx?PRID=180


p

districts of Siddharthnagar, Gorakhpur, 9698


.u

Maharajganj, Kushinagar, Basti, Deoria, Sant https://timesofindia.indiatimes.com/life-style/food-


Kabir Nagar, Barabanki, Gonda, Bahraich and news/recently-awarded-kalanamak-rice-of-up-was-
w

Balrampur and is popularly called as the buddhas-gift-to-


w

scented black pearl of Uttar Pradesh. It is a mankind/photostory/90989475.cms?picid=90989489


scented rice variety which is generally offered https://www.downtoearth.org.in/coverage/kalanam
w

at a better price as compared to other rice ak-101

DPP 2023 DAY 160 24


https://upscmaterial.online/
Download From - https://upscmaterial.online/

Contact us :info@onlyias.com

OnlyIAS Nothing Else Visit :dpp.onlyias.in


Contact : +91-7007 931 912

Q.12) Ans: A achieving both power generation and


Exp: agricultural yield. It can resolve land availability
● Statement 1 is correct: R-value or R-factor or R0 constraints for setting up solar farms and the
(R-naught) or Reproduction Rate refers to how crop cover in between the rows of the solar
many persons an infected person infects on an panels helps check soil erosion to reduce the
average. It indicates the speed at which the dust load on the panels.
Covid infection is spreading in the country, along ● Statement 2 is incorrect: Setting up of such
with the ‘efficiency’ of the spread. The R number integrated agri-photovoltaic farms is typically a
is a key factor in gauging the coronavirus cost-intensive proposition, heavy investments
pandemic.It refers to the 'effective reproduction may be unaffordable for most Indian farmers.It
number' of COVID-19. is advisable to encourage the solar energy
● Statement 2 is incorrect: An R-Factor of 1 would entrepreneurs to consider joining hands with the
mean that an infected person is giving the virus farmers on mutually agreed terms, instead of
to 1 other person. An R-Factor of 5 would mean going in for solo photovoltaic units.It can
that 1 person is spreading the virus to 5 other contribute to achieving the objectives of
people.An R value of 1 is a crucial threshold. An doubling farm incomes by 2022.
R-value of over 1 indicates that the number of
active cases has increased. R should be Reference:
contained below 1 to control the pandemic. An https://pib.gov.in/PressReleasePage.aspx?PRID=170
R number lower than 1 indicates that the disease 3539
will stop spreading as there aren't enough https://www.weforum.org/agenda/2022/07/agrivol

e
people being infected to sustain the outbreak. taic-farming-solar-energy/

n
li
Reference:
https://economictimes.indiatimes.com/news/india/i Q.14) Ans: B

n
ndias-r-value-increases-above-1-for-first-time-in-3- Exp: o
months-says-researcher/articleshow/90956426.cms ● Statement 1 is correct: WHO defines
l.
https://www.bbc.com/news/health-52473523 Fortification as the practice of deliberately
a

https://www.weforum.org/agenda/2020/05/covid- increasing the content of an essential


ri

19-what-is-the-r-number/ micronutrient, i.e. vitamins and minerals


(including trace elements) in a food, so as to
te

improve the nutritional quality of the food


Q.13) Ans: B supply and provide a public health benefit with
a

Exp: minimal risk to health.Fortification is necessary


m

● Statement 1 is correct: Agrivoltaic farming is to address deficiency of micronutrients or


the practice of growing crops underneath solar
c

micronutrient malnutrition, also known as


panels.Agri-voltaic farming- is a mixed system of “hidden hunger”, a serious health risk.
s

solar energy production associating solar panels ● Statement 2 is incorrect: Food fortification is
p

and crops at the same time on the same land defined as the practice of adding vitamins and
.u

area.It is an evolving practice in sustainable minerals to commonly consumed foods during


development that combines the production of processing to increase their nutritional value.It is
w

food and energy. PV-modules are installed in a proven, safe and cost-effective strategy for
w

Agri voltaic system (AVS)inclination angle equal improving diets and for the prevention and
to the latitude of the place of installation. Agri- control of micronutrient deficiencies.The Food
w

PV overcomes the present either-or situation by Safety and Standards Authority of India (FSSAI),

DPP 2023 DAY 160 25


https://upscmaterial.online/
Download From - https://upscmaterial.online/

Contact us :info@onlyias.com

OnlyIAS Nothing Else Visit :dpp.onlyias.in


Contact : +91-7007 931 912

defines and set standards for fortification as project, Sagarmala waterways plan, ports and
“deliberately increasing the content of essential the UDAN scheme.
micronutrients in a food so as to improve the ● Statement 2 is incorrect: This mission will
nutritional quality of food and to provide public cover Economic Zones like textile clusters,
health benefit with minimal risk to health”.It has pharmaceutical clusters, defence corridors,
also launched the ‘F+’ logo to be displayed on electronic parks, industrial corridors, Agri
labels of fortified food products for easy zones will be covered to improve connectivity
identification by consumers. & make Indian businesses more competitive.
● Statement 3 is incorrect:The Ministry of ● Statement 3 is correct: It will leverage
Consumer Affairs, Food and Public Distribution technology extensively including spatial
had launched a centrally sponsored pilot scheme planning tools with ISRO imagery developed by
on “Fortification of Rice and its Distribution BiSAG-N (Bhaskaracharya National Institute for
under Public Distribution System (PDS)” for a Space Applications and Geoinformatics).This
period of three years beginning 2019-20 with will be used for real-time monitoring of
focus on 15 districts. Government has also projects.It will also allow various government
decided to scale up the scheme to cover the departments to track, in real-time and at one
Integrated Child Development Scheme (ICDS) centralised place, the progress of various
and Mid-Day Meal (MDM) scheme in projects, especially those with multi-sectoral
'aspirational districts'. Food Corporation of India and multi-regional impact.
(FCI) has made it mandatory for the rice millers
in all states to install blending infrastructure for Reference: https://www.india.gov.in/spotlight/pm-

e
the production of fortified rice. gati-shakti-national-master-plan-multi-modal-

n
connectivity

li
Reference: https://pib.gov.in/PressReleaseIframePage.aspx?PRI
https://indianexpress.com/article/explained/what- D=1763638

n
is-fortified-rice-how-is-it-prepared-7860432/ o
https://pib.gov.in/PressReleasePage.aspx?PRID=181
l.
4826 Q.16) Ans: A
a

Exp:
ri

● A ‘flex-fuel engine’ is an internal combustion


Q.15) Ans: B engine that can run on more than one type of
te

Exp: fuel and also a mixture.Flex-fuel engines are


● Statement 1 is correct: PM Gati-Shakti capable of running on 100 percent petrol or
a

National Master Plan is a digital platform that ethanol and are already available in countries
m

connects 16 ministries including Roads and such as Brazil, USA and Canada.In order to
Highways, Railways, Shipping, Petroleum and
c

accelerate the introduction of Flex Fuel


Gas, Power, Telecom, Shipping, and Aviation, vehicles, the Production Linked Incentive (PLI)
s

with a view to ensuring holistic planning and scheme has included automobile & auto
p

execution of infrastructure projects.PM Gati components and auto components of flex fuel
.u

Shakti aims to institutionalize holistic planning engines.Flex Fuel Vehicles are capable of
for major infrastructure projects. The projects running on a combination of 100% Petrol or
w

will be designed and executed with a common 100% bio-ethanol and their blends( a
w

vision and will incorporate the infrastructure combination of both). along with strong Hybrid
schemes of various ministries and state Electric technology.FFVs are aimed at reducing
w

governments such as the Bharatmala road the use of polluting fossil fuels and cutting

DPP 2023 DAY 160 26


https://upscmaterial.online/
Download From - https://upscmaterial.online/

Contact us :info@onlyias.com

OnlyIAS Nothing Else Visit :dpp.onlyias.in


Contact : +91-7007 931 912

down harmful emissions.The wide uptake of ➢ Topographic: typically uses a near-


ethanol or methanol as a fuel is intended to infrared laser to map the land.
create an additional revenue stream for ➢ Bathymetric: uses water-penetrating
farmers.This will provide direct benefits to green light to also measure seafloor
farmers and help in doubling the farmer’s and riverbed elevations.
income.They provides option for using
biofuels, which are better than petrol (are Reference:
import substitute, cost effective, pollution-free https://indianexpress.com/article/india/govt-
and indigenous).They also help in addressing launches-lidar-survey-reports-to-augment-water-in-
the problems of surplus food grains (can be forest-areas-7376100/
used in making ethanol) and reduce https://pib.gov.in/PressReleasePage.aspx?PRID=173
dependence on oil imports. 0351

Reference:
https://www.thehindubusinessline.com/blexplainer
Q.18) Ans: C
/all-you-need-to-know-about-flex-fuel-
Exp:
vehicles/article38061000.ece
● Statement 1 is correct: Biotech-PRIDE
https://www.livemint.com/industry/infrastructure/
(Promotion of Research and Innovation
govt-targeting-construction-pace-of-50-km-a-day-
through Data Exchange) Guidelines was
for-national-highways-11647089924405.html
released by the Department of Biotechnology
(DBT), Ministry of Science and Technology.

e
These guidelines "aim at providing a well-
Q.17) Ans: A

n
defined framework and guiding principles to
Exp:

li
facilitate and enable sharing and exchange of
● Statement 1 is correct: LiDAR technology was

n
high-throughput, high-volume biological data
used to create 3-D images of the project areas
generated by research groups across the
o
to recommend soil and water conservation
country".
l.
structures.LiDAR is a remote sensing technology
They have been introduced in the interest of
a

that uses light in the form of a pulsed laser to


data sharing in order to further research and
ri

measure ranges (distances) to a target.These


innovation in India.
light pulses—combined with other data
te

recorded by the airborne system— generate


● Statement 2 is correct: The website of the
precise, three-dimensional information about
a

Indian Biological Data Centre (IBDC) was also


the shape of the Earth and its surface
launched alongside the Biotech-PRIDE
m

characteristics. It is similar to radar and sonar


guidelines.
(that use radio and sound waves, respectively).
c

The guidelines will, in fact, be implemented


● Statement 2 is incorrect: A LiDAR instrument
s

through the IBDC at the Department of


principally consists of a laser, a scanner, and a
p

Biotechnology supported Regional Centre for


specialized GPS receiver. Recently, Ministry of
Biotechnology, Faridabad.
.u

Environment, Forest and Climate Change


1. The Biotech PRIDE Guidelines will facilitate this
released LiDAR (light detection and ranging)
w

and enable exchange of information to


based reports mapping out the water
promote research and innovation in different
w

requirement within forest areas in 10


research groups across the country.
states.There are following two types of LiDAR:
2. The Biotech-PRIDE (Biotech Promotion of
w

Research and Innovation through Data

DPP 2023 DAY 160 27


https://upscmaterial.online/
Download From - https://upscmaterial.online/

Contact us :info@onlyias.com

OnlyIAS Nothing Else Visit :dpp.onlyias.in


Contact : +91-7007 931 912

Exchange) guidelines aim at providing a well- ● Statement 2 is correct: The functions of the
defined framework and guiding principle to Board include:
facilitate and enable sharing and exchange of (i) Advising governments on all issues related
biological knowledge, information and data to establishing DNA laboratories or Data Banks.
and is specifically applicable to high- (ii) Granting accreditation to DNA laboratories.
throughput, high-volume data generated by (iii) the Board is required to ensure that all
research groups across the country. information relating to DNA profiles with the
3. These guidelines do not deal with generation Data Banks, laboratories, and other persons
of biological data per se but are an enabling are kept confidential.
mechanism to share and exchange information The Bill provides for the establishment of a
and knowledge generated as per the existing DNA Regulatory Board, which will supervise
laws, rules, regulations and guidelines of the the DNA Data Banks and DNA laboratories.
country. The Secretary, Department of Biotechnology,
will be the ex officio Chairperson of the Board.
Reference:
https://www.google.com/amp/s/swarajyamag.com/a ● Statement 3 is correct: The use of DNA
mp/story/science%252Fbiotech-pride-guidelines-to- technology, while not infallible, will minimise
provide-framework-for-sharing-of-biological-data errors in criminal investigations and “improve
https://pib.gov.in/PressReleasePage.aspx?PRID=1740 the justice delivery system.
712 Using DNA effectively during criminal
investigations requires proper crime scene

e
examination, trained and reliable policing, a

n
Q.19) Ans: C trusted chain of custody of samples, reliable

li
Exp: analysis, and proper use of expert evidence in
● Statement 1 is incorrect: The DNA Technology court.

n
(Use and Application) Regulation Bill provides Without these prerequisites, a DNA database
o
for the regulation of use of DNA technology for will exacerbate rather than solve problems in
l.
establishing the identity of certain persons. the criminal justice system.
a

Under the Bill, DNA testing is allowed only in For example, false matches or
ri

respect of matters listed in the Schedule to the misinterpretation or planting of evidence can
Bill. These include offences under the Indian lead to the travesty of justice.
te

Penal Code, 1860, and for civil matters such as Hence statement 3 is correct.
paternity suits. Further, the Schedule includes
a

DNA testing for matters related to


m

establishment of individual identity.


c

It allows the profiling of victims, those


accused of crimes, and those reported
s

missing, and storing of their DNA information.


p

The Bill provides for the establishment of a


.u

National DNA Data Bank and Regional DNA


Data Banks, for every state, or two or more
w

states under which DNA information will be


Reference:
w

stored.
w

DPP 2023 DAY 160 28


https://upscmaterial.online/
Download From - https://upscmaterial.online/

Contact us :info@onlyias.com

OnlyIAS Nothing Else Visit :dpp.onlyias.in


Contact : +91-7007 931 912

https://www.google.com/amp/s/m.thewire.in/article/ develop and conserve Indigenous Breeds in a


government/dna-technology-regulation-bill-seen-to- holistic and scientific manner.
harm-minorities-hurt-privacy/amp
● Advanced Reproductive Technology: Including
https://prsindia.org/billtrack/the-dna-technology-use- Assisted Reproductive Technique- In-vitro
and-application-regulation-bill-2019 Fertilization (IVF)/ Multiple Ovulation Embryo
Transfer (MOET) and sex-sorted semen
Q.20) Ans: B technique to improve the availability of
Exp: disease-free female bovines.
● The government of India has launched a ● National Bovine Genomic Center for
campaign namely Kalam Program for Indigenous Breeds (NBGC-IB) will be
Intellectual Property Literacy and Awareness established for selection of breeding bulls of
Campaign (KAPILA). high genetic merit at a young age using highly
The program aims to increase awareness precise gene-based technology.
regarding protection and exploitation of
Intellectual Property (IP) and also to provide
funding support in order to promote filing of Reference:
the Intellectual Property (IP)in Higher https://pib.gov.in/PressReleasePage.aspx?PRID=1783
Education Institutions. 482
A KAPILA portal has also been launched for this
purpose.

e
Under the initiative, activities like IP Clinic, Q.21) Ans: A

n
Case Studies/ Articles regarding Innovation Exp:

li
and Intellectual Property and online awareness
programs and National Intellectual Property ● Statement 1 is correct: The Special 301 Report

n
Literacy Week (NIPLW) have been conducted. is prepared annually by the Office of the United
o
● A DNA forecasting technique for Indigenous States Trade Representative (USTR) that
l.
milch animals: identifies trade barriers to United States
a

Rashtriya Gokul Mission companies and products due to the intellectual


ri

Rashtriya Gokul Mission comprises of two property laws, such as copyright, patents and
components namely: trademarks, in other countries.By statute, the
te

The national program for bovine breeding and annual Special 301 Report include a list of
National mission for bovine productivity. "Priority Foreign Countries"that are judged to
a

Underlying objectives of the Mission: have inadequate intellectual property laws;


m

1. Development and conservation of indigenous these countries may be subject to sanctions. In


c

breeds in a focused and scientific manner. addition, the report contains a "Priority Watch
2. Undertake breed improvement program for List" and a "Watch List", containing countries
s

indigenous cattle breeds so as to improve the whose intellectual property regimes are
p

genetic makeup and increase the stock. deemed of concern.


.u

3. Enhance milk production and productivity. Algeria, Argentina, Chile, China, India,
Distribute disease free high genetic merit bulls Indonesia, Russia, Saudi Arabia, Ukraine and
w

for natural service. Venezuela are on the Priority Watch List.


w

Hence statement 1 is correct


National Kamdhenu Breeding Centre: (NKBC) is
w

being established as a Centre of Excellence to

DPP 2023 DAY 160 29


https://upscmaterial.online/
Download From - https://upscmaterial.online/

Contact us :info@onlyias.com

OnlyIAS Nothing Else Visit :dpp.onlyias.in


Contact : +91-7007 931 912

● Statement 2 is incorrect : Special 301 report in 1967 "to encourage creative activity, to
published annually by the Office of the United promote the protection of intellectual
States Trade Representative (USTR). property throughout the world". WIPO
● World Intellectual Property Organization currently administers 26 international treaties.
(WIPO), international organization designed to It is headquartered in Geneva, Switzerland.
promote the worldwide protection of both WIPO-administered International Treaties and
industrial property (inventions, trademarks, Conventions are there to which India is a
and designs) and copyrighted materials Member are:
(literary, musical, photographic, and other 1. Paris Convention for the Protection of
artistic works). The organization, established Industrial Property
by a convention signed in Stockholm in 1967, 2. Convention Establishing the World Intellectual
began operations in 1970 and became a Property Organization
specialized agency of the United Nations in 3. Berne Convention for the Protection of Literary
December 1974. It is headquartered in and Artistic Works Patent Cooperation Treaty
Geneva. 4. Washington Treaty on Intellectual Property in
respect of Integrated Circuits.
Reference: 5. Nairobi Treaty on the Protection of the
https://www.google.com/amp/s/indianexpress.com/a Olympic Symbol
rticle/india/ip-protection-us-puts-india-on-priority-
watch-list-7297579/lite/ ● Statement 2 is correct: Objective of the library:
1. Its objective is to protect the ancient and

e
traditional knowledge of the country from

n
Q.22) Ans: B exploitation through biopiracy and unethical

li
Exp: patents, by documenting it electronically and
● Statement 1 is incorrect: The Traditional classifying it as per international patent

n
Knowledge Digital Library (TKDL) project, classification systems. o
initiated in India in 2001, is a collaboration Hence statement 2 is correct.
l.
between the Council of Scientific and Industrial 2. Apart from that, the non-patent database
a

Research (CSIR), Ministry of Science and serves to foster modern research based on
ri

Technology, and the Department of Ayurveda, traditional knowledge, as it simplifies access to


Yoga and Naturopathy, Unani, Siddha and this vast knowledge of remedies or practices.
te

Homeopathy (AYUSH), Ministry of Health and 3. It seeks to prevent the granting of patents for
Family Welfare, of India. It was not set up by products developed utilizing TK where there
a

the World Intellectual Property Organisation. has been little, if any, inventive step
m

It is being implemented at the CSIR. 4. It intends to act as a bridge between


c

The interdisciplinary team involved in the information recorded in ancient Sanskrit and
creation of the TKDL for Indian Systems of patent examiners (with its database containing
s

Medicine included traditional medicine information in a language and format


p

experts (Ayurveda, Unani, Siddha and Yoga), understandable to patent examiners)


.u

patent examiners, IT experts, scientists and 5. It facilitates access to information not easily
technical officers. available to patent examiners, thereby
w

minimizing the possibility that patents could be


w

● World Intellectual Property Organisation granted for “inventions” involving only minor
(WIPO) : It is one of the oldest specialised or insignificant modifications.
w

agencies of the United Nations. It was created

DPP 2023 DAY 160 30


https://upscmaterial.online/
Download From - https://upscmaterial.online/

Contact us :info@onlyias.com

OnlyIAS Nothing Else Visit :dpp.onlyias.in


Contact : +91-7007 931 912

● Statement 3 is incorrect: "the best techniques from machine learning


The TKDL contains documentation of publicly and systems neuroscience to build powerful
available traditional knowledge. general-purpose learning algorithms".
1. It relates to Ayurveda, Unani, Siddha and Yoga Various implications of AlphaFold are:
2. It is in digitized format. 1. Knowing protein structure and function is
3. It is available in five languages: English, essential to understanding human diseases.
German, French, Japanese and Spanish. 2. Protein structures using x-ray crystallography,
Hence statement 3 is incorrect. nuclear magnetic resonance spectroscopy, or
cryogenic electron microscopy. These
Reference: techniques are not just time-consuming, they
https://journalsofindia.com/traditional-knowledge- often take years and are based mainly on trial-
digital-library-tkdl/ and-error methods.
3. AlphaFold makes the process of protein
structuring easier. It is a watershed movement
Q.23) Ans: C in science and structural biology in particular.
Exp: 4. AlphaFold has already helped hundreds of
● Statement 1 is incorrect: AlphaFold is an AI- scientists accelerate their discoveries in
based protein structure prediction tool. It is vaccine and drug development since the first
based on a computer system called deep public release of the database nearly a year
neural network. It's Inspired by the human back.
brain, neural networks use a large amount of ● Statement 3 is correct:

e
input data and provide the desired output Its Working Mechanism:

n
exactly like how a human brain would. In this 1. AlphaFold is fed with protein sequences as

li
technology the real work is done by the black input. When protein sequences enter through
box between the input and the output layers, one end, the predicted three-dimensional

n
called the hidden networks. structures come out through the other. Hence
o
statement 3 is correct.
l.
● Statement 2 is incorrect: AlphaFold is 2. It uses processes based on “training, learning,
a

developed by Google AI offshoot DeepMind retraining and relearning.”


ri

and not by the World Health Organization. 3. The first step uses the available structures of
AlphaFold is an AI system developed by 1,70,000 proteins in the Protein Data Bank
te

DeepMind that predicts a protein’s 3D (PDB) to train the computer model. Then, it
structure from its amino acid sequence. It uses the results of that training to learn the
a

regularly achieves accuracy competitive with structural predictions of proteins not in the
m

experiment PDB.
c

DeepMind: DeepMind Technologies is a British 4. Once that is done, it uses the high-accuracy
artificial intelligence subsidiary of Alphabet Inc. predictions from the first step to retrain and
s

and research laboratory founded in 2010. relearn to gain higher accuracy of the earlier
p

DeepMind was acquired by Google in 2014. predictions.


.u

The company is based in London, with research


centres in Canada, France,and the United Reference:
w

States. In 2015, it became a wholly owned https://www.deepmind.com/research/highlighted-


w

subsidiary of Alphabet Inc, Google's parent research/alphafold


company. According to the company's website,
w

DeepMind Technologies' goal is to combine

DPP 2023 DAY 160 31


https://upscmaterial.online/
Download From - https://upscmaterial.online/

Contact us :info@onlyias.com

OnlyIAS Nothing Else Visit :dpp.onlyias.in


Contact : +91-7007 931 912

Q.24) Ans: C TradeMarks (CGPDTM) generally known as the


Exp: Indian Patent Office, is an agency under the
● Due to widespread popularity of DNA based Department for Promotion of Industry and
technological developments, new advances in Internal Trade which administers the Indian
ribonucleic acid (RNA) research often go law of Patents, Designs and Trade Marks.The
unnoticed. RNA—essential for regulation and National IPR Policy is a vision document that
expression of genes—has already been the aims to create and exploit synergies between
subject of research, in areas such as RNA all forms of intellectual property (IP),
interference (RNAi) and antisense concerned statutes and agencies.Few
technology.. RNAi technologies are also now objectives of IPR Policy:
known to formulate drugs capable of reducing ➢ Legal and Legislative Framework: To
cholesterol levels by half. This technology finds have strong and effective IPR laws,
immense importance in treating acute viral which balance the interests of rights
infections like acquired immunodeficiency owners with larger public interest.
syndrome (AIDS), perhaps because of the well- ➢ Administration and Management: To
studied life cycle and pattern of gene modernize and strengthen service-
expression of the human immunodeficiency oriented IPR administration.
virus (HIV).This technique is used for gene ➢ Commercialization of IPRs: Get value
mapping and annotation in plants. RNA for IPRs through commercialization.
interference has been used for studies on ● Statement 2 is incorrect: Patent evergreening
multiple plant species.RNAi has been used to refers to the continuing extension of patent

e
target specific gene, instead of silencing the rights. Section 3(d) Indian Patent Act doesn’t

n
entire gene,it induces sequence-specific gene allow the renewal of patent over a product by

li
silencing , sequences that can cause cancer. It introducing minor changes to it. It has been an
can also be used to treat bacterial diseases,

n
issue of concern for pharmaceutical companies
viruses, and parasites, relieve pain and even (Ex: Novartis issue on the issue of patent of
o
modulate sleep.RNAi has huge significance cancer drug Glivec).India records 572% growth
l.
within the Indian context, considering the in grants for patents in last 7 years A total of
a

deep-seated resistance over the years to Bt 28,391 patents were granted in 2020-21 as
ri

cotton and other genetically modified seeds. compared to 4,227 grants during 2013-14.
RNA-reliant solutions could be a viable
te

● Statement 3 is incorrect: Compulsory


alternative. Licensing (CL) enables a competent
a

government authority to licence the use of a


Reference:
m

patented invention to a third party or


https://www.livemint.com/Opinion/qQSEpNfNPUES
government agency without the consent of
huTezgvMaO/RNA-technologies-and-Indias-path-
c

the patent holder. Section 92 of the Patent


forward.html
s

Act, 1970, provides for issuing CL by the central


https://www.future-
p

government (permitted under the WTO’s


science.com/doi/10.2144/000112792
TRIPS (IPR) Agreement) in circumstances of
.u

“national emergency or in circumstances of


w

extreme urgency or in case of public non-


Q.25) Ans: A
commercial use”. Recently, Patent for black
w

Exp:
pepper micronutrient foliar formulation and
● Statement 1 is correct: The Office of the
Trademark for Tumour Antigen SPAG9 were
w

Controller General of Patents, Designs and


awarded.
DPP 2023 DAY 160 32
https://upscmaterial.online/
Download From - https://upscmaterial.online/

Contact us :info@onlyias.com

OnlyIAS Nothing Else Visit :dpp.onlyias.in


Contact : +91-7007 931 912

Reference: Reference :
https://www.thehindu.com/news/cities/kozhikode/ https://www.pib.gov.in/PressReleasePage.aspx?PRID=
icar-iisr-gets-patent-for-black-pepper-micronutrient- 1763772
mixture/article34719431.ece
https://www.legalserviceindia.com/legal/article-
7213-ipr-law-in-india.html Q.27) Ans: C
Exp:
● Statements 1 and 3 are correct: The following
Q.26) Ans: D are the concerns of the Genetically Modified
Exp: (GM) crops:
● Option 4 is correct : “One Health” : There is no 1. Ecological Concerns: Gene flow due to
single, internationally agreed upon definition cross pollination for the traits involving
of One Health.One Health is understood as a resistance can result in development
collaborative, multisectoral, and trans- of tolerant or resistant weeds that are
disciplinary approach working at local, difficult to eradicate. They could lead
national, and global levels – to achieve optimal to erosion of biodiversity and pollute
health and well-being outcomes recognizing gene pools of endangered plant
the interconnections between people, species.
animals, plants, and their shared 2. Biosafety Concerns: Gene transfer
environment.One Health issues include involves risk of toxicity due to nature
zoonotic diseases, antimicrobial resistance, of the product or changes in the

e
food safety and food security, vector-borne metabolism of organisms. Newer

n
diseases, environmental contamination, and proteins in GM crops, which have not
been consumed as foods, have the risk

li
other health threats shared by people,
animals, and the environment.The overarching of becoming allergens. Genes used for

n
purpose is to encourage collaborations in antibiotic resistance have led to
o
research and sharing of knowledge at multiple transfer of such genes to
l.
levels across various disciplines like human microorganisms, thereby aggravating
a

health, animal health, plants, soil, health problems such as antibiotic


ri

environmental and ecosystem health in ways resistance in bacteria.


that improve, protect and defend the health of 3. Socio-Economic Concerns: Risk of
te

all species. The Pilanesberg Resolution, 2001: patent enforcement can oblige
It was targeted at multilateral and bilateral farmers to depend on giant
a

donors and governmental authorities to engineering companies such as


m

consider potential wildlife health impacts in Mansanto for strains when their crops
are cross pollinated. These plants may
c

development projects.One World-One Health:


The Wildlife Conservation Society (WCS) be viable for only one growing season
s

introduced the term “One World-One Health” and would produce sterile seeds that
p

in 2007 along with 12 recommendations (the do not germinate. Farmers would need
.u

Manhattan Principles) that focused on to buy a fresh supply of seed each year.
establishing a more holistic approach to ● Statement 2 is incorrect: It is a benefit of GM
w

preventing epidemic disease and maintaining Crops. The Following are the benefits of GM
w

ecosystem integrity. Crops:


1. Increased food security for the
w

growing population.

DPP 2023 DAY 160 33


https://upscmaterial.online/
Download From - https://upscmaterial.online/

Contact us :info@onlyias.com

OnlyIAS Nothing Else Visit :dpp.onlyias.in


Contact : +91-7007 931 912

2. Better tolerance to harsh climatic from India. It is funded by the Council


conditions like cold, heat, drought, etc. for Scientific and Industrial Research.
3. Reduced usage of pesticides and 2. Genome India Project: It aims to
herbicides. collect 10000 genetic samples from
4. Prevention of loss of species to citizens across India, to build a
endemic disease. reference genome. It is by the
5. Improved Agricultural performance Department of Biotechnology.
(yields) with less labor and cost input. 3. Human Genome Project: An
6. Improved Crop Protection through international research effort to
introduction of resistance against determine DNA sequence of the entire
plant diseases. human genome. It began in 1990 and
was completed in 2003.
Reference: https://www.thehindu.com/children/the- ● Statement 4 is correct: Nanopore gene
science-behind-gm-crops/article60678649.ece sequencing: It enables direct, real-time
analysis of long DNA or RNA fragments at a
Q.28) Ans: B faster and cheaper rate than previously
Exp: possible with older technologies. It works by
● Statement 1 is correct: Genome sequencing monitoring changes to an electrical current as
means deciphering the exact order of bases nucleic acids are passed through a protein
pairs (complete DNA sequence) in an nanopore. Nucleic acids are essential for all
organism's genome. This entails sequencing all forms of life and are found in all cells and

e
of an organism's chromosomal DNA as well as viruses. Nucleic acids come in two natural

n
DNA contained in the mitochondria and, for forms called DNA and RNA. Resulting signal is

li
plants, in the chloroplast. A genome is an decoded to provide specific DNA or RNA
organism’s complete set of DNA. It includes all sequence.

n
chromosomes, which houses DNA, and genes o
(specific sections of DNA). Human genome Reference: https://www.nih.gov/news-events/news-
l.
contains about 3 billion base pairs that spell releases/researchers-generate-first-complete-gapless-
a

out the instructions for making and sequence-human-genome


ri

maintaining a human being. https://nanoporetech.com/applications/dna-


● Statement 2 is incorrect: Genome Sequencing nanopore-sequencing
te

can help in the following:


1. Revealing population-level
a

predispositions to diseases. Q.29) Ans: C


m

2. Identifying genomic causes of rare Exp:


c

disease. ● Statement 1 is incorrect: India’s first graphene


3. Improving the understanding of how innovation center to be established in Thrissur,
s

viruses spread and evolve. Kerala. It is a joint venture of Digital University


p

4. Identifying genetic mutation. of Kerala, Centre for Materials for Electronics


.u

● Statement 3 is incorrect: Various initiatives Technology (C-MET) and Tata Steel Limited.
taken for genome sequencing are as follows: The state government would provide the basic
w

1. IndiGen Programme: To undertake infrastructure for the project. The Centre


w

whole genome sequencing of a would help attract investors to develop


thousand Indian individuals graphene products.
w

representing diverse ethnic groups

DPP 2023 DAY 160 34


https://upscmaterial.online/
Download From - https://upscmaterial.online/

Contact us :info@onlyias.com

OnlyIAS Nothing Else Visit :dpp.onlyias.in


Contact : +91-7007 931 912

● Statement 2 is incorrect: Earlier in 2020, the creating mechanisms for consent-based


researchers from the City University of Hong sharing of information and building systems for
Kong had produced a laser-induced form of securely storing data.
graphene masks that inactivate the
coronavirus species. An Innovation Center is a ● Statement 2 is incorrect: Aadhaar is not
cross-functional plan that creates a safe haven mandatory for generating the health ID, other
for new ideas. With opportunities for IDs such as PAN card, ration card, electors’
individual and group collaboration across time photo identity card along with Aadhaar card
zones and continents, it's a place that fosters a will be used for enrolment in the scheme.
culture of innovation through the creation, Each Health ID will be linked to a health data
sharing, and testing of ideas. consent manager, which will be used to seek
● Statement 3 is correct: Graphene is a one- the patient’s consent and allow for seamless
atom-thick layer of carbon atoms arranged in a flow of health information from the Personal
hexagonal lattice. It is the building-block of Health Records module.
Graphite, but graphene is a remarkable The Health ID will be voluntary and applicable
substance on its own with a multitude of across states, hospitals, diagnostic laboratories
astonishing properties. It is the thinnest, most and pharmacies.
electrically and thermally conductive material
in the world, while also being flexible, Reference:
transparent and incredibly strong. https://www.google.com/amp/s/theprint.in/health/si
gning-up-for-modi-govts-digital-health-mission-

e
Reference: voluntary-says-project-chief/556785/%3famp

n
https://indianexpress.com/article/india/kerala/first-

li
graphene-innovation-centre-kerala-pinarayi-vijayan-
7753377/ Q.31) Ans: D

n
Exp: o
● Blockchain technology is an innovation in the
l.
Q.30) Ans: B healthcare technology industry.
a

Exp:
ri

● Statement 1 and statement 3 are correct: 1. In the United States, the Food and Drug
Under the Digital Health ID card project, which Administration (FDA) began piloting a program
te

Prime Minister Narendra Modi announced in to support the United States Drug Supply Chain
his 74th Independence Day speech, the health Security Act (DSCSA). This was piloted to
a

ID will be a gateway to an individual’s medical address the requirements necessary to


m

records — doctor visits, diseases and drugs identify, track, and trace prescription
consumed among others. The personal health
c

medications and vaccine distribution.


records will be accessible through an app or a 2. It enables a user-centred means of gathering
s

website with ownership lying with the health information securely and ensuring that
p

individual. it can be verified unshared. Blockchain systems


.u

Under the programme, a patient can share provide a transparent and distributed ledger of
their records with any health information user records that cannot be altered without
w

such as doctors, hospitals or diagnostic labs. recording a change.


w

The pilot project includes the enrolment of 3. This technology is useful for providing robust
individuals, doctors and healthcare providers, safeguards for patient information as the
w

along with the digitisation of medical records, technology can anonymize patient data while

DPP 2023 DAY 160 35


https://upscmaterial.online/
Download From - https://upscmaterial.online/

Contact us :info@onlyias.com

OnlyIAS Nothing Else Visit :dpp.onlyias.in


Contact : +91-7007 931 912

providing a means of distributing information Users can schedule doctor appointments and
across diverse and fragmented healthcare consult with doctors from the comfort of their
systems. home. Users ePrescription will be readily
4. This pilot program demonstrated the utility of available once synced. Aarogyasetu helps you
blockchain in connecting disparate systems to schedule/reschedule/cancel your
and organizations to facilitate complete appointments for covid-19 vaccination. Using
product traceability. It also demonstrated Aarogyasetu you can download the
patient safety as it reduced the time taken to vaccination certificate or can request for
inform the supply chain of a product recall, changes in certificate as well. Your certificate
reducing it from a few days to a few seconds. will be readily available once synced.
Hence statement 3 is incorrect.
Reference:
https://www.google.com/amp/s/www.news- Reference:
medical.net/amp/health/Health-Technology-Around- https://www.aarogyasetu.gov.in
the-World.aspx
Q.33) Ans: C
Exp:
Q.32) Ans: B ● Precision Oncology is a field of medicine where
Exp: the genomic and protein expression signature
● Statement 1 is correct: Aarogyasetu has of a patient’s tumour tissue and the clinical and
transformed into a National Health App, pathological presentation is used for accurate

e
bringing a whole plethora of digital health diagnosis and plan treatment with maximum

n
services powered by Ayushman Bharat Digital benefits and least side effects. Precision

li
Mission (ABDM). oncology is a game-changer in cancer
Ayushman Bharat is a flagship scheme of India treatment, which certainly increases the

n
which was launched as recommended by the overall survival of the patient with the disease.
o
National Health Policy 2017, to achieve the ● Precision medicine: It considers the individual
l.
vision of Universal Health Coverage (UHC). variability in genetics, environment, and
a

Implementing Agency is the National Health lifestyle for each patient.


ri

Authority (NHA) under the Ministry of Health For example, when using precision medicine to
and Family Welfare. treat a patient with cancer, the medicine can
te

be tailored to them based on their unique


● Statement 2 is correct: Using Aarogya setu, genetic make-up.
a

users can now register for your Ayushman This personalized medicine is far more
m

Bharat Health Account (i.e Digital Health ID) effective than other types of treatment as it
c

and leverage it for your interaction with attacks tumors based on the patient’s genetics,
participating healthcare providers, and it also causing gene mutations and making it more
s

allows users to receive digital lab reports, easily destroyed by the cancer medication.
p

prescriptions and diagnosis seamlessly from


.u

verified healthcare professionals and health


service providers.
w
w

● Statement 3 is incorrect: Aarogya Setu helps Source: https://healthmatters.nyp.org/precision-


users to schedule online doctor appointments medicine/
w

powered by eSanjeevani OPD Application.

DPP 2023 DAY 160 36


https://upscmaterial.online/
Download From - https://upscmaterial.online/

Contact us :info@onlyias.com

OnlyIAS Nothing Else Visit :dpp.onlyias.in


Contact : +91-7007 931 912

Reference: Magnetocaloric effect was first theoretically


https://www.proclinical.com/blogs/2022-4/top-10- postulated in the works of Langevin in 1905. He
new-medical-technologies-2022 predicted that the temperature of a
paramagnetic material should change as a
function of the magnetic field.
Q.34) Ans: B The later called magnetocaloric effect was first
Exp: observed in an experiment by Weiss and Picard
● The term “smart pills” refers to miniature in the year 1917 where they could measure a
electronic devices that are formed and temperature change on Ni among applications
designed within the mildew of pharmaceutical of a magnetic field.
capsules however perform extremely
advanced functions such as sensing, imaging, ● Statement 2 is correct: Advancements in
and drug delivery. magnetic materials led to the development of
They will include biosensors or images, magnetic hyperthermia to try to address the
hydrogen ion concentration, or chemical issues of side effects of cancer treatment like
sensors. Once they’re swallowed, they travel chemotherapy. In magnetic hyperthermia,
along the gastrointestinal tract to capture magnetic nanoparticles are subjected to
information that is otherwise tough to get; alternating magnetic fields of few Gauss, which
then these pills are simply eliminated from the produce heat due to magnetic relaxation
system. Their classification as ingestible losses. Usually, the temperature required to
sensors makes them distinct from implantable kill the tumour cells is between 40 and 45°C.

e
or wearable sensors. It is an alternative for However, the drawback in magnetic

n
patients who refuse invasive colon exams, hyperthermia is the lack of control of

li
have haemorrhage or sedation risks or temperature, which may damage the healthy
inflammatory internal organ sickness, or have cells in the body and also have side effects like

n
had a previous incomplete endoscopy. Smart increased blood pressure etc. o
pills permit additional folks to induce screening These problems can be avoided by using
l.
for large intestine cancer with a minimally magnetocaloric materials, as it can provide
a

invasive, radiation-free choice. controlled heating. The advantage of


ri

magnetocaloric materials which heat up or


Reference: cool down with the application and removal of
te

https://www.sciencedirect.com/science/article/pii/B9 the magnetic field, respectively, is that as soon


780128195932000108 as the magnetic field is removed, a cooling
a

effect is generated, unlike in magnetic


m

nanoparticles where overheating persists,


c

Q.35) Ans: C even after removal of the magnetic field.


Exp:
s

● Statement 1 is correct: Certain materials heat Reference:


p

up, when brought into a magnetic field. This https://pib.gov.in/PressReleseDetail.aspx?PRID=1630


.u

phenomenon is called the magnetocaloric 035


effect (MCE). Materials endowing this effect
w

are called magnetocaloric materials (MCM).


w

Such materials include pure elements such as


Gadolinium or alloys such as Lanthanum-iron- Q.36) Ans: D
w

silicon (LaFeSi) or Iron-phosphorus (Fe2P). Exp:

DPP 2023 DAY 160 37


https://upscmaterial.online/
Download From - https://upscmaterial.online/

Contact us :info@onlyias.com

OnlyIAS Nothing Else Visit :dpp.onlyias.in


Contact : +91-7007 931 912

● Statement 1 is incorrect: Carbon capture and Similarly, in the US there are multiple large-
storage (CCS) or carbon capture and scale carbon sites such as the Citronelle Project
sequestration is the process of capturing in Alabama. This saline reservoir injection site
carbon dioxide (CO2) before it enters the is about 1.8 miles (2.9km) deep.
atmosphere, transporting it, and storing it The aim is to prevent the release of CO2 from
(carbon sequestration) for centuries or heavy industry with the intent of mitigating the
millennia. effects of climate change.
There are three steps to the CCS process:

1. Capturing the carbon dioxide for storage ● Statement 3 is incorrect: CO2 can be captured
The CO2 is separated from other gases directly from an industrial source, such as a
produced in industrial processes, such as those cement kiln, using a variety of technologies;
at coal and natural-gas-fired power generation including absorption, adsorption, chemical
plants or steel or cement factories. looping, membrane gas separation or gas
2. Transport hydration.
The CO2 is then compressed and transported
via pipelines, road transport or ships to a site Reference:
for storage. https://www.iea.org/fuels-and-technologies/carbon-
capture-utilisation-and-storage
3. Storage
Finally, the CO2 is injected into rock formations

e
deep underground for permanent storage. Q.37) Ans: A

n
Carbon sequestration is the process of Exp:

li
capturing, securing and storing carbon dioxide ● Statement 1 is incorrect: Green hydrogen is
from the atmosphere. The idea is to stabilize hydrogen produced by splitting water by

n
carbon in solid and dissolved forms so that it electrolysis. This produces only hydrogen and
o
doesn’t cause the atmosphere to warm. The oxygen. The hydrogen and vent the oxygen to
l.
process shows tremendous promise for the atmosphere with no negative impact.
a

reducing the human “carbon footprint.” However it is Blue hydrogen which is


ri

produced when natural gas is split into


● Statement 2 is correct: The CO2 is captured hydrogen and CO2 either by Steam Methane
te

from large point sources, such as a chemical Reforming (SMR) or AutoThermal Reforming
plant or biomass power plant, and then stored (ATR), but the CO2 is captured and then
a

in an underground geological formation which stored. As the greenhouse gases are captured,
m

typically need to be 0.62 miles (1km) or more this mitigates the environmental impacts on
c

under the ground. the planet.


As an example, a storage site for the proposed The ‘capturing’ is done through a process
s

Zero Carbon Humber project in the UK is a called Carbon Capture Usage and Storage
p

saline aquifer named ‘Endurance’, which is (CCUS).


.u

located in the southern North Sea, around


90km offshore. Endurance is approximately 1 ● Statement 2 is correct: To achieve the
w

mile (1.6km) below the seabed and has the electrolysis electricity and power is needed.
w

potential to store very large amounts of CO2. This process to make green hydrogen is
powered by renewable energy sources, such as
w

wind or solar. That makes green hydrogen the

DPP 2023 DAY 160 38


https://upscmaterial.online/
Download From - https://upscmaterial.online/

Contact us :info@onlyias.com

OnlyIAS Nothing Else Visit :dpp.onlyias.in


Contact : +91-7007 931 912

cleanest option – hydrogen from renewable


energy sources without CO2 as a by-product. If
the electricity is generated from renewable
sources such as solar or wind, production of
hydrogen in this way emits no greenhouse
gasses.
● different shades of hydrogen:
1. brown hydrogen is produced using coal where
the emissions are released to the air Source:
2. grey hydrogen is produced from natural gas https://www.petrofac.com/media/stories-and-
where the associated emissions are released to opinion/the-difference-between-green-hydrogen-and-
the air blue-hydrogen/
3. blue hydrogen is produced from natural gas, Reference:
where the emissions are captured using carbon https://pib.gov.in/PressReleasePage.aspx?PRID=1799
capture and storage 067
4. green hydrogen is produced from electrolysis https://www.petrofac.com/media/stories-and-
powered by renewable electricity. opinion/the-difference-between-green-hydrogen-and-
blue-hydrogen/
● Statement 3 is correct: Hon’ble Prime Minister
launched the National Hydrogen Mission last
year. The Mission aims to aid the government Q.38) Ans: B

e
in meeting its climate targets and making India Exp:

n
a green hydrogen hub. This will help in meeting ● Digital therapeutics and diagnostics are a set of

li
the target of production of 5 million tonnes of digital tools or systems for the treatment and
Green hydrogen by 2030 and the related prevention of disease. Typically, these are

n
development of renewable energy capacity. software, mobile, or connected solutions,
o
which fall into disease prevention apps,
l.
Green Hydrogen Mission's aims and objectives: connected medical devices, and remote
a

1. This Policy will provide clean fuel, reduce monitoring systems.


ri

India’s dependence on fossil fuel and also


reduce crude oil imports. Digital therapeutics and diagnostics include
te

2. Green Hydrogen Manufacturers with the components such as:


coordination of distribution companies can 1. Deep tech in human biology (including an
a

Store their unutilized renewable power for up understanding of how diseases occur and
m

to 30 days. progress).
c

Companies can get distribution licensees at 2. Genetic engineering and molecular medicine.
concessional Prices.
s

3. Ensuring ‘grid to market’ Connectivity to 3. ICT services such as cloud computing and 3D
p

attract Investors and ensure timely delivery. printing which enable seamless continuum of
.u

4. Single Portal for all business activities to ensure care and apps that help in tracking health
ease of doing business. parameters in real time.
w

Objective of this Policy: Increasing Production


w

to meet domestic demand and also to become 4. Predictive Analytics which make healthcare
an exporter of Green Hydrogen. decision-making more robust and allow for
w

doctor-patient interactions.

DPP 2023 DAY 160 39


https://upscmaterial.online/
Download From - https://upscmaterial.online/

Contact us :info@onlyias.com

OnlyIAS Nothing Else Visit :dpp.onlyias.in


Contact : +91-7007 931 912

● Cybersecurity has increasingly become a Agristack is concerned. However, leading


concern for Digital Therapeutics (DTx) Technology/Agri-tech/StartUp companies
developers and regulatory bodies. DTx are were identified and invited to collaborate with
vulnerable to security threats, and breaches in the Government of India to develop Proof of
security could cause harm up to and including Concepts (PoC) based on small portions of data
life-threatening situations. from the federated Farmers’ database for
certain identified areas (Districts/Village).

https://pib.gov.in/PressReleaseIframePage.as
px?PRID=1741995

● Statement 3 is correct: It aims to provide a


source: https://jelvix.com/blog/digital-therapeutics larger push to digitize data in India, ranging
from land titles to medical records. Each
5. Digital therapeutics and diagnostics transmits farmer will receive a unique digital
information over the Internet, so the risk of identification (farmers' ID) as part of the
unauthorized access and manipulation of scheme, which will include personal
these solutions, or their underlying data, could information, information about the land they
jeopardize product confidence and patient farm, as well as productivity and financial
care. information.
Hence statement 4 is incorrect. Each ID will be linked to the individual's

e
Aadhaar ID.

n
Reference

li
https://jelvix.com/blog/digital-therapeutics Reference:
https://www.financialexpress.com/lifestyle/health/in https://www.google.com/amp/s/krishijagran.com/agr

n
dias-journey-towards-a-digital-healthcare-ecosystem- ipedia/agristack-potential-benefits-and-concerns-of-
o
taking-the-leap-of-faith/2281853/ digital-agriculture/%3famp=1
l.
https://www.google.com/amp/s/www.downtoearth.o
a

rg.in/news/agriculture/amp/agristack-the-new-digital-
ri

Q.39) Ans: B push-in-agriculture-raises-serious-concerns-77613


Exp:
te

● Statement 1 is correct: AgriStack is a set of


technology and digital databases geared
a

toward farmers and the agriculture sector. Q.40) Ans: B


m

AgriStack will provide a uniform platform for Exp:


c

farmers to deliver end-to-end services ● A Silicon-Valley based firm called Motwani


throughout the agriculture food value chain. Jadeja Foundation organized an online
s

Under AgriStack’, the government aims to exclusive 72-hour hackathon in partnership


p

provide ‘required data sets’ of farmers’ with Bengaluru based HackerEarth and the
.u

personal information to Microsoft to develop a Association of Designers in India (ADI).


farmer interface for ‘smart and well-organized The hackathon called ‘CODE19’ aims to find
w

agriculture’. solutions to help India fight against the COVID-


w

19 (coronavirus) outbreak.
● Statement 2 is incorrect: No private sector
w

companies are involved as far as building of the

DPP 2023 DAY 160 40


https://upscmaterial.online/
Download From - https://upscmaterial.online/

Contact us :info@onlyias.com

OnlyIAS Nothing Else Visit :dpp.onlyias.in


Contact : +91-7007 931 912

The CODE19 hackathon has eight sets of online-hackathon-find-solution-to-covid19-


specific themes, which is what is being divided 6351971/lite/
into projects. The themes are as follows:
Q.41) Ans: C
Medical treatment and testing Exp:
1. Travel and Tourism ● Option 1 is not correct : Greenwashing is the
2. Mitigation – Isolation process of conveying a false impression or
3. protection/masks providing misleading information about how
4. social distancing in India a company’s products are more
5. Social life, welfare and awareness environmentally sound. Greenwashing is
6. Research and development considered an unsubstantiated claim to
7. Open innovation for COVID-19; Education and deceive consumers into believing that a
awareness. company’s products are environmentally
friendly.
● During the CODE19 hackathon, participants ● Option 2 is not correct : The pyrolysis process
will work on finding solutions to the problems is the thermal decomposition of materials at
listed above. This will be achieved by elevated temperatures in an inert
participants via ideation, wireframes, designs, atmosphere. It involves a change of chemical
developments and testing the prototype they composition. The word is coined from the
are creating. Greek-derived elements pyro "fire", "heat",
"fever" and lysis "separating".Pyrolysis is one

e
● The code name of the India - Pakistan kargil of the technologies available to convert

n
war was Operation Vijay. biomass to an intermediate liquid product
that can be refined to drop-in hydrocarbon

li
● The Ministry of Human Resource Development biofuels, oxygenated fuel additives and

n
(MHRD) now Ministry of Education has petrochemical replacements. Pyrolysis is the
o
released guidelines on digital education titled heating of an organic material, such as
l.
'PRAGYATA'. biomass, in the absence of oxygen. Biomass
a

The guidelines have been prepared by the pyrolysis is usually conducted at or above 500
ri

National Council of Educational Research and °C, providing enough heat to deconstruct the
Training (NCERT). These are only advisory in strong bio-polymers mentioned above.
te

nature and state governments can formulate ● Option 3 is correct : Biomining is a process by
their own rules, based on local needs. which garbage is treated with bio-organisms
a

The guidelines include eight steps of or natural elements like air and sunlight so
m

online/digital learning i.e. Plan- Review- that the biodegradable elements in the waste
break down over time. Civic authorities in
c

Arrange- Guide- Yak (talk)- Assign- Track-


Appreciate. charge of collecting waste, usually dump mixed
s

Source: waste biodegradable and non-biodegradable


p

https://pib.gov.in/PressReleasePage.aspx?PRI in the garbage dump sites over the years.


.u

D=1638541 Through bio-mining, this unsegregated


accumulated heap of waste is exposed to bio-
w

Reference: organisms and to air and sun so that the


w

https://www.google.com/amp/s/indianexpress.com/a biodegradable waste among the layers gets


rticle/technology/tech-news-technology/code19- decomposed through the natural process.
w

Whatever is left over will be the non-

DPP 2023 DAY 160 41


https://upscmaterial.online/
Download From - https://upscmaterial.online/

Contact us :info@onlyias.com

OnlyIAS Nothing Else Visit :dpp.onlyias.in


Contact : +91-7007 931 912

biodegradable material, which has to be dealt a country to be declared polio-free, the wild
with separately. transmission of all three kinds has to be
● Option 4 is not correct : Biosparging is the stopped. For eradication, cases of both wild
process of pumping pressurized air or gas into and vaccine-derived polio infection have to be
a polluted area to stimulate in-situ aerobic reduced to zero. Subsequently, WHO removes
biological activity.This technology targets the country from the list of countries with
chemical substances such as mineral oils and active wild poliovirus.
benzene, toluene, ethylbenzene, xylene, and ● Statement 3 is not correct : Injectable Polio
naphthalene (BTEXN) that can be biodegraded Vaccine is made up of heat-killed virus that
under aerobic conditions and is used to treat cannot cause the disease in any case, because
soluble and residual contaminants in the the pathogen is not alive. However, it does
saturated zone.By giving oxygen to the produce the memory in the cells, required for
microorganisms and increasing the immunity. Oral Polio Vaccine is made up of
interactions between air, water, and the live-attenuated virus, which is nearly
aquifer, the injection of air (and gaseous incapable of producing an infection. This type
nutrients if needed) promotes the of vaccination helps in providing immunity to
development of the aerobic microbial wild-type of virus.
population and thereby enhances the
bioavailability of pollutants. Reference :
https://indianexpress.com/article/lifestyle/health-
Reference : specials/india-doesnt-worry-polio-case-new-york-

e
https://www.thehindu.com/news/cities/Mangalore/b 8088436/

n
io-mining-to-clear-legacy-waste-dumped-at-

li
pachchanady-to-begin-soon/article65363566.ece
Q.43) Ans: C

n
Q.42) Ans: C Exp: o
Exp: ● Statement 1 is not correct : Gene/genome
l.
● Statement 1 is not correct : Polio is a highly editing refers to technology that permits to
a

infectious viral disease, which mainly affects change an organism’s DNA. These
ri

young children. The virus is transmitted by technologies allow genetic material to be


person-to-person spread mainly through the added, removed, or altered at particular
te

faecal-oral route or, less frequently, by a locations in the genome. Its applications
common vehicle (e.g. contaminated water or include correcting genetic defects, treating and
a

food) and multiplies in the intestine, from preventing the spread of diseases and
m

where it can invade the nervous system and improving crops etc. A decade ago, scientists in
c

can cause paralysis. Polio is a highly infectious Germany and the US discovered a technique
disease, caused by the virus which multiplies in which allowed them to ‘cut’ DNA strands and
s

the intestine, from where it can invade the edit genes. For agriculture scientists this
p

nervous system and can cause paralysis. Once process allowed them to bring about desired
.u

that happens, the patient is crippled for life changes in the genome by using site directed
because there is no treatment for the nuclease (SDN) or sequence specific nuclease
w

affliction. Colloquially they are referred to as (SSN).


w

wild polio virus. ● Statement 2 is not correct : Advanced research


● Statement 2 is correct : There are three has allowed scientists to develop the highly
w

variants of the poliovirus, numbered 1 to 3. For effective clustered regularly interspaced

DPP 2023 DAY 160 42


https://upscmaterial.online/
Download From - https://upscmaterial.online/

Contact us :info@onlyias.com

OnlyIAS Nothing Else Visit :dpp.onlyias.in


Contact : +91-7007 931 912

palindromic repeat (CRISPR) -associated hydrogen that does enter the atmosphere is
proteins based systems. This system allows quickly ejected into space by the Earth's
for targeted intervention at the genome gravity. The majority of hydrogen is created by
sequence. This tool has opened up various combining natural gas and steam to form
possibilities in plant breeding. Using this tool, syngas (a mixture of hydrogen and carbon
agricultural scientists can now edit the genome monoxide). The hydrogen is separated from
to insert specific traits in the gene the syngas. The electrolysis of water can also
sequence.For any type of gene therapy to produce hydrogen.
work, genetic material first needs to get inside ● Statement 2 is correct: The Ministry of Power
the cells of the person living with the disease. notified the green hydrogen policy to
This can happen in 1 of 2 ways: Genetic accelerate the shift from polluting fuels
material can be delivered to target cells that towards local production and consumption of
remain inside a person’s body , this is called in green fuels. It will help in meeting the target of
vivo ; Genetic material can be delivered to a production of 5 million tonnes of Green
person’s cells after they are taken out of their hydrogen by 2030 and the related
body , this is called ex vivo. CRISPR/Cas9- development of renewable energy capacity.
based disease therapeutics have been The Ministry of Power in a statement said the
achieved both ex vivo and in vivo. In ex vivo implementation of the policy will provide clean
therapy, cells are isolated and edited outside fuel to the common people of the country,
of the body, after which they are transplanted reduce the dependence on fossil fuel and also
back into the body. In in vivo therapy, genetic reduce crude oil imports.Under the policy,

e
materials are directly injected into the body. Green Hydrogen, Ammonia manufacturers

n
may purchase renewable power from the

li
Reference : https://www.hindustantimes.com/india- power exchange or set up renewable energy
news/rules-relaxed-for-some-gene-edited-plants- capacity themselves or through any other,

n
organisms-101648665945313.html developer, anywhere. Open access will be
o
granted by the government within 15 days.
l.
Q.44) Ans: B
a

Exp: Reference :
ri

● Statement 1 is not correct : Although https://www.timesnownews.com/business-


hydrogen is the lightest and most abundant economy/govt-notifies-green-hydrogen-policy-in-
te

element in the universe, it is rarely found in push-towards-national-green-mission-article-


nature in its elemental form and always must 89652951
a

be extracted from other hydrogen-containing https://www.sciencedirect.com/topics/chemistry/ele


m

compounds. Elemental hydrogen does not mental-hydrogen


c

occur in the earth's atmosphere to any


significant extent owing to the low molecular
s

mass of the molecules. The element hydrogen


p

is by far the most abundant in the universe. It Q.45) Ans: B


.u

can be found in the sun and most stars, and it Exp:


makes up the majority of Jupiter's mass. ● Pair 1 is not correctly matched : NASA’s
w

Hydrogen is found in the most abundant form Artemis mission is touted as the next
w

on Earth as water. It is only found in trace generation of lunar exploration, and is named
amounts in the atmosphere as a gas – less after the twin sister of Apollo from Greek
w

than 1 part per million by volume. Any mythology. Artemis is also the goddess of the

DPP 2023 DAY 160 43


https://upscmaterial.online/
Download From - https://upscmaterial.online/

Contact us :info@onlyias.com

OnlyIAS Nothing Else Visit :dpp.onlyias.in


Contact : +91-7007 931 912

moon. NASA's Artemis 1 moon mission will be planetary defense against near-Earth objects
the agency's first big step toward returning (NEOs) . The mission will deliberately crash a
astronauts to the lunar surface. Artemis 1 will space probe into the minor-planet moon
be the first test flight of the agency's new Dimorphos of the double asteroid Didymos to
Space Launch System megarocket and the assess the future potential of a spacecraft
Orion crew capsule. The SLS rocket will launch impact to deflect an asteroid on a collision
the uncrewed Orion spacecraft on an course with Earth through a transference of
approximately 42-day mission, during which it momentum. The asteroid poses no actual
will orbit the moon before returning to Earth. threat to Earth; it was merely selected for the
● Pair 2 is correctly matched : NASA has test.
launched CAPSTONE, a microwave oven-sized
CubeSat weighing just 55 pounds (25 kg). Reference :
CAPSTONE, short for Cislunar Autonomous https://www.indiatoday.in/science/story/lucy-
Positioning System Technology Operations and mission-asteroid-moon-nasa-solar-system-formation-
Navigation Experiment, is designed to test a earth-history-1988987-2022-08-17
unique, elliptical lunar orbit. It aims to help https://dw.com/p/4Fz2c?maca=en-vas-RSS-en-teaser-
reduce risk for future spacecraft by validating -Sci_Tech-IE-24417-html-copypaste
innovative navigation technologies, and by
verifying the dynamics of the halo-shaped
orbit. Its launch It is heading toward an orbit Q.46) Ans: A
intended in the future for Gateway, a Moon- Exp:

e
orbiting outpost that is part of NASA’s Artemis ● The Breakthrough Agenda commits countries

n
program. The orbit is known as a near- to work together to make clean technologies

li
rectilinear halo orbit (NRHO). It is significantly and sustainable solutions the most affordable,
elongated, and is located at a precise balance accessible and attractive option in each

n
point in the gravities of Earth and the Moon. emitting sector (power, road transport, steel,
o
This offers stability for long-term missions like agriculture etc.) globally before 2030.Around
l.
Gateway, NASA said on its website. 35 countries have signed up this agenda
a

● Pair 3 is correctly matched : Lucy will be the including the US, India, EU, developing
ri

first space mission to study the Trojans. The economies and some of those most vulnerable
mission takes its name from the fossilized to climate change.
te

human ancestor (called “Lucy” by her ● Global Methane Pledge is a voluntary non-
discoverers) whose skeleton provided unique binding agreement under which signatory
a

insight into humanity's evolution. Likewise, the countries have promised to cut their methane
m

Lucy mission will revolutionize our knowledge emissions by at least 30 per cent by 2030.India
c

of planetary origins and the formation of the has not signed up for the Global Methane
solar system.Lucy is a NASA space probe on a Pledge. The Pledge was proposed by the
s

twelve-year journey to eight different European Union and the United States of
p

asteroids, visiting a main belt asteroid as well America targeting a 30% reduction in global
.u

as seven Jupiter trojans, asteroids which share methane emissions from 2020 levels by 2030.
Jupiter's orbit around the Sun, orbiting either ● Declaration on “accelerating the transition to
w

ahead of or behind the planet. 100% zero-emission cars and vans” aims to
w

● Pair 4 is not correctly matched : Double work towards all sales of new cars and vans
Asteroid Redirection Test (DART) is a NASA being zero emission globally by 2040, and by
w

space mission aimed at testing a method of no later than 2035 in leading markets.The non-

DPP 2023 DAY 160 44


https://upscmaterial.online/
Download From - https://upscmaterial.online/

Contact us :info@onlyias.com

OnlyIAS Nothing Else Visit :dpp.onlyias.in


Contact : +91-7007 931 912

binding agreement has been signed by address loss and damage. The WIM has been
governments, automotive manufacturers, mandated to share knowledge, strengthen
financial institutions, and civil society dialogues among stakeholders and mobilise
organisations.India has set a target to expertise to enhance action and support to
transition 30% of passenger cars and 70% of address loss and damage. But neither the WIM
commercial vehicles to electric vehicles by nor any other established mechanism delivers
2030. funding to help countries manage loss and
● Glasgow Leaders’ Declaration on Forests and damage.Loss and Damage is mentioned in Article
Land Use: The declaration commits the 8 of the Paris Agreement. However, there is no
countries to halt and reverse deforestation and reference to provision of finance for climate
land degradation by 2030.It stresses the need reparation/compensation in this Article.At
for transformative steps to move the world COP26 in Glasgow, a large coalition of climate-
onto a sustainable and resilient land-use vulnerable countries advocated for creating a
path.India is not a part of this declaration. new finance facility or fund dedicated to loss and
damage. The proposal for loss and damage
Reference: financing was once again rejected by developed
https://indianexpress.com/article/explained/global- countries. A 2-year Glasgow Dialogue was
methane-pledge-explained-7605172/ established at the COP26. It will discuss possible
https://indianexpress.com/article/india/india-to- arrangements for loss and damage funding. It
work-towards-zero-emission-cars-by-2040-7617406/ was also agreed to operationalise and fund the
https://www.thehindu.com/sci-tech/energy-and- Santiago Network on Loss and Damage (SNLD).

e
environment/declaration-on-forests-and-land- ● Statement 3 is correct: The Glasgow agreement

n
use/article37573916.ece will fund the Santiago Network, which is

li
supposed to build technical expertise about
climate adaptation in vulnerable nations. (The

n
Q.47) Ans: A Santiago Network is a network to enable loss
o
Exp: and damage fund flow from developed to
l.
● Statement 1 is incorrect:There is no official developing countries. It was created at the
a

definition of ‘Loss and damage‘. Loss generally Madrid COP in 2019 ).Loss and damage refers to
ri

refers to the complete forfeiture of items like the costs that some countries are already facing
land, ecosystems, or of human lives, while from climate change, and these countries have
te

damage refers to the harm to infrastructure and for years wanted payment to help deal with it.It
property that could be repaired. The term will work towards the implementation of
a

includes both economic and non-economic relevant approaches at the local, national and
m

losses.A close definition is found in the literature regional level, in developing countries that are
c

review commissioned by the UNFCCC. Loss is the particularly vulnerable to the adverse effects of
negative impacts in relation to which reparation climate change.
s

or restoration is impossible while Damage is the


p

negative impacts in relation to which reparation Reference:


.u

or restoration is possible. https://www.downtoearth.org.in/news/climate-


● Statement 2 is incorrect: Loss and damage first change/cop26-not-on-official-agenda-but-loss-
w

appeared in the negotiated outcome of the UN damage-comeback-gives-hope-say-experts-80467


w

climate talks in 2007 as part of the Bali Action


Plan. In 2013, Warsaw International Mechanism
w

(WIM) was formed to avert, minimise and Q.48) Ans: D

DPP 2023 DAY 160 45


https://upscmaterial.online/
Download From - https://upscmaterial.online/

Contact us :info@onlyias.com

OnlyIAS Nothing Else Visit :dpp.onlyias.in


Contact : +91-7007 931 912

Exp: kind contributions from the insurance sector and


● Statement 1 is incorrect: A global coalition of 10 partner institutions.
organisations launched the Global Resilience
Index Initiative (GRII) November 9, 2021 to build Reference: https://www.cdri.world/press-
a universal model for assessing resilience to releases/global-resilience-index-initiative-launched
climate risks. It was launched during the UNFCCC https://www.downtoearth.org.in/news/climate-
Conference of Parties (CoP) 26, Glasgow.The change/towards-better-adaptation-global-model-to-
curated, open-source resource can be used in assess-resilience-to-climate-risks-launched-80116
aggregated risk management across sectors and
geographies.The GRII draws upon significant
cross-sector risk modelling experience, including Q.49) Ans: D
public-private partnerships between Exp:
governments, academia, insurance and ● The Net-Zero Banking Alliance(NZBA) is
engineering. It will help in mobilising the trillions convened by the United Nations Environment
of investment needed to meet the Paris goals Programme Finance Initiative (UNEP FI) and is
on climate-resilient development. the banking element of the Glasgow Financial
● Statement 2 is incorrect: The mission of the GRII Alliance for Net Zero (GFANZ), chaired by UN
is to address the data emergency that is Special Envoy on Climate Action and Finance.
contributing to the climate crisis by helping Representing almost 40% of global banking
sectors across the global economy quantify the assets, with over 100 members from 40
value of building climate resilience and the costs countries, the Alliance is a critical step in the

e
of doing nothing. It will enable asset owners to mobilisation of the financial sector for climate. It

n
compare portfolio risks across geographies and recognizes the vital role of banks in supporting

li
hazards, as well as helping countries to prioritise the global transition of the real economy to a
national adaptation investments. low-carbon, sustainable, and inclusive

n
The coalition behind the GRII is seeking to economy.By joining the NZBA, members commit
o
achieve two initial goals: to reducing emissions attributable to their
l.
1. Offer global open reference risk data using operations and—much more significantly—to
a

metrics built on insurance risk modelling their lending and investment portfolios to net
ri

principles; zero by 2050.The Net-Zero Banking Alliance


2. Provide shared standards and facilities (NZBA), hosted by United Nations Environment
te

applicable to a wide range of uses, including Programme-Finance Initiative (UNEP FI) is the
corporate climate risk disclosure, national newest net zero alliance.The Alliance reinforces,
a

adaptation planning and reporting, and the accelerates and supports the implementation of
m

planning of pre-arranged humanitarian decarbonisation strategies, providing an


c

finance. internationally coherent framework and


● GRII was launched during COP26 by 10 global guidelines in which to operate, supported by
s

organisations including United Nations office peer-learning from pioneering banks. It


p

for Disaster Risk Reduction (UNDRR), Insurance recognises the vital role of banks in supporting
.u

Development Forum (IDF), Coalition for the global transition of the real economy to net-
Disaster Resilient Infrastructure (CDRI) etc.The zero emissions.
w

outcomes of this risk analysis will help close the


w

insurance protection gap and direct investment Reference: https://www.unepfi.org/net-zero-


and aid to where they are needed the most. GRII banking/
w

has been initiated with partial funding and in-

DPP 2023 DAY 160 46


https://upscmaterial.online/
Download From - https://upscmaterial.online/

Contact us :info@onlyias.com

OnlyIAS Nothing Else Visit :dpp.onlyias.in


Contact : +91-7007 931 912

https://www.americanprogress.org/article/the-net- https://indianexpress.com/article/cities/delhi/air-
zero-banking-alliance/ panel-directs-work-from-home-schools-and-colleges-
shut-in-delhi-ncr-7626384/

Q.50) Ans: A
Exp: Q.51) Ans: A
● Statement 1 is correct: The Commission for Air Exp:
Quality Management (CAQM) was formed by an ● Statement 1 is incorrect:Published annually
ordinance, “Commission for Air Quality since 2005, the Climate Change Performance
Management (CAQM) in National Capital Region Index (CCPI) is an independent monitoring tool
and Adjoining Areas Ordinance 2020”, in for tracking the climate protection performance
October 2020. It has given the central pollution of 60 countries and the EU in four categories —
control board powers to operationalise GHG emissions, renewable energy, energy use
measures under the Graded Response Action and climate policy. It aims to enhance
Plan (GRAP) on air pollution.The GRAP is a set of transparency in international climate politics
emergency measures to be implemented to and enables comparison of climate protection
control air pollution depending upon the air efforts and progress made by individual
quality. countries.Germanwatch, the NewClimate
● Statement 2 is correct: The Commission will Institute and the Climate Action Network publish
supersede bodies such as the central and state the index annually.
pollution control boards of Delhi, Punjab, ● Statement 2 is correct: India Ranked 10 in the

e
Haryana, UP and Rajasthan. It will have the Climate Change Performance Index 2022

n
powers to issue directions to these state Report The country still benefits from its

li
governments on issues pertaining to air relatively low per capita emissions. Denmark is
pollution. The Commission will have the power the highest ranked country (top three places

n
to impose a fine of up to Rs 1 crore and remained empty).In the overall ranking,
o
imprisonment of up to 5 years in case its Australia, Kazakhstan, Russia, Saudi Arabia and
l.
directions are contravened. South Korea are among the worst performers.
a

● Statement 3 is incorrect: CAQM will be a The report finds that India is already on track to
ri

permanent body and will have over 20 members meet its 2030 emissions target -which is
and to be chaired by a government official of the compatible with a well below the 2°C scenario-
te

rank of Secretary or Chief Secretary. It has close to achieving its Nationally Determined
exclusive jurisdiction over the NCR, including Contribution (NDC) target of a 40% share for
a

areas in Haryana, Punjab, Uttar Pradesh and non-fossil fuel installed power capacity by 2030,
m

Rajasthan, in matters of air pollution, and will be and on course for a targeted 33 to 35% reduction
c

working along with CPCB and ISRO, apart from in energy intensity by the same year. According
the respective state governments. It is also a to the report, India should set a net-zero target
s

statutory authority. for 2050 and leverage its domestic success on


p

renewables and emissions intensity into


.u

Reference: international initiatives.


https://www.hindustantimes.com/cities/delhi-
w

news/worst-nov-for-delhi-s-air-11-severe-pollution- Reference:
w

days-101638048719960.html https://newclimate.org/resources/publications/the-
climate-change-performance-index-2022
w

https://www.germanwatch.org/en/21110

DPP 2023 DAY 160 47


https://upscmaterial.online/
Download From - https://upscmaterial.online/

Contact us :info@onlyias.com

OnlyIAS Nothing Else Visit :dpp.onlyias.in


Contact : +91-7007 931 912

Q.52) Ans: B Reference: https://theprint.in/health/why-november-


Exp: has-brought-back-delhi-pollution-with-a-vengeance-
● Potential reasons for rise in pollution in after-cleaner-october/763972/
winter:
• Temperature inversion: Low surface
temperature causes lowering of temperature Q.53) Ans: A
inversion layer which leads to entrapment of Exp:
pollutants at lower levels of atmosphere. The ● Statement 1 is correct: A coalition of 22
condition, opposite to normal vertical countries have agreed to create zero emissions
distribution of temperature, is known as shipping trade routes between ports to speed up
Temperature Inversion. As the winter season the decarbonisation of the global maritime
sets in, dust particles and pollutants in the air industry. The signatory countries signed the
become unable to move. Due to stagnant 'Clydebank Declaration for Green Shipping
winds, these pollutants get locked in the air Corridors' (launched at the COP26 climate
and affect weather conditions, resulting in summit in Glasgow). It aimed at establishing
smog. green shipping corridors – zero-emission
• Denser wind: Cold denser winds have low maritime routes between 2 (or more) ports.
speed which reduces the dispersion rate of ● Statement 2 is incorrect: As part of the
pollutants creating visibility challenges. Due to declaration, the signatory countries will support
its inland location much more efforts are the establishment of at least six green shipping
required to clean the air of the region as corridors by 2050. A green corridor is defined as

e
compared to other metropolitan cities with a shipping route between two major port hubs

n
similar emission levels. on which the technological, economic and

li
• Pre-Harvest season and stubble burning: regulatory feasibility of zero-emissions ships is
Pre-Harvest season coincides with Delhi accelerated by public and private action.

n
Winter As the new harvest season starts, ● Statement 3 is incorrect: India has not yet
o
farmers set fire to leftover rice stalks and signed this declaration.The strategy for going
l.
straw after harvest for clearing out fields carbon neutral will include using ships that run
a

resulting in a dense layer of smog over the on zero-carbon fuels and updating port
ri

Northern Plains, including Delhi NCR. infrastructure. According to the World Economic
• Bursting of Firecrackers: The coinciding of Forum, International shipping which transports
te

bursting of firecrackers on Diwali with Delhi’s 90% of world trade accounts for about 3% of
winter also impacts the air quality in the short global carbon emissions.Hence, without efforts
a

term.The transport sector is the main source to decarbonize, the sector’s emissions could
m

of PM2.5 emissions and also makes up 80 increase by 50% by 2050 based on the projected
c

percent of nitrogen oxides and carbon expansion of seaborne trade.


monoxide in Delhi’s air.
s

• Burning of waste for warmth: Chlorides in Reference:


p

the air of Delhi indicates incineration of https://www.scotsman.com/news/environment/cop


.u

municipal waste and burning of plastics and 26-clydebank-declaration-launched-to-pioneer-six-


rubber tyres for warmth on the streets. As per zero-emission-shipping-routes-3452608
w

a Study by IIT Kanpur, controlling the burning https://www.gov.uk/government/publications/cop-


w

of garbage in the open can reduce pollutants in 26-clydebank-declaration-for-green-shipping-


Delhi’s air by up to 10%. corridors/cop-26-clydebank-declaration-for-green-
w

shipping-corridors

DPP 2023 DAY 160 48


https://upscmaterial.online/
Download From - https://upscmaterial.online/

Contact us :info@onlyias.com

OnlyIAS Nothing Else Visit :dpp.onlyias.in


Contact : +91-7007 931 912

Q.54) Ans: B ● Statement 1 is incorrect: Recently, the Centre


Exp: has set in motion the process of creating the
● Pakke Tiger Reserve, also known as Pakhui exclusive body- National Interlinking of Rivers
Tiger Reserve.It is a Project Tiger reserve in the Authority (NIRA)- to implement river-linking
Pakke Kessang district of Arunachal Pradesh in projects in India.NIRA is an independent
northeastern India.The reserve is protected by autonomous body for planning, investigation,
the Department of Environment and Forest of financing and the implementation of the river
Arunachal Pradesh.This Tiger Reserve has won interlinking projects in the country. NIRA will be
India Biodiversity Award 2016 in the category headed by a Government of India Secretary-rank
of ‘Conservation of threatened species’ for its officer. It will replace the existing National
Hornbill Nest Adoption Programme.Pakke Water Development Agency (NWDA) and will
Wildlife Sanctuary lies in the undulating and function as an umbrella body for all river linking
hilly foothills of the Eastern Himalayas in projects.
Arunachal Pradesh.It is bounded by Bhareli or ● Statement 2 is correct: NIRA will coordinate
Kameng River in the west and north, and by with neighbouring countries and concerned
Pakke River in the east.It is home to over 2000 states and departments as directed by the
species of plants, 300 species of birds, 40 Ministry of Jal Shakti or the Ministry of External
species of mammals, 30 species of amphibians Affairs. Recently, the Cabinet approved the
and 36 species of reptiles. Many species of the funding and implementation of Ken-Betwa RLP.
flora and fauna are globally threatened, and It's The first project under the National
PTR is one of the last remaining strongholds Perspective Plan for interlinking of rivers. Ken-

e
left for these species.It is known for its amazing Betwa RLP transfers water from Ken to Betwa

n
sightings of four resident hornbill River through the construction of Daudhan Dam

li
species.Recently Arunachal Pradesh Cabinet and a canal linking the two rivers. Both these
adopted the Pakke Tiger Reserve 2047 rivers are tributaries of river Yamuna. It will carry

n
declaration on climate change-resilient and water from water surplus areas to drought prone
o
responsive Arunachal Pradesh envisages a and water deficit areas in Bundelkhand Region.
l.
low emission and climate-resilient ● Statement 3 is correct: NIRA has powers on
a

development through five broad themes: issues related to environment, wildlife and
ri

Environment, forest and climate change; forest clearances under river linking projects
health and well-being of all; sustainable and and their legal aspects and has the power to
te

adaptive living; livelihoods and opportunities raise funds and act as a repository of borrowed
and evidence generation and collaborative funds or money received on deposit or loan
a

action. given on interest. It has power to set up a Special


m

Purpose Vehicle (SPV) for individual link


c

Reference: https://indianexpress.com/article/north- projects. As of now, there are six Interlinking of


east-india/assam/arunachal-pradesh-cabinet-adopts- Rivers projects: Ken- Betwa, Damanganga-
s

declaration-on-climate-change-7621512/ Pinjal, Par- Tapi -Narmada, Manas- Sankosh -


p

https://theprint.in/india/arunachal-to-be-net-zero- TeestaGanga, Mahanadi-Godavari, and


.u

carbon-emission-by-2047-chief-secretary/827433/ Godavari-Cauvery (Grand Anicut).


w

Reference:
w

Q.55) Ans: B https://indianexpress.com/article/india/centre-


Exp: river-linking-india-nira-nwda-gajendra-shekhawat-
w

7622936/

DPP 2023 DAY 160 49


https://upscmaterial.online/
Download From - https://upscmaterial.online/

Contact us :info@onlyias.com

OnlyIAS Nothing Else Visit :dpp.onlyias.in


Contact : +91-7007 931 912

https://theprint.in/india/river-linking-nwda-asks- their liveability to attract external economic


states-to-spell-out-expectations-from-centre-on- investments.
future-course/807865/
Reference:
https://pib.gov.in/PressReleasePage.aspx?PRID=177
Q.56) Ans: A 5142
Exp: https://indiaaheadnews.com/india/union-minister-
● Statement 1 is correct: Recently, the Ministry of gajendra-singh-shekhawat-launches-river-cities-
Jal Shakti along with the Ministry of Housing and alliance-184065/
Urban Affairs has launched the River Cities
Alliance (RCA).The primary objective of RCA is to
provide the member cities with a platform to Q.57) Ans: B
discuss and exchange information on aspects Exp:
that are vital for sustainable management of ● E-Amrit’, a web portal on electric vehicles (EVs)
urban rivers such as minimizing their water was launched by India at the COP26 Summit in
footprint, reducing impacts on river and water Glasgow, UK. It is a one-stop destination for all
bodies, capitalizing on natural, intangible, information on electric vehicles—busting myths
architectural heritage and associated services around the adoption of EVs and complement
and develop self-sufficient, self-sustainable initiatives of the government on raising
water resources through recycle, reuse awareness on EVs. The portal has been
strategy.It will focus on three broad themes- developed and hosted by NITI Aayog under a

e
Networking, Capacity Building and Technical collaborative knowledge exchange programme

n
Support. State Governments are implementing with the UK government and as part of the UK–

li
Action Plans for restoration of water quality of India Joint Roadmap 2030. E-Amrit is a one-stop
the identified polluted river stretches.The destination for all information on electric

n
National Mission for Clean Ganga (NMCG) and vehicles—busting myths around the adoption of
o
National Institute for Urban Affairs (NIUA) have EVs, their purchase, investment opportunities,
l.
collaborated together to launch the River Cities policies, subsidies, etc.In the recent past, India
a

Alliance (RCA). has taken many initiatives to accelerate the


ri

● Statement 2 is incorrect: RCA includes cities decarbonization of transport and adoption of


from both Ganga basin and Non-Ganga basin electric mobility in the country. Schemes such as
te

states. The alliance has been launched initially FAME(Faster Adoption and Manufacturing of
with 30 cities namely Dehradun, Rishikesh, (Hybrid &) Electric Vehicles) and PLI(Production
a

Haridwar, Srinagar, Varanasi, Kanpur, Prayagraj, Linked Incentive) are especially important in
m

Farrukhabad, Mirzapur, Mathura, Bijnor, creating an ecosystem for the early adoption of
c

Ayodhya, Patna, Bhagalpur, Begusarai, Munger, EVs.


Sahibganj, Rajmahal, Howrah, Jangipur, Hugli-
s

Chinsurah, Behrampore, Maheshtala, Reference:


p

Aurangabad, Chennai, Bhubaneshwar, https://www.livemint.com/news/india/india-


.u

Hyderabad, Pune, Udaipur and Vijayawada.The launches-e-amrit-portal-on-evs-at-cop26-details-


Alliance is open to all river cities of India. Any here-11636593705433.html
w

river city can join the Alliance at any time.RCA https://www.indiascienceandtechnology.gov.in/stihi


w

gives opportunities to these cities to strengthen ghlights/e-amrit-portal-electric-mobility-india


governance aspects for river cities and improves
w

DPP 2023 DAY 160 50


https://upscmaterial.online/
Download From - https://upscmaterial.online/

Contact us :info@onlyias.com

OnlyIAS Nothing Else Visit :dpp.onlyias.in


Contact : +91-7007 931 912

Q.58) Ans: C Q.59) Ans: D


Exp: Exp:
● Flash drought is characterized by a period of ● Statement 1 is incorrect: Coal gasification is the
rapid drought intensification with impacts on process of converting coal into synthesis gas
agriculture, water resources, ecosystems, and (also called syngas), which is a mixture of
the human environment.It is set in motion by hydrogen (H2), carbon monoxide (CO) and
lower-than normal rates of precipitation, carbon dioxide (CO2). The syngas technology
abnormally high temperatures, winds, and allows conversion of non-mineable coal/lignite
radiation. Together, these changes in weather into combustible gases through in situ
can rapidly alter the local climate. When gasification of the material. In the past, a
precipitation deficit occurs over an extended number of efforts have been made to gasify coal
period of time (e.g., several weeks), soil in India. These efforts started in the 1960s and
moisture is depleted by evapo-transpiration are continuing even now with varying
which desiccates land surface. capacities/scales. Syn-Gas produced from Coal
● Persistent atmospheric conditions can amplify gasification can be used in producing Synthetic
evaporative demand (extent to which the Natural Gas (SNG), energy fuel (methanol &
environment ‘tries’ to evaporate water) at the ethanol), ammonia for fertilizers and petro-
surface via increased solar insolation and chemicals. Coal gasification is considered a
global warming. cleaner option compared to burning of coal.
● Cyclones are violent Storms with an intense ● Statement 2 is incorrect: In order to take ahead
spiral and accompanied by strong winds and the Vision of 100 MT coal Gasification by 2030,

e
heavy rains. Anticyclones have a dual impact Ministry of Coal has chalked out an

n
on flash drought development: (1) They implementation strategy which include:

li
suppress rainfall which limits soil moisture o Mapping of gasification potential of
replenishment. (2) Less cloud coverage and coalfields especially in the North east.

n
high surface temperatures increase o Development of indigenous technology
o
evaporative demand of moisture. suitable for various feedstock (low ash coal,
l.
● Tropics and SubTropics have high potential for coal mixed with pet coke and high ash coal).
a

drought development due to high precipitation o Development of suitable business models for
ri

variability and evaporative demand. setting up various projects.


● Flash drought development is more likely to o Marketing strategy for end products.
te

occur in the May-June (period associated with o Policy support with a view to encourage
increasing rainfall) if onset of ITCZ-induced Atmanirbhar Bharat Scheme.
a

rainfall is delayed/reduced, in combination o Coordination with various stake holding


m

with increased evaporative demand. Ministries.


c

o Providing quantifiable targets to various


Reference: companies and monitoring the
s

https://www.downtoearth.org.in/news/climate- implementation of activities.


p

change/study-identifies-india-among-global-flash- ● Statement 3 is incorrect: Recently, a blueprint


.u

drought-hotspots-from-1980-2015- for the ‘National Coal Gasification Mission’


80131#:~:text=Published%3A%20Wednesday%2010 prepared by the Union Coal Ministry. The
w

%20November%202021,and%20September%20from government aims to achieve gasification of 100


w

%201980%2D2015. Million Tonnes (MT) of coal by 2030 and this is


https://indianexpress.com/article/technology/scienc the first time that a mission document has been
w

e/flash-droughts-affect-india-7639227/ released. It is implemented by the Ministry of

DPP 2023 DAY 160 51


https://upscmaterial.online/
Download From - https://upscmaterial.online/

Contact us :info@onlyias.com

OnlyIAS Nothing Else Visit :dpp.onlyias.in


Contact : +91-7007 931 912

Coal.The gasification technology will also help through the dam-canal network and
India overcome the shortage of oil, gas, groundwater pumping.
methanol, ammonia, urea and other products, ➢ Navigation: The large canals linking the
making the country Atma Nirbhar. These rivers are expected to facilitate inland
products will help in reduction in crude oil navigation.
import. India currently imports 50 to 70 lakh ➢ Transportation: Boost inland
tonnes of urea every year, and it would help waterway transportation in India and
increase the availability of domestically thus transportation of raw material
produced fertilisers.Most of India’s known coal and products will be faster and cost
deposits are non-recoverable. Underground coal effective.
gasification could help extract those plentiful Environmental:
reserves that are deep, scattered and covered by ➢ Tackle flood and drought: The
forests. problems of floods and droughts
would be tackled nationwide by
Reference: transferring the river water from water
https://www.financialexpress.com/industry/coal- surplus regions to water deficit
gasification-ministry-proposes-tax-sops-for-syngas- regions.
tech-15-methanol-blending-with-petrol/2374047/ ➢ Water availability: Solve the water
https://www.livemint.com/news/india/centre-
crisis by providing alternative, and
allows-50-concession-in-revenue-share-to-promote-
perennial water resources. The poor
coal-gasification-11651851860899.html
section would be given drinkable

e
water with greater equity.

n
Social:

li
Q.60) Ans: A
➢ Food security as ILR helps in achieving
Exp:

n
food security and reducing volatility in
● Recently, the Centre has set in motion the o
the food-grain prices.
process of creating the exclusive body-
➢ Reduce burden on women and girls to
l.
National Interlinking of Rivers Authority
fetch water from long distances. o
a

(NIRA)- to implement river-linking projects in


Improve Health and Education in the
ri

India.
region through reduced disease
Multidimensional Benefits of river-linking
te

burden (due to unsafe drinking water).


projects
Economical: ➢ Arrest the distress migration from the
a

region.
➢ Increase GDP: Enhancement in
m

economic activities especially in


Reference:
c

sectors like agriculture, logistics etc.


https://indianexpress.com/article/india/centre-
which will result in an annual increase
s

river-linking-india-nira-nwda-gajendra-shekhawat-
of GDP accompanied by increased
p

7622936/
employment opportunities and the
.u

https://theprint.in/india/river-linking-nwda-asks-
growth of the services sector.
states-to-spell-out-expectations-from-centre-on-
➢ Enhanced irrigation: Irrigation is a
w

future-course/807865/
crucial input for agriculture growth.
w

For example, the Bhakra dam resulted


in increased irrigation intensity
w

Q.61) Ans: B
substantially in Punjab and Haryana

DPP 2023 DAY 160 52


https://upscmaterial.online/
Download From - https://upscmaterial.online/

Contact us :info@onlyias.com

OnlyIAS Nothing Else Visit :dpp.onlyias.in


Contact : +91-7007 931 912

Exp: grasslands in the Serengeti (Kenya-Tanzania,


● Statement 1 is correct : The Asiatic cheetah is Africa), arid mountain ranges in the Sahara .
classified as a critically endangered species by The cheetah is threatened by several factors
the IUCN Red List, and is believed to survive such as habitat loss, conflict with humans,
only in Iran. Iran is the only country in the poaching and high susceptibility to diseases. In
world to have living population of Asiatic 2016, the global cheetah population was
cheetahs. The Asiatic cheetah (Acinonyx estimated at around 7,100 individuals in the
jubatus venaticus) is a critically endangered wild; it is listed as Vulnerable on the IUCN Red
cheetah subspecies currently only surviving in List. African cheetahs are slightly larger than
Iran. It once occurred from the Arabian their Asian counterparts.
Peninsula and the Near East to the Caspian ● Statement 3 is not correct : Kuno National
region, Transcaucasus, Kyzylkum Desert and Park in Madhya Pradesh was established in
India, but was extirpated in these regions 1981 as wildlife sanctuary and was later given
during the 20th century. The Asiatic cheetah the status of national park in 2018. It was
survives in protected areas in the eastern- named after Kuno River, one of the major
central arid region of Iran, where the human tributaries of the Chambal River. This river
population density is very low. In India last flows through the entire length, bisecting the
spotted cheetah died in Chhattisgarh in 1947 national park. It was chosen by Wildlife
and it was declared extinct in the country in Institute of India and Wildlife Trust of India as
1952.Cheetah has been protected under a possible habitat for the reintroduction of
Appendix I of the Convention on cheetahs. It was also chosen as a possible site

e
International Trade in Endangered Species of to implement the Asiatic Lion Reintroduction

n
Wild Fauna and Flora (CITES) since 1 July 1975 Project, which seeks to establish a second lion

li
which means commercial international trade population in India.
in wild-sourced cheetah is prohibited. The low

n
density of cheetahs throughout their range Reference: o
means they require conservation action on a https://indianexpress.com/article/india/cheetahs-
l.
scale that is seldom seen in terrestrial return-india-pm-narendra-modi-madhya-pradesh-
a

conservation. This includes transboundary kuno-national-park-8155880/


ri

cooperation, land use planning across large


landscapes to maintain habitat connectivity,
te

and human wildlife conflict mitigation. Most Q.62) Ans: B


cheetah range (76%) is on unprotected lands Exp:
a

where they are often persecuted in retaliation ● Statement 1 is not correct : Indus River dolphin
m

for livestock or game depredation. : Their Scientific Name is Platanista gangetica


c

● Statement 2 is not correct : The African minor. According to a recent study of South
cheetah (Acinonyx jubatus jubatus) is the Asian river dolphins, Indus and Ganges River
s

nominate cheetah subspecies native to East dolphins are not one, but two separate species.
p

and Southern Africa. The African cheetah lives Currently, they are classified as two subspecies
.u

mainly in the lowland areas and deserts of the under Platanista gangetica. Like other
Kalahari, the savannahs of Okavango Delta, freshwater dolphins (such as Ganges river
w

and the grasslands of the Transvaal region in dolphins) , the Indus river dolphin is an
w

South Africa. In Namibia, cheetahs are mostly important indicator of the health of a river.
found in farmlands. The cheetah occurs in a Other dolphins found in Indian waters include:
w

variety of habitats such as the Savannah Ganges River Dolphins, Irrawaddy dolphins.

DPP 2023 DAY 160 53


https://upscmaterial.online/
Download From - https://upscmaterial.online/

Contact us :info@onlyias.com

OnlyIAS Nothing Else Visit :dpp.onlyias.in


Contact : +91-7007 931 912

Like other freshwater dolphins (such as congratulated the team that worked on this
Ganges river dolphins) , the Indus river project. The Climate Equity Monitor provides
dolphin is an important indicator of the health an online dashboard for assessing, at the
of a river. The Indus river dolphin was international level, equity in climate action,
declared the State aquatic animal of Punjab in inequalities in emissions, energy and resource
2019. Enumeration of freshwater dolphins is consumption across the world, and ongoing
being undertaken as a nationwide project of climate policies of several countries. The
the Central Government. Project Dolphin was website has been developed by independent
Announced on the eve of Independence Day in researchers from India.
the year 2020, it will be on the lines of Project ● Statement 2 is not correct : The equitable
Tiger, which has helped increase the tiger sharing of the global carbon budget is the
population. fundamental equity principle that will
● Statement 2 is correct : They can only be underpin the assessments that will
found in the lower parts of the Indus River in progressively appear on the website. Aimed at
Pakistan and in River Beas, a tributary of the monitoring the performance of Annex-I
Indus River in Punjab, India . Indus river Parties under the UNFCCC (developed
dolphins are believed to have originated in the countries) based on the foundational
ancient Tethys Sea. They have adapted to life principles of the Climate Convention, namely
in the muddy river and are functionally blind. equity and the principle of common but
They rely on echolocation to navigate, differentiated responsibilities and respective
communicate and hunt prey including prawns, capabilities (CBDR-RC). Annex I Parties include

e
catfish, and carp. the industrialized countries that were

n
members of the OECD (Organisation for

li
Reference : https://www.thehindu.com/sci- Economic Co-operation and Development) in
tech/energy-and-environment/punjab-bats-for- 1992, plus countries with economies in

n
conservation-of-indus-river- transition (the EIT Parties), including the
o
dolphin/article37275721.ece Russian Federation, the Baltic States, and
l.
https://www.worldwildlife.org/species/indus-river- several Central and Eastern European States.
a

dolphin The performance and policies of the Non


ri

Annex-I Parties (developing countries) will be


also provided for comparison. The website is
te

Q.63) Ans: D expected to be a valuable tool for policy


Exp: makers, public institutions, researchers,
a

● Statement 1 is not correct : Recently, the academics, students, and the general public
m

Climate Equity Monitor dashboard has been from developing countries to keep equity and
c

launched by the Ministry of Environment, climate justice considerations clearly in view in


Forest and Climate Change. CEM is an online their perspective.
s

dashboard for assessing, at the international


p

level, equity in climate action, inequalities in Reference :


.u

emissions, energy and resource consumption. https://pib.gov.in/PressReleasePage.aspx?PRID=1768


Climate Equity Monitor, that focuses on equity 174
w

and climate action from a data and evidence-


w

based perspective will encourage a vigorous


discussion on this crucial issue and engage Q.64) Ans: C
w

experts from all countries. The Minister also Exp:

DPP 2023 DAY 160 54


https://upscmaterial.online/
Download From - https://upscmaterial.online/

Contact us :info@onlyias.com

OnlyIAS Nothing Else Visit :dpp.onlyias.in


Contact : +91-7007 931 912

● Statement 1 and statement 3 are correct : Global Methane Pledge was launched at
India proposed a five-fold strategy for it to play COP26 in 2021 to catalyse action to reduce
its part in helping the world get closer to 1.5 methane emissions. Led by the United States
degrees Celsius on the first day of the global and the European Union, the Pledge now has
climate meet in 26th Conference of Parties 111 country participants who together are
(CoP26) to the United Nations Framework responsible for 45% of global human-caused
Convention on Climate Change, Glasgow. methane emissions. By joining the Pledge,
Recently, after a long rejection of net-zero countries commit to work together in order to
target, at the COP26 summit in Glasgow, India collectively reduce methane emissions by at
pledged to achieve net-zero emissions as part least 30% below 2020 levels by 2030. Global
of a five-point action plan. India’s developing Methane Pledge was announced with the aim
economy is still heavily reliant on oil and coal, of reducing methane emissions by 30% by the
the most polluting fossil fuel, which makes up year 2030, as compared to 2020 levels. It is
70% of its energy production. Coal nearly 80-85 times more potent than carbon
consumption in the country has increased by dioxide in terms of its global warming capacity.
39% over the last decade. India is the world’s This makes it a critical target for reducing
third-biggest emitter of greenhouse gases global warming more quickly while
after China and the US. India’s 5-point Action simultaneously working to reduce other
Plan includes 1) India will reach carbon greenhouse gases.Increasing emissions of
neutrality by 2070 ; 2) India also raised India’s Methane are driving a rise in tropospheric
Nationally Determined Contributions (NDCs) ozone air pollution, which causes more than

e
of achieving 450 giga watt non-fossil energy one million premature deaths annually.

n
capacity to 500 giga watt by 2030 ; 3) India will ● Statement 2 is not correct : Besides the EU

li
fulfil 50% of its energy requirements from and US, more than 103 countries have signed
renewable energy sources by 2030. 4) It will up so far, including major methane emitters

n
reduce total projected carbon emissions by a like Nigeria and Pakistan. China, Russia and
o
billion tonnes between now and 2030. 5) India India have not signed up. Australia said it will
l.
will reduce carbon intensity of its economy by not back the pledge.India is a partner country
a

45%. Under Paris Agreement, India promised in Global Methane Initiative (GMI) . It is an
ri

to reduce its emissions intensity (emissions per international public-private partnership


unit of GDP), by 33 to 35% by 2030 compared focused on reducing barriers to the recovery
te

to 2005 levels. and use of methane as a clean energy source.


GMI provides technical support to deploy
a

Reference : methane-to-energy projects around the world


m

https://www.downtoearth.org.in/news/climate- that enable Partner Countries to launch


c

change/cop26-modi-offers-panchamrita-concoction- methane recovery and use projects.


for-climate-conundrum-at-glasgow-80001 ● Statement 3 is not correct : Approximately
s

40% of methane emitted is from natural


p

Q.65) Ans: C sources and about 60% comes from human-


.u

Exp: influenced anthropogenic sources, including


● Statement 1 is correct : Methane is the livestock farming, rice agriculture, biomass
w

simplest hydrocarbon, consisting of one burning and so forth. Methane is produced by


w

carbon atom and four hydrogen atoms (CH4). the breakdown or decay of organic material
It is flammable, and is used as a fuel worldwide. and can be introduced into the atmosphere by
w

Methane is a powerful greenhouse gas. The either natural processes such as the decay of

DPP 2023 DAY 160 55


https://upscmaterial.online/
Download From - https://upscmaterial.online/

Contact us :info@onlyias.com

OnlyIAS Nothing Else Visit :dpp.onlyias.in


Contact : +91-7007 931 912

plant material in wetlands, the seepage of gas Janeiro, Berlin, Kyoto, Bali, Stockholm, Paris,
from underground deposits or the digestion of and Incheon, all named after towns that hold
food by cattle or human activities such as oil key UN climate conferences. Over the past 40
and gas production, rice farming or waste years, satellites have observed huge iceberg
management. calving events, changes in

Reference : https://theprint.in/environment/what-is- Reference :


the-global-methane-pledge-that-103-countries- https://indianexpress.com/article/world/glacier-in-
signed-at-cop26/761154/ antarctica-named-after-glasgow-climate-summit-
7600845/
Q.66) Ans: C
Exp:
● Option C is correct : The 26th session of the Q.67) Ans: D
Conference of the Parties (COP 26) to the Exp:
United Nations Framework Convention on ● United Nations Environment Programme
Climate Change (UNFCCC) is being held in (UNEP) : It is a leading global environmental
Glasgow, UK.Scientists from the University of authority established on 5th June 1972. It sets
Leeds in England have studied a chain of the global environmental agenda, promotes
glaciers in the Getz basin of Antarctica. 14 sustainable development within the United
glaciers in the Getz Basin of West Antarctica Nations system, and serves as an authoritative
are thinning by an average of 25% between advocate for global environment protection.

e
1994 and 2018 due to climate change. The 315 Headquarters is In Nairobi, Kenya.

n
gigatonnes of ice were lost from the region in ● Option 1 is correct : Emissions Gap Report :

li
the last 25 years and contributing to rising UNEP : Recently, the United Nations
global sea levels. The Getz basin is part of Environment Programme (UNEP)’s Emissions

n
Antarctica's largest ice shelf. The shelf is Gap Report 2021 has been published. This is
o
subject to more changeable oceanic forcing - a the twelfth edition of the UNEP Emissions Gap
l.
process where relatively warm deep ocean Report. It informs that the new national
a

water melts the glaciers from below - than climate pledges combined with other
ri

other Antarctic shelves. Recently, the 100-km mitigation measures put the world on track for
long body of ice in Antarctica, which has been a global temperature rise of 2.7°C by the end
te

experiencing rapid melting, was formally of the century.Emissions Gap Report assesses
named Glasgow after the Glasgow climate the gap between anticipated emissions in 2030
a

summit. This glacier, a glacier in far-off and levels consistent with the 1.5°C and 2°C
m

Antarctica has been formally named Glasgow targets of the Paris Agreement. Every year, the
c

Glacier in honour of Glasgow, Scotland, which report features ways to bridge the gap.
is hosting the high-level UN climate ● Option 2 is correct : Sustainable Cooling
s

conference. It is a 100-kilometre-long body of Handbook for Cities : The Sustainable Urban


p

ice which has been rapidly melting. Scientists Cooling Handbook is a report published by the
.u

have studied a chain of glaciers in the Getz United Nations Environment Programme
basin of West Antarctica. 14 glaciers in the (UNEP). The world cities are heating up at
w

Getz Basin (part of Antarctica's largest ice twice the global average. This is mainly due to
w

shelf) are thinning by an average of 25% urban heat island effect. Urban population
between 1994 -2018. Aside from Glasgow, the exposed to hot temperatures (more than 35
w

eight newly named glaciers are Geneva, Rio de degrees Celsius) are to increase by 800% by

DPP 2023 DAY 160 56


https://upscmaterial.online/
Download From - https://upscmaterial.online/

Contact us :info@onlyias.com

OnlyIAS Nothing Else Visit :dpp.onlyias.in


Contact : +91-7007 931 912

2050 as compared to 2016. Around 1.6 billion https://www.unep.org/resources/report/beating-


urban population is to face this heat. With heat-sustainable-cooling-handbook-cities
increase in temperature in cities, the low-
income countries, especially in western Africa Q.68) Ans: D
and southern Asia will be worst hit. Such Exp:
regions are to lose 5% of working hours. The ● Statement 1 is not correct : Background
electric grid failures during extreme weather extinction rate, also known as the normal
are increasing. This along with increasing heat extinction rate, refers to the standard rate of
waves will expose large population to severe extinction in Earth's geological and biological
heat stress. history before humans became a primary
● Option 3 is correct : Annual Frontiers Report contributor to extinctions. This is primarily the
:The United Nations Environment Programme pre-human extinction rates during periods in
(UNEP) has released its Annual Frontiers between major extinction events. Extinction is
Report named Noise, Blazes and Mismatches. a part of life, and species disappear all the
The document has been released 10 days time. About 98% of all the species that have
ahead of the UN Environment Assembly. The ever existed on our planet are now extinct.
Frontiers report identifies and offers solutions Earth’s normal extinction rate is often
to three environmental issues: urban noise thought to be somewhere between 0.1 and 1
pollution, wildfires and phenological shifts that species per 10,000 species per 100 years. This
merit attention and action from governments is known as the background rate of
and the public at large to address the triple extinction.Extinctions are a normal part of the

e
planetary crisis of climate change, pollution evolutionary process, and the background

n
and biodiversity loss. extinction rate is a measurement of "how

li
● Option 4 is correct : Food Waste Index Report often" they naturally occur. Normal extinction
: The United Nations Environment rates are often used as a comparison to

n
Programme (UNEP) released the Food Waste present day extinction rates, to illustrate the
o
Index Report 2021. It has revealed that 17% of higher frequency of extinction today than in all
l.
all food available at consumer level (11% in periods of non-extinction events before it.
a

households, 5% in food service and 2% in retail) Background extinction rates have not
ri

was wasted in 2019 and around 690 million remained constant, although changes are
people had to go hungry. It presents the most measured over geological time, covering
te

comprehensive food waste data collection, millions of years


analysis and modelling to date, generating a ● Statement 2 is not correct : A mass extinction
a

new estimate of global food waste. It also event is when species vanish much faster than
m

publishes a methodology for countries to they are replaced. This is usually defined as
c

measure food waste, at household, food about 75% of the world's species being lost in
service and retail level, to track national a 'short' amount of geological time - less than
s

progress towards 2030. In contrast to the Food 2.8 million years. Currently our planet is
p

Loss Index, the Food Waste Index measures experiencing a Sixth mass extinction as the
.u

total food waste (rather than loss or waste result of human-induced climate change
associated with specific commodities). referred to as the Holocene or Anthropocene
w

extinction .
w

Reference : ● The Sixth Mass Extinction : The Holocene


https://www.unep.org/resources/emissions-gap- extinction also referred as the 6th mass
w

report-2021 extinction or Anthropocene extinction is an

DPP 2023 DAY 160 57


https://upscmaterial.online/
Download From - https://upscmaterial.online/

Contact us :info@onlyias.com

OnlyIAS Nothing Else Visit :dpp.onlyias.in


Contact : +91-7007 931 912

ongoing extinction event of species during the around 10 to 15 meters. It is characterised by


present Holocene epoch as a result of human not-too-broad leaves.
activity. The rates of extinction are faster and ● Muthuvar Tribal Community : Muthuvan
are estimated to be 100-1000 times faster people were loyal subjects of Madurai
than in the previous one . Some researchers dynasty. When this dynasty was deposed,
have pointed out that we are currently surviving royal members migrated to
experiencing a sixth mass extinction as the Travancore in Kerala. They carried idols of
result of human-induced climate change Madurai Meenakshi, deity of the royal family
(referred to as the Anthropocene extinction). while migrating. ‘Muthuvar word’ is used to
Currently, only an estimated 2% of all of the denote same community in Tamil Nadu.
species that ever lived are alive but the Muthuvan people are ancient tribes of this
absolute number of species is greater than land. This community is independent and
ever before. It is described as the most serious reluctant to interact with others. They grow
environmental problem since the loss of ragi, lemon grass and cardamom.
species will be permanent. The loss of species
has been occurring since human ancestors Reference :
developed agriculture over 11,000 years ago. https://www.thehindu.com/news/national/kerala/ne
Since then, the human population has w-tree-species-named-after-muthuvar-
increased from about 1 million to 7.7 billion. tribe/article37579221.ece

Reference :

e
https://indianexpress.com/article/technology/science Q.70) Ans: B

n
/witnessing-onset-sixth-mass-extinction-study- Exp:

li
7731909/ ● Flash droughts are a type of extreme event
distinguished by a sudden onset and rapid

n
Q.69) Ans: C intensification of drought conditions with
o
Exp: severe impacts.
l.
● Option 3 is Correct : Cryptocarya ● Statement 1 is not correct : The majority of
a

Muthuvariana: A group of scientists has the country-level and regional scale flash
ri

identified a new plant species from the forest droughts in India occur during the monsoon
area of Idamalakkudy colony in Idukki, where season. especially across the central,
te

Muthuvar tribal community resides. Scientists northwest and northeast regions of the
including from University of Kerala and country and primarily between May and
a

Jawaharlal Nehru Tropical Botanical Garden & September from 1980-2015. More than 82% of
m

Research Institute (TBGRI), identified the plant. country-level flash droughts occurred during
c

New species of the plant has been named as the monsoon season in India. A considerably
‘Cryptocaria Muthuvariana’, named after the long dry spell with significantly low
s

Muthuvar tribal community. Muthuvar Tribal precipitation anomalies during the monsoon
p

community plays a significant role in results in an increase in air temperature.


.u

conserving the forest. For the first time, any Increased air temperature and precipitation
plant species has been named after a local deficit together cause a rapid depletion of soil
w

tribal community in the state. Cryptocarya moisture leading to flash drought. Therefore,
w

Muthuvariana species of the tree belongs to flash droughts in the monsoon season are
genus Cryptocarya. The tree grows to a top of primarily caused by the monsoon breaks.
w

Flash droughts generally have short periods of

DPP 2023 DAY 160 58


https://upscmaterial.online/
Download From - https://upscmaterial.online/

Contact us :info@onlyias.com

OnlyIAS Nothing Else Visit :dpp.onlyias.in


Contact : +91-7007 931 912

warm surface temperature. The increased air India amended the MTP Act 1971 to further
temperature depletes the newly replenished empower women by providing comprehensive
soil moisture in the surface due to the abortion care to all. The new Medical
monsoon. The decrease in soil moisture due to Termination of Pregnancy (Amendment) Act
warm, dry atmospheric conditions along with 2021 expands the access to safe and legal
anthropogenic reasons such as intense abortion services on therapeutic, eugenic,
cropping and cropping of water-intensive humanitarian and social grounds to ensure
crops in low moisture areas delete the soil universal access to comprehensive care.
moisture at a rapid pace resulting in flash Opinion of one Registered Medical
droughts. Delayed onset of summer monsoon Practitioner (RMP) for termination of
leads to precipitation deficit, which in turn pregnancy up to 20 weeks of gestation.
increases the surface temperature thus Opinion of two RMPs for termination of
contributing to flash droughts. pregnancy of 20-24 weeks of gestation.
● Statement 2 is correct : Unlike slow-evolving Opinion of the State-level medical board is
drought, which is caused by a decline in essential for a pregnancy to be terminated
precipitation, flash drought occurs when low after 24 weeks in case of substantial foetal
precipitation is accompanied by abnormally abnormalities.
high temperatures (e.g., heat waves), high ● Statement 2 is not correct : Under the Act, a
winds, and/or changes in radiation. Unlike pregnancy may be terminated up to 20 weeks
conventional droughts that develop over by a married woman in the case of failure of
months, flash droughts intensify very quickly. contraceptive method or device. It allows

e
This happens because rains stay away for 15- unmarried women to also terminate a

n
20 days at a stretch.These rapid changes can pregnancy for this reason . Increases the

li
quickly raise evapotranspiration rates and upper gestation limit from 20 to 24 weeks for
remove available water from the landscape. special categories of women, including

n
Higher temperature increases survivors of rape, victims of incest and other
o
evapotranspiration the process by which water vulnerable women (differently abled women,
l.
is transferred from the land to the atmosphere minors, among others).The Act allows abortion
a

by evaporation from soil and by transpiration after 24 weeks only in cases where a Medical
ri

from plants. Geographic differences and Board diagnoses substantial foetal


climate patterns also impact the development abnormalities. This implies that for a case
te

of flash drought . requiring abortion due to rape, that exceeds


24-weeks, the only recourse remains through a
a

Reference : Writ Petition.


m

https://indianexpress.com/article/technology/science ● Statement 3 is correct: The “name and other


c

/flash-droughts-affect-india-7639227/ particulars of a woman whose pregnancy has


been terminated shall not be revealed”, except
s

Q.71) Ans: C to a person authorised in any law that is


p

Exp: currently in force.


.u

● statement 1 is not correct : The Medical


Termination of Pregnancy Act, 1971 (“MTP Reference :
w

Act”) was passed due to the progress made in https://www.newindianexpress.com/nation/2022/aug


w

the field of medical science with respect to /24/need-to-fine-tunerules-to-allow-abortion-till-24-


safer abortions. In a historic move to provide weeks-of-pregnancy-sc-on-mtp-act-2490856.html
w

universal access reproductive health services,

DPP 2023 DAY 160 59


https://upscmaterial.online/
Download From - https://upscmaterial.online/

Contact us :info@onlyias.com

OnlyIAS Nothing Else Visit :dpp.onlyias.in


Contact : +91-7007 931 912

Q.72) Ans: A Reference :


Exp: https://pib.gov.in/PressReleaseIframePage.aspx?PRID
● Statement 1 is not correct : PM Adi Adarsh =1849956
Gram Yojna (PMAAGY) : Government has
modified the earlier scheme of ‘Special Central
Assistance to Tribal Sub-Scheme (SCA to TSS) Q.73) Ans: B
with nomenclature ‘Pradhan Mantri Adi Exp:
Adarsh Gram Yojna (PMAAGY) . PMAAGY aims ● Statement 1 is not correct : Har Ghar Jal’ is a
at mitigating gaps and providing basic flagship programme of the Union
infrastructure in villages with significant tribal Government, implemented by Jal Jeevan
population in convergence with funds available Mission (JJM) under the Ministry of Jal Shakti,
under different schemes in Central Scheduled in partnership with States/ UTs to ensure tap
Tribe Component. PMAAGY is envisaged to water connection in every rural household by
cover 36,428 villages having at least 50% 2024. More than 52% rural households in India
Scheduled Tribes' population and 500 are now connected with tap water which was
Scheduled Tribess across States / UTs with only 17% at the time of programme’s launch in
notified Scheduled Tribes' during the period. 2019. JJM also implements source
● Statement 2 is not correct : Pradhan Mantri sustainability measures as mandatory
Adi Adarsh Gram Yojna (PMAAGY)’, for elements, such as recharge and reuse through
implementation during 2021-22 to 2025-26, grey water management, water conservation,
which aims at mitigating gaps and providing and rain water harvesting

e
basic infrastructure in villages with significant ● Statement 2 is correct : The mission aims to

n
tribal population in convergence with funds assist, empower and facilitate: States/ UTs in

li
available under different schemes in Central the planning of participatory rural water
Scheduled Tribe Component.A sum of ₹20.38 supply strategy for ensuring potable drinking

n
lakh per village as ‘gap-filling’ has been water security on a long-term basis to every
o
provisioned for approved activities including rural household and public institution ; States/
l.
administrative expenses under PMAAGY . An UTs for creation of water supply infrastructure,
a

amount of Rs. 7,276 cr has been approved by so that every rural household has a Functional
ri

the Cabinet for the scheme in next 5 years. Tap Connection (FHTC) by 2024 and water in
Besides, States / UTs are encouraged for adequate quantity of prescribed quality is
te

convergence of resources as Central / State made available on regular basis ; Gram


Scheduled Tribe Component (STC) funds and Panchayats (GPs)/ rural communities to plan,
a

other financial resources available with them implement, manage, own, operate, and
m

for saturation of infrastructure and services in maintain their own in-village water supply
c

the villages identified under PMAAGY. systems ; States/ UTs to develop robust
● Statement 3 is correct : PM Adi Adarsh Gram institutions having a focus on service delivery
s

Yojna (PMAAGY) envisions to mitigate gaps in and financial sustainability of the sector by
p

prominent 8 sectors of development viz. Road promoting a utility approach ; Further


.u

connectivity (Internal and Inter village /block), enhancing capacity building of the
Telecom connectivity (Mobile /internet), stakeholders and creating awareness in the
w

School, Anganwadi Centres, Health Sub- community on the significance of water for
w

Centre, Drinking water facility, Drainage and improvement in quality of life.


Solid waste management. ● Statement 3 is not correct : Recently, Goa and
w

Dadra & Nagar Haveli and Daman & Diu

DPP 2023 DAY 160 60


https://upscmaterial.online/
Download From - https://upscmaterial.online/

Contact us :info@onlyias.com

OnlyIAS Nothing Else Visit :dpp.onlyias.in


Contact : +91-7007 931 912

(D&NH and D&D) became the first ‘Har Ghar corporation can raise loans and take measures
Jal’ certified State and UT in the country for discharging such loans with the prior
respectively. The people from all the villages sanction of the central government and it can
have declared their village as Har Ghar Jal acquire both movable and immovable
through a resolution passed by Gram Sabha, property and all incomes from the property
certifying that all households in the villages shall vest with the corporation. Employees'
have access to safe drinking water through State Insurance Corporation (abbreviated as
taps. Village Water and Sanitation Committee ESIC) is one of the two main statutory social
(VWSC) or paani Samiti has been constituted in security bodies under the ownership of
all the 378 villages of Goa and 96 villages of Ministry of Labour and Employment,
D&NH and D&D. Government of India, the other being the
Employees' Provident Fund Organisation. The
Reference : fund is managed by the Employees' State
https://www.pib.gov.in/PressReleasePage.aspx?PRID= Insurance Corporation (ESIC) according to
1852929 rules and regulations stipulated in the ESI Act
1948.
● Statement 3 is correct : The employees
Q.74) Ans: C registered under the scheme are entitled to
Exp: medical treatment for themselves and their
● Statement 1 is not correct : Employees' State dependents, unemployment cash benefit in
Insurance Scheme was launched under ESI certain contingencies and maternity benefit in

e
Act, 1948 by Ministry of Labour and case of women employees. In case of

n
Employment. It aims to provide social security employment-related disablement or death,

li
to the industrial workers in certain there is provision for a disablement benefit
contingencies such as sickness, maternity, and a family pension respectively.

n
temporary or permanent physical o
disablement and death due to employment Reference :
l.
injury resulting in loss of wages or earning https://www.thehindu.com/news/cities/Madurai/esic
a

capacity. Employees’ state Insurance -hospital-to-be-ready-in-a-year/article65815124.ece


ri

Corporation of India is a multidimensional


social system which provides socio-economic
te

protection to the worker population and Q.75) Ans: C


immediate dependent or family covered under Exp:
a

the ESI scheme. The Employees' State ● Statement 1 is not correct : Recently, the
m

Insurance Scheme (ESI) is an integrated Criminal Procedure (Identification) Act, 2022


c

measure of social Insurance embodied in the has come into force after being passed by the
Employees' State Insurance Act, 1948. Parliament in April 2022. It replaces the
s

● Statement 2 is not correct : Employees' State Identification of Prisoners Act, 1920, a


p

Insurance Corporation (ESIC), established by colonial era law, and authorises police officers
.u

ESI Act 1948 , is an autonomous corporation to take measurements of people convicted,


under Ministry of Labour and Employment, arrested or facing trial in criminal
w

Government of India. The ESI Scheme is cases.Criminal Procedure (Identification) Act,


w

administered by a statutory corporate body 2022 provides Legal sanction to the police to
called the Employees' State Insurance take physical and biological samples of
w

Corporation (ESIC). As it is a legal entity, the convicts as well as those accused of crimes.

DPP 2023 DAY 160 61


https://upscmaterial.online/
Download From - https://upscmaterial.online/

Contact us :info@onlyias.com

OnlyIAS Nothing Else Visit :dpp.onlyias.in


Contact : +91-7007 931 912

The police as per section 53 or section 53A of ● Statement 1 is not correct : Fast Track Courts
the Code of Criminal Procedure (CrPC), 1973, (FTCs) are set up by the State Governments in
can collect Data. Data that can be collected: consultation with the concerned High Courts.
Finger-impressions, Palm-Print impressions, The 11th Finance Commission had
Footprint impressions, Photographs, Iris and recommended a scheme for the creation of
Retina scan, Physical, Biological samples and 1734 FTCs in the country for the disposal of
their analysis, Behavioural Attributes including long pending cases. Fast Track Special Courts
signatures, Handwriting or any other are dedicated courts expected to ensure swift
examination dispensation of justice. They have a better
● Statement 2 is correct : Any person convicted, clearance rate as compared to the regular
arrested or detained under any preventive courts and hold speedy trials. Besides
detention law will be required to provide providing quick justice to the hapless victims, it
"measurements" to a police officer or a prison strengthens the deterrence framework for
official.National Crime Records Bureau (NCRB) sexual offenders. The 14th Finance
will store, preserve, share with any law Commission had recommended the setting up
enforcement agency and destroy the record of 1800 FTCs during 2015-20 dealing with cases
of measurements at national level. The of heinous crimes; civil cases related to
records can be stored up to a period of 75 women, children, senior citizens, HIV/AIDS etc.
years. The National Crime Records Bureau, and property related cases pending for more
abbreviated to NCRB, is an Indian government than 5 years. The Commission also urged State
agency responsible for collecting and Governments to utilize enhanced fiscal space

e
analyzing crime data as defined by the Indian available through tax devolution (32% to 42%)

n
Penal Code (IPC) and Special and Local Laws for this purpose. 892 FTCs are functional across

li
(SLL)It aims to ensure the unique identification the country (June 2022). Total more than 33
of those involved with crime and to help lakhs pending cases have been disposed by

n
investigating agencies solve cases. FTCs since 2014. o
● Statement 3 is not correct : Indian Penal Code, ● Statement 2 is correct : Article 247 gives
l.
1860 (IPC) and the Criminal Procedure Code, power to Parliament to establish certain
a

1974 (CrPC) are the laws that govern criminal additional courts for the better
ri

law in India. Indian penal code IPC is the administration of laws made by it or of any
principal criminal code of India that defines existing laws with respect to a matter
te

crimes and provides punishments for almost enumerated in the Union List.247. Power of
all kinds of criminal and actionable wrongs. Parliament to provide for the establishment of
a

CrPC is the procedural law that provides a certain additional courts Notwithstanding
m

detailed procedure for punishments under anything in this Chapter, Parliament may by
c

penal laws. law provide for the establishment of any


additional courts for the better administration
s

Reference : of laws made by Parliament or of any existing


p

https://www.thehindu.com/news/national/explained- laws with respect to a matter enumerated in


.u

what-is-the-criminal-procedure-identification-act- the Union List


2022/article65757554.ece
w

Reference : https://www.barandbench.com/news/fill-
w

up-judges-vacancies-fast-track-courts-hearing-rape-
Q.76) Ans: A pocso-cases-delhi-lg-to-aap-government
w

Exp:

DPP 2023 DAY 160 62


https://upscmaterial.online/
Download From - https://upscmaterial.online/

Contact us :info@onlyias.com

OnlyIAS Nothing Else Visit :dpp.onlyias.in


Contact : +91-7007 931 912

https://www.hindustantimes.com/india-news/unclog- majority of all the then members of the Rajya


jails-to-mark-75thyear-of-independence-supreme- Sabha and agreed to by the Lok Sabha. This
court-to-centre-101659726980417.html means that this resolution should be passed in
the Rajya Sabha by an effective majority and in
the Lok Sabha by a simple majority. This
Q.77) Ans: B resolution can be introduced only in Rajya
Exp: Sabha and not in Lok Sabha. But, no such
● Statement 1 is correct : The Vice President is resolution can be moved unless at least 14
the second highest constitutional office in days’ advance notice has been given. No
India. He/She serves for a five-year term, but ground has been mentioned in the
can continue to be in office. irrespective of the Constitution for his removal.
expiry of the term, until the successor assumes ● The President of India be removed from office
office. The Vice President may resign his office on the ground of violation of the Constitution.
by submitting his resignation to the President S/he can be removed by the Parliament by the
of India. The resignation becomes effective process of impeachment which is mentioned in
from the day it is accepted. Eligibility the Article 61 of the Constitution of India.
conditions for becoming an Vice President are
: The person Should be a citizen of India Should Reference : https://www.hindustantimes.com/india-
have completed 35 years of age ; Should be news/explained-how-the-vice-president-of-india-is-
qualified for election as a member of the Rajya elected-101659719738263.html
Sabha ; Should not hold any office of profit

e
under the Union government or any state

n
government or any local authority or any other Q.78) Ans: A

li
public authority. As per Article 66 of the Exp:
Constitution of India, the Vice-President is ● Statement 1 is correct : Tribunals were not

n
elected by the members of the Electoral part of the original constitution, it was
o
College. Electoral College consists of: Elected incorporated in the Indian Constitution by
l.
members of Rajya Sabha, Nominated members 42nd Amendment Act, 1976. Article 323-A
a

of Rajya Sabha and Elected members of Lok deals with Administrative Tribunals. Article
ri

Sabha. Electoral college of president consists 323-B deals with tribunals for other matters.
of both elected and nominated members of While Article 323 A contemplates the
te

the Parliament (in the case of president, only establishment of tribunals for public service
elected members) but it does not include the matters only, Article 323 B contemplates the
a

members of the state legislative assemblies establishment of tribunals for certain other
m

(in the case of President, the elected matters . While tribunals under Article 323 A
c

members of the state legislative assemblies can be established only by Parliament,


are included). tribunals under Article 323 B can be
s

● Statement 2 is not correct : The Vice-President established both by Parliament and state
p

holds office for a term of five years from the legislatures with respect to matters falling
.u

date on which he enters upon his within their legislative competence. Under
office.However, he can resign from his office at Article 323 A, only one tribunal for the Centre
w

any time by addressing the resignation letter to and one for each state or two or more states
w

the President. He can also be removed from may be established. There is no question of the
the office before completion of his term. He hierarchy of tribunals, whereas under Article
w

can be removed by a resolution passed by a 323 B a hierarchy of tribunals may be created.

DPP 2023 DAY 160 63


https://upscmaterial.online/
Download From - https://upscmaterial.online/

Contact us :info@onlyias.com

OnlyIAS Nothing Else Visit :dpp.onlyias.in


Contact : +91-7007 931 912

● Under Article 323 B, the Parliament and the Reference :


state legislatures are authorised to provide for https://pib.gov.in/PressReleasePage.aspx?PRID=1853
the establishment of tribunals for the 303
adjudication of disputes relating to the
following matters:
a. Taxation Q.79) Ans: A
b. Foreign exchange, import and export Exp:
c. Industrial and labour ● Statement 1 is not correct : The thylacine
d. Land reforms (Thylacinus cynocephalus) is an extinct
e. Ceiling on urban property carnivorous marsupial that was native to the
f. Elections to Parliament and state legislatures Australian mainland and the islands of
g. Food stuff Tasmania and New Guinea. Tasmanian Tiger
h. Rent and tenancy rights (Thylacinus cynocephalus), the only animal in
● Statement 2 is not correct : Armed Forces the Thylacinidae family to survive in modern
Tribunal (AFT) : It is a military tribunal in India. times, was a marsupial mammal that raises
It was established under the Armed Forces young ones in a pouch. They were slow-paced
Tribunal Act, 2007. It has provided the power carnivorous that usually hunted alone or in
for the adjudication or trial by AFT of disputes pairs at night. The sharply clawed animal had a
and complaints with respect to commission, dog-like head and ate kangaroos, other
appointments, enrolments and conditions of marsupials, small rodents, and birds.The last
service in respect of persons subject to the known live animal was captured in 1930 in

e
Army Act, 1950, The Navy Act, 1957 and the Air Tasmania. It is commonly known as the

n
Force Act, 1950. Besides the Principal Bench in Tasmanian tiger (because of its striped lower
back) or the Tasmanian wolf (because of its

li
New Delhi, AFT has Regional Benches at
Chandigarh, Lucknow, Kolkata, Guwahati, canid-like characteristics).It was confined to

n
Chennai, Kochi, Mumbai and Jaipur. Each Tasmania in recent times and disappeared
o
Bench comprises of a Judicial Member and an from mainland Australia over 2000 years ago,
l.
Administrative Member. The Judicial Members mainly because of over-hunting by humans,
a

are retired High Court Judges and diseases and competition from the Dingo
ri

Administrative Members are retired Members (Canis lupus), a wild dog native to Australia.
of the Armed Forces who have held the rank of The Thylacine was also persecuted because it
te

Major General/ equivalent or above for a was believed to be a threat to sheep and in its
period of three years or more, Judge Advocate latter years it was hunted for the purposes of
a

General (JAG), who have held the appointment collection by museums and zoos. The last
m

for at least one year are also entitled to be known thylacine died in captivity over 80
years ago, in Tasmania’s Hobart Zoo in 1936.
c

appointed as the Administrative Member.


Armed Forces Tribunal has both Original and It is also known as the Tasmanian Wolf and
s

Appellate Jurisdiction. Tribunal is empowered bears some resemblance to a dog, with its
p

to adjudicate appeals against any order, distinguishing features being the dark stripes
.u

decision, finding or sentence passed by a beginning at the rear of its body and extending
court-martial or any related matter. It is also into its stiff tail and abdominal pouch.
w

empowered to grant bail to an accused that is ● Statement 2 is correct : According to the


w

in military custody. International Union for Conservation of


Nature (IUCN), the world authority on rare
w

and threatened species, the thylacine a dog-

DPP 2023 DAY 160 64


https://upscmaterial.online/
Download From - https://upscmaterial.online/

Contact us :info@onlyias.com

OnlyIAS Nothing Else Visit :dpp.onlyias.in


Contact : +91-7007 931 912

sized predatory marsupial also known as the Sharing of Benefits Arising from their
Tasmanian tiger was extinct in 1982. The last Utilization to the Convention on Biological
known individual died in 1936 in Hobart Zoo; Diversity, also known as the Nagoya Protocol
the last reliable sighting of a wild one dates on Access and Benefit Sharing is a 2010
back to 1933. The species died out some time supplementary agreement to the 1992
after the mid-1960s. Convention on Biological Diversity. The
Cartagena Protocol on Biosafety is a legally
Reference : binding global protocol that seeks to
https://indianexpress.com/article/explained/explaine contribute to ensuring the safe transfer,
d-sci-tech/tasmanian-tiger-de-extinction-project- handling and use of living modified organisms
explained-8098355/ (LMOs) created through modern
biotechnology.

Q.80) Ans: B Reference :


Exp: https://pib.gov.in/PressReleasePage.aspx?PRID=1852
● Statement 1 is not correct : Traditional 528
Knowledge Digital Library TKDL, first of its kind
globally, is a database of Indian traditional
knowledge established in 2001, by Council of Q.81) Ans: A
Scientific and Industrial Research (CSIR) and Exp:
Ministry of AYUSH. It contains information ● Statement 1 is correct : Project Elephant (PE)

e
related to Ayurveda, Unani, Siddha, Sowa was launched by the Government of India in

n
Rigpa, and Yoga in five international languages the year 1992 as a Centrally Sponsored

li
(English, German, French, Japanese and Scheme with following objectives: To protect
Spanish). It seeks to prevent misappropriation elephants, their habitat and corridors ; To

n
of country's traditional medicinal knowledge address issues of man-animal conflict; Welfare
o
through patenting worldwide. TKDL is an of captive elephants to promote not to harm
l.
effective deterrent against bio-piracy. elephants for their tusks.The project aims to
a

● Statement 2 is correct : Biological Diversity ensure the long-term survival of the


ri

Act, 2002 provides for preservation, population of elephants in their natural


maintenance of Traditional knowledge along habitats by protecting them, their habitats and
te

with equitable sharing of benefits arising from migration corridors. Other goals of Project
use of the knowledge with communities. The Elephant are supporting the research of the
a

Biological Diversity Act, 2002 was born out of ecology and management of elephants,
m

India’s attempt to realise the objectives creating awareness of conservation among


c

enshrined in the United Nations Convention on local people, providing improved veterinary
Biological Diversity (CBD) 1992 which care for captive elephants
s

recognizes the sovereign rights of states to use ● Statement 2 is not correct : Under project
p

their own Biological Resources. Elephant , Financial support is being provided


.u

● Statement 3 is not correct : India is a signatory to major elephant bearing States in the
to Convention on Biological Diversity and country. The Project is being mainly
w

Nagoya Protocol, which are concerned with implemented in 16 States / UTs, viz. Andhra
w

trade-in bioresources and use of Traditional Pradesh, Arunachal Pradesh, Assam,


Knowledge . The Nagoya Protocol on Access to Chhattisgarh, Jharkhand, Karnataka, Kerala,
w

Genetic Resources and the Fair and Equitable Maharashtra, Meghalaya, Nagaland, Orissa,

DPP 2023 DAY 160 65


https://upscmaterial.online/
Download From - https://upscmaterial.online/

Contact us :info@onlyias.com

OnlyIAS Nothing Else Visit :dpp.onlyias.in


Contact : +91-7007 931 912

Tamil Nadu, Tripura, Uttarakhand, Uttar Q.82) Ans: C


Pradesh, West Bengal. Main activities under Exp:
the Project. The Indian Elephant is widely seen ● Statement 1 is correct : India signed the
in 16 of the 28 states of India, especially is Antarctic Treaty on 19th August 1983 and
found in the central and southern Western soon thereafter received consultative status
Ghats, North East India, eastern India and on 12th September 1983.India is one of the 29
northern India and in some parts of southern Consultative Parties to the Antarctic
peninsular India. It is included in Schedule I of Treaty.The primary purpose of the Antarctic
the Indian Wildlife (Protection) Act, 1972 and Treaty is to ensure "in the interests of all
in Appendix I of the Convention on mankind that Antarctica shall continue forever
International Trade in Endangered Species of to be used exclusively for peaceful purposes
Flora and Fauna (CITES).As per the last count in and shall not become the scene or object of
2017, India has 29,964 elephants. Karnataka international discord." To this end it prohibits
has the highest number of elephants (6,049), military activity, except in support of science;
followed by Assam (5,719) and Kerala (3,054) prohibits nuclear explosions and the disposal
● Statement 3 is not correct : Recently, the of nuclear waste; promotes scientific research
Government of India has announced the and the exchange of data; and holds all
notification of one more Elephant Reserve territorial claims in abeyance.The original
(ER) Agasthiyamalai in Tamil Nadu during a Parties to the Treaty were the 12 nations active
programme in the Periyar Wildlife Sanctuary in in the Antarctic during the International
Kerala. This will be the 32nd Elephant Reserve Geophysical Year of 1957-58. The Treaty was

e
in the country after Singphan ER in Nagaland signed in Washington on 1 December 1959 and

n
was notified in 2018 .Agasthiyamalai is Tamil entered into force on 23 June 1961. The

li
Nadu’s 5th Elephant Reserve and also a Consultative Parties comprise the original
Biosphere Reserve.Designation of the area as a Parties and other States that have become

n
dedicated reserve forest will also provide Consultative Parties by acceding to the Treaty
o
connectivity of the elephant migratory paths in and demonstrating their interest in Antarctica
l.
the landscape through proper research and by carrying out substantial scientific activity
a

effective management. As per 2010 Census, a there.


ri

total of 1800 Asian elephants are likely to be ● Statement 2 is not correct : The Madrid
located within the Periyar-Agasthyamalai Protocol was signed by India which came into
te

landscape. Of this, around 300 are locally force on 14th January, 1998.The Protocol on
situated in the “southern side in the Environmental Protection to the Antarctic
a

Agasthiyarmalai Elephant Reserve and Treaty (also known as the Madrid Protocol)
m

Mahendragiri hill ranges in the Neyyar, provides for comprehensive protection of


c

Shendurney, and Peppara Wildlife Sanctuaries Antarctica. The Protocol designates Antarctica
and Kalakkad-Mundanthurai Tiger Reserve, all as a ‘natural reserve, devoted to peace and
s

located in Thiruvananthapuram Forest science’; establishes environmental principles


p

Division”, as per the official notification. that must be a fundamental consideration in


.u

the planning and conduct of all activities ;


Reference : prohibits mining indefinitely ; requires that all
w

https://www.thehindu.com/news/cities/chennai/tami proposed activities must be subject to a prior


w

l-nadu-notifies-fifth-elephant-reserve-in- assessment of their environmental impacts;


agasthyamalai-landscape/article65762526.ece establishes the Committee for Environmental
w

Protection, to develop advice and formulate

DPP 2023 DAY 160 66


https://upscmaterial.online/
Download From - https://upscmaterial.online/

Contact us :info@onlyias.com

OnlyIAS Nothing Else Visit :dpp.onlyias.in


Contact : +91-7007 931 912

recommendations to the Antarctic Treaty is located at Geneva, Switzerland. It plays a


Parties on implementation of the Protocol; coordinating, advisory and servicing role in the
requires the development of contingency plans working of the Convention (CITES). The
to respond to environmental emergencies. Conference of the Parties to CITES, is the
● Statement 3 is not correct : India has two supreme decision-making body of the
active research stations; Maitri Convention and comprises all its Parties.
(commissioned in 1989) at Schirmacher Hills, ● Statement 3 is not correct : The CITES works by
and Bharati (commissioned in 2012) at subjecting international trade in specimens of
Larsemann Hills in Antarctica. India has selected species to certain
successfully launched 40 annual scientific controls.Appendices I, II and III to the
expeditions to Antarctica till date. With Convention are lists of species afforded
Himadri station in Ny-Alesund, Svalbard, different levels or types of protection from
Arctic, India now belongs to the elite group of over-exploitation.Appendix I lists species that
nations that have multiple research stations are the most endangered among CITES-listed
within the Polar Regions. animals and plants.Appendix II lists species
that are not necessarily now threatened with
Reference : extinction but that may become so unless
https://pib.gov.in/PressReleasePage.aspx?PRID=1760 trade is closely controlled.Appendix III is a list
870 of species included at the request of a Party
that already regulates trade in the species and
Q.83) Ans: B that needs the cooperation of other countries

e
Exp: to prevent unsustainable or illegal

n
● Statement 1 is correct : The Convention on exploitation.Species may be added to or

li
International Trade in Endangered Species of removed from Appendix I and II, or moved
Wild Fauna and Flora (CITES) is an between them, only by the Conference of the

n
international agreement to which States and Parties. However, species may be added to or
o
regional economic integration organizations removed from Appendix III at any time and by
l.
adhere voluntarily. CITES was drafted as a any Party unilaterally.
a

result of a resolution adopted in 1963 at a


ri

meeting of members of the International Reference :


Union for Conservation of Nature (IUCN). The https://www.indiatoday.in/environment/story/what-
te

IUCN is a membership Union uniquely is-wild-life-protection-amendment-bill-cleared-by-lok-


composed of both government and civil sabha-1983226-2022-08-03
a

society organisations. It provides public,


m

private and non-governmental organisations Q.84) Ans: C


c

with the knowledge and tools that enable Exp:


human progress, economic development and ● Statement 1 is not correct : Bharat stage
s

nature conservation to take place together. emission standards (BSES) are emission
p

CITES entered into force in July 1975. standards instituted by the Government of
.u

Currently there are 184 Parties (include India to regulate the output of air pollutants
countries or regional economic integration from compression ignition engines and Spark-
w

organizations). ignition engines equipment, including motor


w

● Statement 2 is not correct : The CITES vehicles. The standards and the timeline for
Secretariat is administered by UNEP (The implementation are set by the Central
w

United Nations Environment Programme) and Pollution Control Board under the Ministry of

DPP 2023 DAY 160 67


https://upscmaterial.online/
Download From - https://upscmaterial.online/

Contact us :info@onlyias.com

OnlyIAS Nothing Else Visit :dpp.onlyias.in


Contact : +91-7007 931 912

Environment, Forest and Climate Change. The Reference: https://www.business-


standards, based on European regulations standard.com/article/current-affairs/govt-allows-
were first introduced in 2000. Progressively retrofitment-of-cng-and-lpg-kits-in-bs-vi-compliant-
stringent norms have been rolled out since vehicles-122082201096_1.html
then. All new vehicles manufactured after the
implementation of the norms have to be
compliant with the regulations. Q.85) Ans: A
● Statement 2 is not correct : Both BS4 and BS6 Exp:
are emission norms that set the maximum ● Statement 1 is correct : The Centre for Wildlife
permissible levels for pollutants emitting from Rehabilitation and Conservation (CWRC) at
a car or a two-wheeler exhaust. Compared to Panbari has completed 20 years of providing
the BS4, BS6 emission standards are stricter. emergency care, treatment and rehabilitation
While manufacturers use this change to to indigenous wild animals displaced due to
update their vehicles with new features and various reasons. Analysis CWRC is the only
safety standards, the biggest or the significant facility in India where orphaned and/or
change comes in the form of stricter injured wild animals of several species are
permissible emission norms.For a cleaner hand-raised and/or treated and subsequently
environment and to address the growing returned to the wild.A joint venture between
concerns about global warming, the Indian Wildlife Trust of India, the International Fund
government has fast-forwarded the BS6 for Animal Welfare and the Assam Forest
pollution norms to 1 April 2020. Hence, the Department, CWRC was constructed with the

e
jump from BS4 to BS6 instead of moving to BS5 support of the Animal Welfare Division,

n
and then to BS6. BSVI or BS6 is the sixth Government of India. It is recognised by the

li
emission benchmark or criterion in reducing Centre Zoo Authority (CZA).
pollution levels when compared to the BSIV or ● Statement 2 is not correct : Strategically

n
BS4 emission norms. Both are the maximum located adjacent to the Panbari Reserve
o
emission norms which set the maximum Forest near Kaziranga National Park in Assam,
l.
pollution from the exhaust of the motor the centre attends to a wide range of wildlife
a

vehicle.Petrol vehicles that meet BS6 emergencies resulting from natural or


ri

standards must emit no more than 60mg/km anthropogenic causes. Kaziranga National Park
of NOx (Nitrogen Oxides). In the case of the Kaziranga National Park in Assam is a World
te

BS4 vehicle, the limit was 80mg/km Heritage Site, notified by the UNESCO in 1985.
● Statement 3 is correct : The BS6 emission The Park holds the world's largest population
a

norm is not only for motor vehicles but also of the greater one-horned rhinoceros and
m

for the fuel being dispensed. Sulphur and provides refuge to a large number of wild
c

nitrogen oxide plays an important role in petrol animals including the endangered Royal Bengal
and diesel. Compared to the BS4, the BS6 fuel tiger and the Asian elephant. NH37 separates
s

has less sulphur and NOx. While the content of Kaziranga's low-lying flooded grasslands from
p

sulphur in BS4 fuel is 50ppm, it is five times the wooded highlands of the Karbi Anglong
.u

lower in the BS6 fuel, which is at 10ppm. As for hills to the south.
the nitrogen oxide, the permitted level has
w

been reduced by 70% and 25% for diesel and Reference :


w

petrol engines, respectively. https://www.thehindu.com/news/national/other-


states/kaziranga-wildlife-rehab-centre-completes-20-
w

years/article65821438.ece

DPP 2023 DAY 160 68


https://upscmaterial.online/
Download From - https://upscmaterial.online/

Contact us :info@onlyias.com

OnlyIAS Nothing Else Visit :dpp.onlyias.in


Contact : +91-7007 931 912

Q.86) Ans: D promote the services trade. Recently, India


Exp: hosted the 1st Session of “India-Mauritius
● Option 4 is correct : Sawa lake : This year, for High-Powered Joint Trade Committee” under
the first time in its centuries-long history, Iraq the “India-Mauritius Comprehensive Economic
Lake Sawa dried up. A combination of Cooperation and Partnership Agreement
mismanagement by local investors, (CECPA)”.
government neglect and climate change has ● Statement 2 is not correct : Singapore
ground down its azure shores to chunks of (27.01%) and USA (17.94%) have emerged as
salt.SawaLake is an endorheic basin located in top 2 sourcing nations in FDI equity flows into
the Iraqi governorate of Muthanna near to the India in FY 2021-22 followed by Mauritius
Euphrates River . This lake has no inlet or (15.98%), Netherland (7.86%) and Switzerland
outlet, but it draws water from the Euphrates (7.31%). CECPA is a kind of free trade pact that
through a system of joint cracks and fissures aims to provide an institutional mechanism to
which transport water to aquifers beneath it. encourage and improve trade between the
The water’s level fluctuates during dry and wet two countries. It is a limited agreement that
seasons Sawa Lake is characterized by arid will cover only select sectors. It will cover
climate Because of its saline water, no plants sectors such as Trade in Goods, Rules of
grow in the lake or on its shores. Fish and Origin, Trade in Services, Technical Barriers to
algae are the most important aquatic Trade (TBT), Sanitary and Phytosanitary (SPS)
organisms.This lake was known as the “pearl of measures, Dispute Settlement, etc.
the south" . It is an endorheic lake in the

e
Muthanna area, Iraq near to the Euphrates Reference :

n
River. This lake has no inlet or outlet, but it https://pib.gov.in/PressReleasePage.aspx?PRID=1848

li
draws water from the Euphrates through a 300
system of joint cracks and fissures which

n
transport water to aquifers beneath it. The Q.88) Ans: A o
Lake was declared a Ramsar site in 2014. Exp:
l.
● Statement 1 is not correct : Egypt released a
a

Reference : commemorative postage stamp to celebrate


ri

https://timesofindia.indiatimes.com/world/rest-of- 75 years of diplomatic relations between


world/iraqs-pearl-of-the-south-lake-sawa-dry-amid- Egypt and India. Egypt’s recognition of the
te

water-crisis/articleshow/92176133.cms independence of India on 18 August 1947


marked the establishment of ties. It led to the
a

historic Friendship Treaty in 1955. India and


m

Q.87) Ans: A Egypt have had traditionally strong bilateral


Exp:
c

relations.Egypt–India relations are bilateral


● Statement 1 is correct : The India-Mauritius relations between Egypt and India. Modern
s

CECPA is the first trade agreement signed by Egypt–India relations go back to the contacts
p

India with a country in Africa.It is a kind of free between Saad Zaghloul and Mahatma Gandhi
.u

trade pact that aims to provide an on the common goals of their respective
institutional mechanism to encourage and movements of independence.
w

improve trade between the two countries. ● Statement 2 is correct : Egypt was one of the
w

Under this agreement, countries reduce or founding members of the Non-Aligned


eliminate the duties on the products. The Movement (NAM). The preparatory meeting
w

countries also give relaxation in the norms to for the First NAM Conference in Belgrade was

DPP 2023 DAY 160 69


https://upscmaterial.online/
Download From - https://upscmaterial.online/

Contact us :info@onlyias.com

OnlyIAS Nothing Else Visit :dpp.onlyias.in


Contact : +91-7007 931 912

held in Cairo between 5 and 12 June 1961. contribute to sustainable development,


President Gamal Abdel Nasser, together with prosperity, social inclusion and the protection
Josip Broz Tito of Yugoslavia, Jawaharlal Nehru of human life and well-being. It works to
of India, Kwame Nkrumah of Ghana and facilitate the exchange of information
Sukarno of Indonesia played the leading role in internationally and to improve the capacity for
the early days of the movement. Following the decision making in member countries. India
Egyptian revolution of 1952 and president has been a member of ITF, an
Nasser's charismatic authority in other Arab intergovernmental organisation for transport
countries other countries in the Arab world policy, since 2008.
followed Egyptian lead in joining the Non- ● Statement 3 is correct : The ITF maintains the
Aligned Movement. International Road Traffic and Accident
● Statement 3 is not correct : India and Egypt Database (IRTAD), a comprehensive database
also organize bilateral military exercise of statistics related to road safety. IRTAD also
Exercise Desert Warrior. The exercise was acts as a permanent working group of the
conducted to enhance mutual understanding ITF.The IRTAD database contains validated, up-
between the Indian Air Force and the Egyptian to-date crash and exposure data from 32
Air Force. Indian Air Force (IAF) and Royal Air countries: The International Road Traffic and
Force of Oman (RAFO) are taking part in a Accident Database (IRTAD) includes safety and
bilateral exercise names “Eastern Bridge- VI”, traffic data, aggregated by country and year
in a bid to bolster operational efficiency from 1970. All data is collected directly from
through interoperability. relevant national data providers in the IRTAD

e
countries. It is provided in a common format,

n
based on definitions developed and agreed by

li
Reference : the IRTAD Group.
https://m.economictimes.com/news/india/egypt-

n
releases-postage-stamp-celebrating-75th-anniversary- Reference o :
of-diplomatic-ties-with- https://pib.gov.in/PressReleasePage.aspx?PRID=1852
l.
india/articleshow/93669503.cms 524
a
ri

Q.89) Ans: C Q.90) Ans: A


Exp: Exp:
te

● Statement 1 is not correct : The International ● Statement 1 is not correct : India follows the
Transport Forum (ITF) is an inter- One China policy OCP and has no formal ties
a

governmental organisation within the OECD with Taiwan yet.The OCP acknowledges that
m

(Organisation for Economic Co-operation and there is only one Chinese government as
c

Development) system. It is the only global opposed to separate Chinese states. While
body with a mandate for all modes of adhering to the One China policy in joint
s

transport. It acts as a think tank for transport statements till , India never acknowledged
p

policy issues and organises the annual global Taiwan as a part of China.The Taipei Economic
.u

summit of transport ministers. The ITF's motto and Cultural Center in India is the
is "Global dialogue for better transport". representative office of Taiwan in India,
w

● Statement 2 is not correct : The organisation functioning as a de facto embassy in the


w

brings together 64 member countries with the absence of diplomatic relations. It was
aim to advance the global transport policy established in 1995. India is among the 179 of
w

agenda, and ensure that it continues to the 193 member states of the UN that do not

DPP 2023 DAY 160 70


https://upscmaterial.online/
Download From - https://upscmaterial.online/

Contact us :info@onlyias.com

OnlyIAS Nothing Else Visit :dpp.onlyias.in


Contact : +91-7007 931 912

maintain formal diplomatic ties with Taiwan. nuclear non-proliferation regime and an
Recent disputes have compelled India to revisit essential foundation for the pursuit of nuclear
India’s ‘One China Policy’ and there has been a disarmament. It was designed to prevent the
growing sentiment in India that it should spread of nuclear weapons, to further the
establish closer ties with Taiwan. Since , India goals of nuclear disarmament and general and
has stopped mentioning the OCP and has complete disarmament, and to promote
urged China to uphold the One India Policy. cooperation in the peaceful uses of nuclear
● Statement 2 is correct : India has signed a energy
Double-Taxation Avoidance Agreement and a ● Statement 2 is correct : 191 states have
Bilateral Investment Treaty with Taiwan. become parties to the treaty, though North
Taiwan-India relationship has made significant Korea, which acceded in 1985 but never came
progress for the past decades. The two into compliance, announced its withdrawal
countries signed “Double Taxation Avoidance from the NPT in 2003, following detonation of
Agreement” and “Customs Mutual Assistance nuclear devices in violation of core obligations.
Agreement” in 2011 which further enhanced Four UN member states have never accepted
the economic and trade relationship in the NPT, three of which possess or are
particular for both sides.The Double Taxation thought to possess nuclear weapons: India,
Avoidance Agreement or DTAA is a tax treaty Israel, and Pakistan. In addition, South Sudan,
signed between India and another country ( or founded in 2011, has not joined.
any two/multiple countries) so that taxpayers ● Statement 3 is not correct :The NPT was
can avoid paying double taxes on their income opened for signature in 1968 and entered into

e
earned from the source country as well as the force on 5 March 1970. On 11 May 1995, the

n
residence country. At present, India has double Treaty was extended indefinitely. With 191

li
tax avoidance treaties with more than 80 States parties, it is the most widely adhered to
countries around the world. treaty in the field of nuclear non-proliferation,

n
peaceful uses of nuclear energy and nuclear
o
Reference : disarmament.The Treaty represents the only
l.
https://www.indiatoday.in/world/story/taiwan-seeks- binding commitment in a multilateral treaty to
a

india-support-for-entry-into-interpol-china-blocks- the goal of disarmament by the nuclear-


ri

move-1990190-2022-08-19 weapon States.


te

Reference :
Q.91) Ans: B https://www.thehindu.com/news/national/not-
a

Exp: signing-the-npt-one-of-indias-most-consequential-
m

● Statement 1 is not correct :The Treaty on the decisions-former-envoy/article65635838.ece


c

Non-Proliferation of Nuclear Weapons,


commonly known as the Non-Proliferation
s

Treaty or NPT, is an international treaty whose Q.92) Ans: B


p

objective is to prevent the spread of nuclear Exp:


.u

weapons and weapons technology, to ● Statement 1 is correct : Essential Commodities


promote cooperation in the peaceful uses of act was enacted,to stop hoarding and black
w

nuclear energy, and to further the goal of marketing of food and gives consumers
w

achieving nuclear disarmament and general protection against irrational spikes in prices of
and complete disarmament. The Treaty is essential commodities. The ECA was enacted
w

regarded as the cornerstone of the global way back in 1955. It has since been used by the

DPP 2023 DAY 160 71


https://upscmaterial.online/
Download From - https://upscmaterial.online/

Contact us :info@onlyias.com

OnlyIAS Nothing Else Visit :dpp.onlyias.in


Contact : +91-7007 931 912

Government to regulate the production, through auction to ensure transparency and


supply and distribution of a whole host of non-discrimination in the allocation of the
commodities it declares ‘essential’ in order to mineral wealth of the country. The auction
make them available to consumers at fair regime has matured since then. The Act
prices.The list of items under the Act include provides for a non-exclusive reconnaissance
drugs, fertilisers, pulses and edible oils, and permit (for minerals other than coal, lignite,
petroleum and petroleum products. and atomic minerals). Reconnaissance means
● Statement 2 is not correct : There is no specific preliminary prospecting of a mineral through
definition of essential commodities in the certain surveys.
Essential Commodities Act, 1955. Section 2(A) ● Statement 2 is correct : The entry at serial No.
of the Act states that an “essential commodity” 54 of List I (Central List) mandates the central
means a commodity specified in the Schedule government to own the minerals within the
of the Act.By declaring a commodity as exclusive economic zone of India (EEZ). In
essential, the government can control the pursuance to this Mines & Minerals
production, supply, and distribution of that (Development and Regulation) (MMDR) Act of
commodity, and impose a stock limit.If the 1957 was framed. The central government has
Centre finds that a certain commodity is in the ownership over all offshore minerals (ie,
short supply and its price is spiking, it can notify minerals extracted from the sea or ocean
stock-holding limits on it for a specified floor in the Indian maritime zones such as the
period.The States act on this notification to territorial waters, continental shelf and
specify limits and take steps to ensure that exclusive economic zones).

e
these are adhered to. ● Statement 3 is not correct : Captive mines are

n
● Statement 3 is not correct : The Act gives mines owned by companies. The coal or

li
powers to the central government to add or mineral produced by these mines is for the
remove a commodity in the Schedule. The exclusive use of the mine's owner company.

n
Centre, if it is satisfied that it is necessary to do Non-captive mines are mines where the
o
so in the public interest, can notify an item as produced coal or minerals can be used for
l.
essential, in consultation with state their own use as well as for sale. Captive
a

governments.Under the Act, the government Mines: Captive mines are the mines that are
ri

can also fix the maximum retail price (MRP) of owned by companies. The coal or mineral
any packaged product that it declares an produced from these mines is for the exclusive
te

“essential commodity”. use of the owner company of the mines. The


company cannot sell coal or mineral outside.
a

Some electricity generation companies used to


m

Reference : have captive mines. Non- Captive Mines: Non-


c

https://www.thehindu.com/business/Economy/to- captive Mines are mines from which the


rein-in-tur-dal-price-surge-centre-invokes-essential- produced coals of minerals could be used for
s

commodities-act/article65761631.ece its own consumption and as well as for selling


p

it. Mines and Minerals (Development and


.u

Q.93) Ans: A Regulation) Act, 1957 empowered central to


Exp: reserve any mine for the particular end-use.
w

● Statement 1 is not correct : The Mine and These were the captive mines. Now, the Mines
w

Minerals Development and Regulation and Minerals (Development and Regulation)


(MMDR) Act was amended in 2015 to usher in Amendment Bill, 2021 removed the
w

a new regime of granting mineral concessions

DPP 2023 DAY 160 72


https://upscmaterial.online/
Download From - https://upscmaterial.online/

Contact us :info@onlyias.com

OnlyIAS Nothing Else Visit :dpp.onlyias.in


Contact : +91-7007 931 912

distinction. Now captive mines will also be able the budget low. Despite a clear mandate of the
to sell their stock. MGNREGA Act that the wage remuneration
cannot be lower than the minimum wage in
Reference : each State, the former remains much below
https://www.thehindu.com/news/national/kerala/tra the latter. This has led to the violation of the
de-unions-in-kerala-to-take-out-protest-march-to- provisions of the Act, as well as the
parliament-on-august-2/article65699177.ece fundamental rights of MGNREGA workers.

Q.94) Ans: C Reference : https://www.business-


Exp: standard.com/article/economy-policy/demand-for-
● Statement 1 is not correct : The scheme was mgnrega-word-drops-in-monsoon-deficit-states-by-
introduced as a social measure that guarantees 35-in-july-122082300392_1.html
“the right to work”. The key tenet of this social
measure and labour law is that the local
government will have to legally provide at least Q.95) Ans: B
100 days of wage employment in rural India to Exp:
enhance their quality of life.Mahatma Gandhi ● Statement 1 is not correct : Launched in 2014
National Rural Employment Generation under the Ministry of Finance, PMJDY is a
Scheme is a centrally sponsored scheme Central Sector Scheme. The Objective of
implemented by Ministry of Rural PMJDY is to Ensuring access to various
Development (MRD). (MGNREGA) has made financial services to the excluded sections i.e.

e
immense contributions towards livelihood weaker sections & low income groups at an

n
security and the creation of durable assets in affordable cost and using the technology for
rural areas. It has been a valuable employment

li
the same. Pradhan Mantri Jan-Dhan Yojana
tool and safety net as seen in the migrant crisis. (PMJDY) is National Mission for Financial

n
● Statement 2 is not correct : Unlike earlier Inclusion to ensure access to financial services,
o
employment guarantee schemes, MGNREGA namely, Banking/ Savings & Deposit Accounts,
l.
aims at addressing the causes of chronic Remittance, Credit, Insurance, Pension in an
a

poverty through a rights-based framework. At affordable manner.Ensure access of financial


ri

least one-third of beneficiaries have to be products & services at an affordable cost . Use
women as mandated by the act. As per the of technology to lower cost & widen reach.
te

Para 15 of Schedule-II of the Mahatma Gandhi ● Statement 2 is correct : Under the scheme, a
National Rural Employment Guarantee Act, basic savings bank deposit (BSBD) account can
a

“Priority shall be given to women in such a way be opened in any bank branch or Business
m

that at least one-third of the beneficiaries shall Correspondent (Bank Mitra) outlet, by
be women who have registered and requested
c

persons not having any other account. The


for work. basic financial services under the scheme
s

● Statement 3 is correct : Wages must be paid include Banking/ Savings & Deposit Accounts,
p

according to the statutory minimum wages Remittance, Credit, Insurance, Pension in an


.u

specified for agricultural labourers in the state affordable manner.


under the Minimum Wages Act, 1948. ● Statement 3 is not correct : As per extant
w

According to the NREGA ‘At a Glance’ report, Reserve Bank of India guidelines, a PMJDY
w

the average MGNREGA wages paid in FY 21-22 account is treated as inoperative if there are
remain at a meagre ₹209 per day. The official no customer induced transactions in the
w

MGNREGA wages also contribute to keeping account for over a period of two years. In

DPP 2023 DAY 160 73


https://upscmaterial.online/
Download From - https://upscmaterial.online/

Contact us :info@onlyias.com

OnlyIAS Nothing Else Visit :dpp.onlyias.in


Contact : +91-7007 931 912

August 2021, out of total 43.04 crore PMJDY Control) Order (DPCO) 2013 was notified. The
accounts, 36.86 crore (85.6%) were operative. organization is also entrusted with the task of
Continuous increase in percentage of recovering amounts overcharged by
operative accounts is an indication that more manufacturers for the controlled drugs from
and more of these accounts are being used by the consumers. It also monitors the prices of
customers on a regular basis. decontrolled drugs in order to keep them at
reasonable levels. Functions of National
Reference : Pharmaceutical Pricing Authority : To
https://newsonair.com/2022/08/29/pmjdy- implement and enforce the provisions of the
completes-eight-yearsaccounts-grew-three-fold-from- Drugs (Prices Control) Order in accordance
14-72-crore-in-march-2015-to-46-25-crore-in-aug- with the powers delegated to it; To deal with
2022/ all legal matters arising out of the decisions of
the Authority; To monitor the availability of
drugs, identify shortages, if any, and to take
Q.96) Ans: A remedial steps; To collect/ maintain data on
Exp: production, exports and imports, market share
● Statement 1 is not correct : The National of individual companies, profitability of
Pharmaceutical Pricing Authority (NPPA) is a companies etc, for bulk drugs and
government regulatory agency that controls formulations; To undertake and/ or sponsor
the prices of pharmaceutical drugs in India. relevant studies in respect of pricing of drugs/
National Pharmaceutical Pricing Authority pharmaceuticals; To recruit/ appoint the

e
(NPPA) was constituted vide Government of officers and other staff members of the

n
India Resolution dated 29th August 1997. Authority, as per rules and procedures laid

li
NPPA was set-up as an independent Regulator down by the Government; To render advice to
on 29th August, 1997 for pricing of drugs and the Central Government on changes/ revisions

n
to ensure availability and accessibility of in the drug policy; To render assistance to the
o
medicines at affordable prices. NPPA is neither Central Government in the parliamentary
l.
a Statutory nor a Constitutional Body. The matters relating to the drug pricing.
a

Authority, interalia, has been entrusted with


ri

the task of fixation/revision of prices of Reference :


pharmaceutical products (bulk drugs and https://www.thehindubusinessline.com/specials/puls
te

formulations), enforcement of provisions of e/coming-up-nppa-25/article65787354.ece


the Drugs (Prices Control) Order and
a

monitoring of the prices of controlled and


m

decontrolled drugs in the country. Q.97) Ans: B


c

● Statement 2 is correct : NPPA was constituted Exp:


as an attached office of the Department of ● Statement 1 is not correct : Grameen Udyami
s

Pharmaceuticals (DoP), Ministry of Chemicals Project is a unique multiskilling project, funded


p

and Fertilizers as an independent Regulator for by NSDC that aims to train 450 tribal students
.u

pricing of drugs and to ensure availability and in Madhya Pradesh and Jharkhand. Under the
accessibility of medicines at affordable initiative, the endeavor is to multiskill India’s
w

prices.For the purpose of implementing youth and impart functional skills to them for
w

provisions of Drugs (Prices Control) Order enabling livelihoods. The scheme aims to
(DPCO), powers of Government have been promote skill development and
w

vested in NPPA. Later, the Drugs (Prices entrepreneurship among tribal youth and to

DPP 2023 DAY 160 74


https://upscmaterial.online/
Download From - https://upscmaterial.online/

Contact us :info@onlyias.com

OnlyIAS Nothing Else Visit :dpp.onlyias.in


Contact : +91-7007 931 912

promote Skill India Mission, National Skill provided by the nature of the government of a
Development Corporation (NSDC) recently country, free of cost or by taxing the few
launched Phase II of Grameen Udyami project people to offer smass benefit to the public in
in Ranchi (Jharkhand). general. These commodities or services
● Statement 2 is not correct : It is a unique develop the infrastructure and living standard
multiskilling project, funded by NSDC that aims of a country. Features of Public Goods are •
to train 450 tribal students in Madhya Pradesh Non-Rival: The public goods are non-
and Jharkhand. The project is being competitive, i.e. it can serve many people at
implemented in six states Maharashtra, the same time without hindering the usage of
Rajasthan, Chhattisgarh, Madhya Pradesh, one another. • Non-Excludable: These goods
Jharkhand, and Gujarat.In the first phase of are usually free of cost and can be used by
this training, participants from rural and tribal anyone without any restriction. • Non-
areas of Maharashtra, Rajasthan, Chhattisgarh, Rejectable: The consumption of such goods
Madhya Pradesh and Gujarat were included. cannot be dismissed or unaccepted by the
Under which training of 157 participants in public since it is available collectively to all the
seven groups started in Bhopal, Madhya people. • Free-Riding: The goods categorized
Pradesh in the month of May, 2022 and about under public goods benefit even those who
133 participants completed the training have not paid for it. Such people are termed as
successfully on June 27, 2022. After which its free-riders.
second phase was started in Ranchi. ● Private Goods : Private goods are the
● Statement 3 is correct : National Skill products or services which are manufactured

e
Development Corporation : The National Skill or produced by the companies owned by

n
Development Corporation (NSDC) is a non- entrepreneurs who aim at meeting customer’s

li
profit public limited company established on requirement to earn profits through the
July 31, 2008, under Section 25 of the trading of such goods in the free market.

n
Companies Act, 1956. The Ministry of Finance Following are the various characteristics of
o
established NSDC as a Public Private these goods: • Rival: Private products involve
l.
Partnership (PPP) model. The Government of rivalry or competition among consumers for
a

India, through the Ministry of Skill its usage since consumption by one person
ri

Development and Entrepreneurship (MSDE), will restrict its use by another. • Excludable:
owns 49% of NSDC, while the private sector These goods involve cost, and therefore the
te

owns the remaining 51%. The organisation non-payers are excluded from the
provides funding to develop scalable and consumption. • Rejectable: Private goods can
a

successful initiatives for vocational training. be unaccepted or rejected by the consumers


m

since they have multiple alternatives and the


c

Reference : right to select the product according to their


https://newsonair.gov.in/News?title=Govt-launches- preference. • Traded in Free Market: Such
s

2nd-phase-of-Grameen-Udyami-Project-to-promote- goods can be freely bought and sold in the


p

skill-development-and-entrepreneurship-amongst- market at a given price. • Opportunity Cost:


.u

tribal-youth&id=446299 These goods have an opportunity, i.e. the


consumer has to let go of the benefit from a
w

Q.98) Ans: D similar product while selecting a particular


w

Exp: private commodity.


● Both Statements 1 and 2 are not correct :
w

Public goods are the commodities or services

DPP 2023 DAY 160 75


https://upscmaterial.online/
Download From - https://upscmaterial.online/

Contact us :info@onlyias.com

OnlyIAS Nothing Else Visit :dpp.onlyias.in


Contact : +91-7007 931 912

Reference : Exp:
https://m.economictimes.com/tech/technology/no- ● Statement 1 and statement 2 are correct : The
consideration-to-levy-any-charges-for-upi-services- tax to GDP ratio is the ratio of tax collected
finance-ministry/articleshow/93695449.cms compared to national gross domestic product
(GDP). It gives policymakers and analysts a
parameter that can be used to compare tax
Q.99) Ans: C receipts from year to year. It is used as a
Exp: measure to determine how well the
● Option C is correct : Moonlighting is the act of government controls a country's economic
working at an extra job beyond regular resources. The tax to GDP ratio measures the
working hours, usually without the size of a country's tax revenue compared to its
knowledge of the employer. Since the side GDP. The higher the tax to GDP ratio, the
job was mostly at nighttime or on weekends, better the country's financial position. The
it was referred to as moonlighting. Employers ratio denotes the government's ability to fund
are suspicious of this practice because it can its expenditures. A greater tax to GDP ratio
mean that a worker may not give their indicates that the government can cast a wider
organisation the time it needs, and not give fiscal net. It helps a government become less
any extra time to either organisation. Holidays reliant on borrowing.
and time-off are also meant to rest a worker ● Statement 3 is not correct : India consists of
and improve their efficiency, but taking on one direct taxpayer for every 16 voters
another job could make this difficult. In India, present. Income tax is paid by only 1% of

e
private companies usually do not allow India’s population. India’s Gross tax to GDP

n
holding multiple jobs. Shops and which was 11% in FY19, fell to 9.9% in FY20

li
Establishment Acts of various states restrict and marginally improved to 10.2% in FY21
double employment.In the last two years, (partly due to decline in GDP) and is envisaged

n
coronavirus-induced lockdowns increased the to be 10.8% in FY22, this is much lower than
o
tendency to moonlight among workers in the emerging market economy average of 21
l.
certain industries. This was because apart from percent and OECD average of 34
a

financial insecurity at the time, working from percent.There is the presence of a large
ri

home allowed a few categories of workers to informal/unorganized sector in India which


get more work done, freeing up time for a makes it vulnerable, causing greater tax
te

second job . Also the gig economy concept has evasion. There is greater dominance of the
gained greater legitimacy in recent years, agriculture sector which is exempted from
a

too.Food delivery start-up Swiggy recently paying taxes. There are a high number of
m

announced an “industry-first” policy of disputes between tax authorities and


c

allowing its employees to take up gigs or taxpayers, with one of the lowest proportions
projects outside of their regular employment of recovery of tax arrears. The direct to
s

at the company, during the hours away from indirect tax ratio in India is around 35:65,
p

work. Swiggy calls these new norms the which is lower than most of the OECD
.u

“moonlighting” policy. economies where the ratio is 67:33 in favor of


Reference : https://www.business- direct taxes. There have been a number of
w

standard.com/article/companies/falling-margins-high- generous government policies which benefited


w

attrition-moonlighting-causing-trouble-for-it-sector- the richer private sector by providing various


122082600362_1.html tax exemptions. Another factor that
w

Q.100) Ans: C

DPP 2023 DAY 160 76


https://upscmaterial.online/
Download From - https://upscmaterial.online/

Contact us :info@onlyias.com

OnlyIAS Nothing Else Visit :dpp.onlyias.in


Contact : +91-7007 931 912

contributes to the low tax to GDP ratio is low


per capita income and high poverty.
Reference :
https://indianexpress.com/article/opinion/columns/t
he-wrong-diagnosis-india-tax-gdp-ratio-quality-of-
expenditure-8076901/

e
n
li
n
o
l.
a
ri
te
a
m
c
s
p
.u
w
w
w

DPP 2023 DAY 160 77


https://upscmaterial.online/

You might also like